Giao trinh toan roi rac

95

Transcript of Giao trinh toan roi rac

Create by F4VN Ebook Team !Upload and edit by lythanhthuan --http://free4vn.org-- [email protected]

TrÇn Quèc ChiÕn To¸n rêi r¹c

Ch­¬ng 1 . C¬ së to¸n häc 1 − 1

Ch­¬ng 1 : c¬ së to¸n häc 1.1. Quy n¹p to¸n häc 1.1.1. Nguyªn lý quy n¹p to¸n häc Gi¶ sö r»ng víi mçi sè nguyªn d­¬ng n = 1,2,... ta cã mÖnh ®Ò l«gic S(n) hoÆc ®óng hoÆc sai. Gi¶ thiÕt

a) B­íc c¬ së : S(1) ®óng. b) B­íc qui n¹p : NÕu víi mäi k > 1, S(i) ®óng víi mäi i < k , th× S(k) ®óng. Khi ®ã S(n) ®óng víi mäi n.

Ta cã thÓ minh ho¹ nguyªn lý qui n¹p qua h×nh ¶nh sau : Cho mét d·y viªn bi xÕp theo thø tù 1,2,..., n,... Gi¶ sö viªn bi thø nhÊt cã mµu ®á vµ nÕu víi mäi k > 1, k-1 viªn bi ®Çu mµu ®á th× viªn bi thø k còng mµu ®á. Khi ®ã ta kÕt luËn r»ng tÊt c¶ viªn bi ®Òu mµu ®á. 1.1.2. Bµi to¸n xÕp Tromino Tromino lµ vËt gåm 3 « vu«ng ®¬n vÞ kÝch th­íc 1x1 ghÐp l¹i d¹ng bªn Cho mét h×nh B gåm nhiÒu « vu«ng ®¬n vÞ ghÐp l¹i. Ta nãi h×nh B cã thÓ phñ Tromino nÕu cã thÓ dïng c¸c qu©n Tromino xÕp kÝn h×nh B víi ®iÒu kiÖn Tromino kh«ng chång lªn nhau vµ kh«ng phñ ra ngoµi h×nh B. Bµn cê kÝch th­íc n x n gäi lµ khuyÕt nÕu thiÕu 1 « vu«ng ®¬n vÞ. Ta chøng minh r»ng bµn cê khuyÕt 2n x 2n cã thÓ phñ Tromino. a) B­íc c¬ së : n =1. Tromino chÝnh lµ bµn cê khuyÕt 2x2. Bµn cê khuyÕt 4x4 cã thÓ phñ Tromino nh­ h×nh bªn: b) B­íc qui n¹p : Gi¶ thiÕt r»ng bµn cê khuyÕt 2n−1x2n−1 cã thÓ phñ Tromino. Ta ph¶i chøng minh r»ng bµn cê khuyÕt 2nx2n cã thÓ phñ Tromino. ThËt vËy bµn cê khuyÕt 2nx2n cã thÓ chia thµnh 4 bµn cê khuyÕt con 2n−1x2n−1 nh­ sau : 2n-1 2n−1 2n

TrÇn Quèc ChiÕn To¸n rêi r¹c

Ch­¬ng 1 . C¬ së to¸n häc 1 − 2

Bµn cê con chøa « khuyÕt cã thÓ phñ Tromino theo gi¶ thuyÕt qui n¹p. Qu©n Tromino ë gi÷a lµm cho c¸c bµn cê con kh¸c bÞ khuyÕt. Nh­ vËy còng theo gi¶ thiÕt qui n¹p chóng còng cã thÓ phñ Tromino. Nh­ vËy bµn cê khuyÕt 2nx2n còng ®­îc phñ b»ng c¸c qu©n Tromino. Tæng qu¸t ta cã thÓ chøng minh : Mäi bµn cê khuyÕt n x n víi n2 -1 chia hÕt 3 vµ n ≠ 5 ®Òu cã thÓ phñ Tromino. • Bµi tËp Sö dông nguyªn lÝ qui n¹p to¸n häc chøng minh c¸c ®¼ng thøc sau ®óng víi mäi n nguyªn d­¬ng.

1. 1 + 3 + 5 + ... + (2n − 1) = n2 2. 12 + 22 + ... + n2 = n(n+1)(2n + 1)/6 3. 1(1!) + 2(2!) + ... + n(n!) = (n + 1)! − 1

4. !2

1 +

!32

+ ... + )!1( +n

n = 1 −

)!1(1+n

5. 12 − 22 + 32 − ... + (−1)n+1.n2 = (−1)n+1.n(n+1)

TrÇn Quèc ChiÕn To¸n rêi r¹c

Ch­¬ng 1 . C¬ së to¸n häc 1 − 3

1.2. TËp hîp 1.2.1. C¸c kh¸i niÖm c¬ b¶n • §Þnh nghÜa: Kh¸i niÖm tËp hîp lµ kh¸i niÖm nÒn t¶ng cho to¸n häc còng nh­ øng dông cña nã. TËp hîp ®­îc coi lµ kÕt hîp c¸c ®èi t­îng cã cïng b¶n chÊt (thuéc tÝnh, dÊu hiÖu) chung nµo ®ã. TËp hîp th­êng ®­îc ký hiÖu b»ng c¸c ch÷ c¸i A, B, C , ... C¸c phÇn tö cña tËp hîp ký hiÖu b»ng c¸c ch÷ th­êng a, b, c,... §Ó chØ x lµ phÇn tö cña X ta viÕt : x ∈ X (®äc : x thuéc X ) §Ó chØ x kh«ng ph¶i lµ phÇn tö cña X ta viÕt : x ∉ X (®äc : x kh«ng thuéc X ) TËp kh«ng cã phÇn tö gäi lµ tËp rçng vµ ký hiÖu ∅ • BiÓu diÔn tËp hîp:

• LiÖt kª c¸c phÇn tö : A = { a, b, c } X = { x1, x2, ... , xn }

• BiÓu diÔn tËp hîp b»ng c¸ch m« t¶ tÝnh chÊt : C = { n | n lµ sè ch½n } Y = { x | x lµ nghiÖm ph­¬ng tr×nh x2 + 2x - 5 = 0 } • Lùc l­îng tËp hîp: Sè phÇn tö cña A, ký hiÖu lµ A hoÆc card(A), gäi lµ lùc l­îng cña tËp A. NÕu A < ∞ , ta nãi A lµ tËp h÷u h¹n, nÕu A = ∞ , ta nãi A lµ tËp v« h¹n. Trong ch­¬ng tr×nh nµy ta gi¶ thiÕt c¸c tËp hîp lµ h÷u h¹n . • Quan hÖ bao hµm: Cho hai tËp A, B.

NÕu mçi phÇn tö thuéc A còng thuéc B ta nãi A lµ tËp con cña B (hoÆc A bao hµm trong B) vµ ký hiÖu

A ⊂ B NÕu A kh«ng ph¶i tËp con cña B ta ký hiÖu

A ⊄ B

NÕu A ⊂ B vµ B ⊂ A ta nãi A b»ng B vµ ký hiÖu

A = B

TËp tÊt c¶ tËp con cña A ký hiÖu lµ P(A) • §Þnh lý 1. NÕu A = n , th× P (A) = 2n Chứng minh Quy n¹p theo n. • §Þnh lý 2. Quan hÖ bao hµm cã c¸c tÝnh chÊt sau ®©y. • Ph¶n x¹ ∀ A : A ⊂ A • Ph¶n ®èi xøng ∀A, B : A ⊂ B & B ⊂ A ⇒ A = B • B¾c cÇu ∀A, B, C : A ⊂ B & B ⊂ C ⇒ A ⊂ C

TrÇn Quèc ChiÕn To¸n rêi r¹c

Ch­¬ng 1 . C¬ së to¸n häc 1 − 4

Chứng minh HiÓn nhiªn. 1.2.2. C¸c phÐp to¸n tËp hîp Cho c¸c tËp A vµ B. Ta ®Þnh nghÜa c¸c phÐp to¸n sau. • PhÐp hiÖu: HiÖu cña A vµ B, ký hiÖu A \ B lµ tËp:

A \ B = { x x ∈ A & x ∉ B } • PhÇn bï: Cho tËp X vµ A ⊂ X. PhÇn bï cña A (trong X) lµ tËp

A X = X \ A • PhÐp hîp: Hîp cña A vµ B, ký hiÖu A ∪ B lµ tËp

A ∪ B = { x x ∈ A hoÆc x ∈ B } • PhÐp giao: Giao cña A vµ B, ký hiÖu A ∩ B lµ tËp

A ∩ B = { x x ∈ A & x ∈ B } • Ph©n ho¹ch:

- NÕu A ∩ B = ∅, ta nãi A vµ B rêi nhau.

- NÕu c¸c tËp X1, X2, ... , Xn tho¶

A = X1 ∪ X2 ∪ ... ∪ Xn vµ chóng rêi nhau tõng ®«i mét, ta nãi { X1, X2, ... , Xn } lµ mét ph©n ho¹ch cña tËp hîp A. • §Þnh lý 1 (nguyªn lÝ céng). Gi¶ sö { X1, X2, ... , Xn } lµ mét ph©n ho¹ch cña tËp S. Khi ®ã

S= X1+ X2 + ... + Xn Chứng minh HiÓn nhiªn. • HÖ qu¶ :

A ∪ B = A+ B − A ∩ B • §Þnh lý 2. Cho c¸c tËp A, B, C trong tËp vò trô U, khi ®ã ta cã : a) LuËt kÕt hîp :

( A ∪ B ) ∪ C = A ∪ ( B ∪ C ) ( A ∩ B ) ∩ C = A ∩ ( B ∩ C )

b) LuËt giao ho¸n :

A ∪ B = B ∪ A A ∩ B = B ∩ A

TrÇn Quèc ChiÕn To¸n rêi r¹c

Ch­¬ng 1 . C¬ së to¸n häc 1 − 5

c) LuËt ph©n bè : A ∪ ( B ∩ C ) = (A ∪ B) ∩ (A ∪ C ) A ∩ ( B ∪ C ) = (A ∩ B) ∪ (A ∩ C )

d) LuËt phñ ®Þnh kÐp

A = A

e) LuËt ®èi ngÉu De Morgan:

BABA ∩=∪ & BABA ∪=∩

nn AAAAAA ∩∩∩=∪∪∪ ...... 2121

nn AAAAAA ∪∪∪=∩∩∩ ...... 2121 Chứng minh HiÓn nhiªn 1.2.3. TÝch §Ò-c¸c • §Þnh nghÜa: • TÝch §Ò-c¸c cña hai tËp A, B lµ tËp

A x B = { (a,b) a ∈ A & b ∈ B } • TÝch §Ò-c¸c cña c¸c tËp X1, X2, ... , Xn lµ tËp

X1x X2 x ... x Xn = { (x1, x2, ... , xn) x1∈ X1 & x2 ∈ X2 & ... & xn ∈ Xn } ◊ VÝ dô. Cho A = {a, b} vµ B = {1, 2, 3}. Ta cã

A x B = {(a,1), (a,2), (a,3), (b,1), (b,2), (b,3)} vµ

B x A = {(1,a), (1,b), (2,a), (2,b), (3,a), (3,b)} • §Þnh lý 3. Ta cã

X1x X2 x ... x Xn = X1. X2. ... . Xn • Bµi tËp 1. Chøng minh c¸c tÝnh chÊt sau ®©y. a. ∀ A, B, C : A ⊂ B ⇒ (A \ C) ⊂ (B \ C) b. ∀ A, B, C : A ⊂ B ⇒ (A∩C) ⊂ (B∩C) c. ∀ A, B, C : A ⊄ B ⇒ A \ B ≠ ∅ d. ∀ A, B, C : A ⊄ B vµ B ∩ C = ∅ ⇒ (A ∪ C) ⊄ (B ∪ C) 2. Chøng minh c¸c tÝnh chÊt sau ®©y. a. ∀ A, B : A ⊂ B ⇔ A ∩ B = A & A ∪ B = B b. ∀ A, B : A \ B = A ⇔ B \ A = B c. ∀ A, B, C : A⊂ (B∪C) ⇔ A \ B ⊂ C d. ∀ A,B, C : A⊂ B⊂ C ⇔ A∪B = B∩C

TrÇn Quèc ChiÕn To¸n rêi r¹c

Ch­¬ng 1 . C¬ së to¸n häc 1 − 6

3. Chøng minh c¸c tÝnh chÊt sau ®©y. a. ∀ A, B, C : (A∩B)×C = (A×C)∩(B×C) b. ∀ A, B, C : (A∪B)×C = (A×C)∪(B×C) c. ∀ A, B, C : (A \ B)×C = (A × C) \ (B × C) d. ∀ A, B : A × B = ∅ ⇔ A = ∅ hoÆc B = ∅

TrÇn Quèc ChiÕn To¸n rêi r¹c

Ch­¬ng 1 . C¬ së to¸n häc 1 − 7

1.3. Quan hÖ 1.3.1. §Þnh nghÜa • Quan hÖ hai ng«i R tõ tËp X vµo tËp Y lµ tËp con cña tÝch X x Y. NÕu X = Y th× ta nãi R lµ quan hÖ (hai ng«i) trªn X. NÕu (x,y) ∈ R ta viÕt x R y . • Quan hÖ R trªn X gäi lµ ph¶n x¹ nÕu ∀ x ∈ X : (x,x) ∈ R • Quan hÖ R trªn X gäi lµ ®èi xøng nÕu (x,y) ∈ R ⇒ (y,x) ∈ R • Quan hÖ R trªn X gäi lµ ph¶n ®èi xøng nÕu (x,y) ∈ R & (y,x) ∈ R ⇒ x = y • Quan hÖ R trªn X gäi lµ b¾c cÇu (truyÒn øng) nÕu (x,y) ∈ R & (y,z) ∈ R ⇒ (x,z) ∈ R ◊ VÝ dô. 1. Quan hÖ “®ång nhÊt” I = {(x, x) | x ∈ X} 2. Quan hÖ “chia hÕt” D = {(x, y) ∈ N2 | ∃n ∈ N, y = n.x } 3. Quan hÖ ®ång d­ ‘’modul p’’ M = {(x, y) | ∃n ∈ N, x − y = n.p } 4. Quan hÖ ‘bao hµm’ trªn tËp c¸c tËp con cña tËp X,

B = {(A, B) | A ⊂ B , A, B ∈ P(X) } 5. Quan hÖ ‘’song song’’ P = {(d, e) | d // e, d, e ∈ tËp hîp c¸c ®­êng th¼ng} 1.3.2. Quan hÖ t­¬ng ®­¬ng vµ ph©n ho¹ch • §Þnh nghÜa. Quan hÖ R trªn X gäi lµ t­¬ng ®­¬ng nÕu nã lµ ph¶n x¹, ®èi xøng vµ b¾c cÇu. • §Þnh lý 1. Cho ph©n ho¹ch

S = { X1, X2, ... , Xn } cña tËp X. Ta ®Þnh nghÜa quan hÖ R trªn X nh­ sau

x R y ⇔ ∃ i : x ∈ Xi & y ∈ Xi Khi ®ã R lµ quan hÖ t­¬ng ®­¬ng. Chứng minh HiÓn nhiªn. • §Þnh lý 2. Cho R lµ quan hÖ t­¬ng ®­¬ng trªn X. Víi mçi a ∈ X ta ®Æt

[a] = { x ∈ X | x R a }.

TrÇn Quèc ChiÕn To¸n rêi r¹c

Ch­¬ng 1 . C¬ së to¸n häc 1 − 8

Khi ®ã

S = {[a] a ∈ X } lµ mét ph©n ho¹ch cña X. Chứng minh HiÓn nhiªn. • Ghi chó. C¸c tËp [a] ë ®Þnh lÝ trªn gäi lµ c¸c líp t­¬ng ®­¬ng. TËp hîp c¸c líp t­¬ng ®­¬ng nµy gäi lµ tËp th­¬ng cña tËp X vµ kÝ hiÖu lµ X/~ ◊ VÝ dô 1. X lµ tËp 10 viªn bi hoÆc mµu ®á, hoÆc mµu xanh, hoÆc mµu tr¾ng. Gäi A lµ tËp c¸c viªn bÞ mµu ®á, B lµ tËp c¸c viªn bÞ mµu xanh vµ C lµ tËp c¸c viªn bÞ mµu tr¾ng. HiÓn nhiªn {A,B,C} lµ ph©n ho¹ch cña X vµ chóng còng lµ c¸c líp t­¬ng ®­¬ng cña quan hÖ R sau :

x R y ⇔ x, y cïng mµu. ◊ VÝ dô 2. XÐt quan hÖ ‘’®ång d­ modul p’’ trªn tËp sè nguyªn N. Khi ®ã ∀ k ∈ N, 0 ≤ k < p ta cã [k] = { x | x = q.p + k} vµ N /~ = {[0], [1], ..., [p -1]} 1.3.3. Quan hÖ thø tù • §Þnh nghÜa. Quan hÖ hai ng«i R x¸c ®Þnh trªn tËp X gäi lµ quan hÖ thø tù nÕu cã ®ång thêi c¶ ba tÝnh chÊt ph¶n x¹, ph¶n ®èi xøng vµ b¾c cÇu. Quan hÖ thø tù cã thªm tÝnh chÊt

∀ x, y ∈ X : x R y hoÆc y R x

gäi lµ quan hÖ thø tù toµn phÇn.

NÕu quan hÖ kh«ng cã tÝnh chÊt trªn th× gäi lµ quan hÖ thø tù bé phËn. TËp hîp trªn ®ã cã x¸c ®Þnh mét quan hÖ thø tù gäi lµ tËp hîp s¾p thø tù. Ta dïng kÝ hiÖu ≤ ®Ó chØ quan hÖ thø tù, khi ®ã kÝ hiÖu x ≤ y ®äc lµ ‘’x bÐ h¬n hoÆc b»ng y’’. ◊ VÝ dô. Trªn tËp sè tù nhiªn N, quan hÖ “bÐ h¬n hoÆc b»ng” lµ quan hÖ thø tù toµn phÇn, quan hÖ “chia hÕt” D lµ quan hÖ thø tù bé phËn. • §Þnh nghÜa. Cho A lµ tËp con cña tËp s¾p thø tù < X, ≤ > • PhÇn tö a ∈ A gäi lµ phÇn tö cùc tiÓu (cùc ®¹i) cña tËp A nÕu

∀ x ∈ A, x ≤ a (a ≤ x) ⇒ x = a

• PhÇn tö b ∈ A gäi lµ phÇn tö bÐ nhÊt (lín nhÊt) cña tËp A nÕu

∀ x ∈ A, b ≤ x (x ≤ b)

• PhÇn tö c ∈ X gäi lµ cËn d­íi (cËn trªn) cña tËp A nÕu

∀ x ∈A, c ≤ x (x ≤ c)

• Ký hiÖu Inf(A) = { c ∈ X | c lµ cËn d­íi cña tËp A}

TrÇn Quèc ChiÕn To¸n rêi r¹c

Ch­¬ng 1 . C¬ së to¸n häc 1 − 9

Sup(A) = { c ∈ X | c lµ cËn trªn cña tËp A}

PhÇn tö lín nhÊt (bÐ nhÊt ) cña Inf(A) (Sup(A)), nÕu tån t¹i, gäi lµ cËn d­íi ( cËn trªn ) ®óng cña

tËp A vµ kÝ hiÖu lµ ∧(A) ( ∨(A)). Mét tËp cã thÓ cã nhiÒu cùc tiÓu (cùc ®¹i), tuy nhiªn kh«ng ph¶i lóc nµo nã còng cã phÇn tö bÐ nhÊt (lín nhÊt). • §Þnh lý 3. Cho A lµ tËp con cña tËp s¾p thø tù ( X, ≤ ). NÕu tËp A cã phÇn tö bÐ nhÊt (lín nhÊt), th× phÇn tö ®ã lµ duy nhÊt. Chøng minh Gi¶ sö, tr¸i l¹i, tËp A cã c¸c phÇn tö bÐ nhÊt lµ b vµ b ’. Theo ®Þnh nghÜa ta cã b ≤ b’ vµ b’ ≤ b Do tÝnh ph¶n ®èi xøng suy ra b = b’ T­¬ng tù chøng minh ®èi víi phÇn tö lín nhÊt . • HÖ qu¶. PhÇn tö cËn d­íi ®óng ∧(A) (cËn trªn ®óng ∨(A)) nÕu tån t¹i lµ duy nhÊt . ◊ VÝ dô. XÐt quan hÖ “chia hÕt” trªn tËp hîp M = N \ {0,1}. 1. TËp A = {2, 4, 6} cã cùc tiÓu lµ 2 vµ còng lµ phÇn tö bÐ nhÊt. Cã c¸c cùc ®¹i lµ 4 vµ 6 nh­ng kh«ng cã phÇn tö lín nhÊt. 2. Inf (A) ={2}, ∧(A) = 2 vµ Sup(A) = {12k | k ∈ M}, ∨(A) = 12 3. TËp M cã c¸c phÇn tö cùc tiÓu lµ c¸c sè nguyªn tè, kh«ng cã c¸c phÇn tö bÐ nhÊt, phÇn tö cùc ®¹i, phÇn tö lín nhÊt. • Bµi tËp 1. T×m quan hÖ hai ng«i cã tÝnh chÊt. a. Ph¶n x¹ vµ b¾c cÇu nh­ng kh«ng ph¶n ®èi xøng b. Ph¶n x¹ vµ ®èi xøng nh­ng kh«ng b¾c cÇu c. §èi xøng vµ b¾c cÇu nh­ng kh«ng ph¶n x¹ d. Ph¶n x¹ vµ ®èi xøng nh­ng kh«ng b¾c cÇu 2. Cho R lµ quan hÖ hai ng«i x¸c ®Þnh trªn tËp X. KÝ hiÖu R−1 = {(x, y) | (y, x) ∈ R}. T×m c¸c tÝnh chÊt cña R−1 theo c¸c tÝnh chÊt cña R. 3. Chøng minh r»ng quan hÖ ®ång d­ modul 5 lµ quan hÖ t­¬ng ®­¬ng trªn tËp N. T×m líp t­¬ng ®­¬ng [2] vµ tËp th­¬ng N /~ . 4. Chøng minh r»ng quan hÖ chia hÕt lµ quan hÖ thø tù trªn tËp hîp N \ {0,1}. T×m c¸c phÇn tö cùc ®¹i, cùc tiÓu, bÐ nhÊt vµ lín nhÊt. 5. Trªn tËp X = {2, 3, 4, 5, 6, 7, 8, 9, 10} cã quan hÖ chia hÕt D vµ A = {2, 4, 6}. T×m c¸c phÇn tö cùc ®¹i, cùc tiÓu, bÐ nhÊt, lín nhÊt cña A vµ X.

TrÇn Quèc ChiÕn To¸n rêi r¹c

Ch­¬ng 1 . C¬ së to¸n häc 1 − 10

1.4. ¸nh X¹ 1.4.1. §Þnh nghÜa Cho tËp X, Y vµ quan hÖ R ⊂ X x Y. MiÒn x¸c ®Þnh cña R trªn X ®­îc ®Þnh nghÜa vµ ký hiÖu lµ

dom(R) = { x ∈ X ∃ y ∈ Y : (x,y) ∈ R} Quan hÖ f ⊂ X x Y gäi lµ ¸nh x¹ tõ X vµo Y nÕu: i) dom(f) = X ii) if (x,y) ∈ f & (x,y’) ∈ f , th× y = y’ • Ký hiÖu f : X → Y vµ y = f(x) nÕu (x,y) ∈ f . TËp X gäi lµ tËp nguån (miÒn x¸c ®Þnh), tËp Y lµ tËp ®Ých (miÒn gi¸ trÞ) cña ¸nh x¹ f. PhÇn tö y gäi lµ ¶nh cña phÇn tö x qua ¸nh x¹ f. Hai ¸nh x¹ f : X → Y vµ g : X→ Y gäi lµ b»ng nhau, kÝ hiÖu lµ f ≡ g, nÕu

∀ x ∈ X, f(x) = g(x)

◊ VÝ dô 1 Quan hÖ “®ång nhÊt” I = {(x, x) | x ∈ X} x¸c ®Þnh trªn tËp X kh«ng rçng bÊt kú lµ ¸nh x¹ IdX : X→X , IdX(x) = x ∀ x ∈X ◊ VÝ dô 2 Quan hÖ “ lÊy tæng” S = {(x, y, z) | x + y = z } x¸c ®Þnh trªn tËp sè tù nhiªn N lµ ¸nh x¹ s : N x N → N , s(x, y) = x + y ∀ (x,y) ∈ N2

◊ VÝ dô 3 Cho X = {1, 2, 3, 4} vµ Y = {a, b, c, d, e}. Quan hÖ f = {(1, a), (2, a), (3, c), (4, d)} lµ ¸nh x¹ f : X → Y, f(1) = a, f(2) = a, f(3) = c, f(4) = d Quan hÖ g = {(1, a), (2, b), (3, c), (4, d)} lµ ¸nh x¹ g : X → Y, g(1) = a, g(2) = b, g(3) = c, g(4) = d • Cho ¸nh x¹ f : X → Y vµ c¸c tËp con A ⊂ X, B ⊂ Y. TËp f(A) = { f(x) | x ∈ A} gäi lµ ¶nh cña tËp A. TËp f−1(B) = { x ∈ X | f(x) ∈ B} gäi lµ t¹o ¶nh toµn phÇn cña tËp B. §Æc biÖt Im(f) = f(X) vµ Dom(f) = f−1(Y). ◊ VÝ dô 4

Trong vÝ dô 3 ë trªn chän A = {2, 3} vµ B = {a, c}. Ta cã

f(A) = {a, c}, f−1(B) = {1, 2, 3}, f−1(e) = ∅, Im(f) = {a, c, d} • §Þnh lý 1. Cho ¸nh x¹ f : X → Y vµ A, B lµ tËp con cña X cßn C, D lµ tËp con cña Y. Khi ®ã 1. f(A∪B) = f(A) ∪ f(B)

2. f(A∩B) ⊂ f(A) ∩ f(B)

3. f−1(C∪D) = f−1(C) ∪ f−1(D) 4. f−1(C∩D) = f−1(C) ∩ f−1(D) Chứng minh Ta chøng minh tÝnh chÊt 1:

TrÇn Quèc ChiÕn To¸n rêi r¹c

Ch­¬ng 1 . C¬ së to¸n häc 1 − 11

∀ y: y ∈ f(A∪B) ⇔ y ∈ f(A) ∪f(B)

ThËt vËy y ∈ f(A∪B) ⇔ ∃ x ∈ A∪B sao cho y = f(x) ⇔ ( ∃ x ∈ A : y = f(x)) hoÆc ( ∃ x ∈ B : y = f(x))

⇔ y ∈ f(A) hoÆc y ∈ f(B) ⇔ y ∈ f(A) ∪ f(B)

T­¬ng tù chøng minh c¸c tÝnh chÊt cßn l¹i. • D·y c¸c phÇn tö cña tËp X lµ hµm f tõ tËp {1, 2, 3, ...} vµo X. Ký hiÖu fn = f(n) ∀ n ∈ {1, 2, 3, ...} ta cã d·y

f1, f2, ..., fn ,... • ¸nh x¹ f : X → Y gäi lµ ®¬n ¸nh nÕu

∀x, x’ ∈ X : f(x) = f(x’) ⇒ x = x’ ◊ Ghi chó: NÕu tån t¹i ®¬n ¸nh f : X → Y , th× X ≤ Y • ¸nh x¹ f : X → Y gäi lµ toµn ¸nh nÕu

∀ y ∈ Y ∃ x ∈ X : y = f(x) ◊ Ghi chó: NÕu tån t¹i toµn ¸nh f : X → Y , th× Y ≤ X • ¸nh x¹ f : X → Y gäi lµ song ¸nh nÕu f võa lµ ®¬n ¸nh võa lµ toµn ¸nh. ◊ Ghi chó: NÕu tån t¹i song ¸nh f : X → Y , th× X = Y ◊ VÝ dô 5. ¸nh x¹ “®ång nhÊt” IdX ë vÝ dô 1 lµ song ¸nh. ¸nh x¹ “lÊy tæng” s ë vÝ dô 2 lµ toµn ¸nh. ¸nh x¹ g ë vÝ dô 3 kh«ng lµ ®¬n ¸nh vµ còng kh«ng lµ toµn ¸nh. ¸nh x¹ f ë vÝ dô 3 lµ song ¸nh. 1.4.2. C¸c phÐp to¸n ¸nh x¹ • §Þnh nghÜa. Cho ¸nh x¹ f : X → Y, x |→ f(x) vµ ¸nh x¹ g : Y → Z, y|→ g(y) . ¸nh x¹ h : X → Z, x → h(x) = g[f(x)] gäi lµ tÝch cña ¸nh x¹ f vµ ¸nh x¹ g vµ ký hiÖu lµ h = g o f. ◊ VÝ dô 6. Cho c¸c ¸nh x¹ f, g : R → R víi f(x) = 2x + 1 vµ g(x) = sin(x) ∀ x ∈ R. Ta cã

(g o f)(x) = sin(2x + 1) & (f o g)(x) = 2sin(x) + 1 • §Þnh lý 2. TÝch c¸c ¸nh x¹ cã tÝnh kÕt hîp

h o (g o f) = (h o g) o f Chøng minh

TrÇn Quèc ChiÕn To¸n rêi r¹c

Ch­¬ng 1 . C¬ së to¸n häc 1 − 12

Gi¶ sö f : X → Y, g : Y → Z vµ h : Z → V. Ta chøng minh r»ng ∀ x ∈ X, [h o (g o f)](x) = [(h o g) o f](x)

ThËt vËy ∀ x ∈ X, [h o (g o f)](x) = h[(g o f)(x)] = h[g[f(x)]] ∀ x ∈ X, [(h o g) o f](x) = (h o g)[f(x)] = h[g[f(x)]]

Tõ ®ã suy ra ∀ x ∈ X, [h o (g o f)](x) = [(h o g) o f](x)

• §Þnh lý 3. Cho f : X → Y, g : Y → Z vµ h = g o f : X → Z. Khi ®ã 1. NÕu f vµ g lµ ®¬n ¸nh (toµn ¸nh) th× h lµ ®¬n ¸nh (toµn ¸nh) 2. NÕu h lµ ®¬n ¸nh (toµn ¸nh) th× f lµ ®¬n ¸nh (toµn ¸nh) 3. NÕu h lµ ®¬n ¸nh vµ f lµ toµn ¸nh th× g lµ ®¬n ¸nh 4. NÕu h lµ toµn ¸nh vµ g lµ ®¬n ¸nh th× f lµ toµn ¸nh Chøng minh Cho f vµ g ®¬n ¸nh, ta chøng minh h còng ®¬n ¸nh, tøc lµ

∀ x1 , x2 ∈ X, h(x1) = h(x2) ⇒ x1 = x2

ThËt vËy

h(x1) = h(x2) ⇒ g[f(x1)] = g[f(x2)] ⇒ f(x1) = f(x2) ⇒ x1 = x2 Suy ra h lµ ®¬n ¸nh. T­¬ng tù chøng minh c¸c tÝnh chÊt cßn l¹i. • §Þnh nghÜa. ¸nh x¹ f : X → Y gäi lµ ¸nh x¹ kh¶ nghÞch nÕu cã ¸nh x¹ g : Y → X sao cho g o f = IdX vµ f o g = IdY. Trong tr­êng hîp ®ã ta gäi g ¸nh x¹ ng­îc cña ¸nh x¹ f vµ ký hiÖu lµ g = f−1. Do tÝnh ®èi xøng nªn nÕu g lµ ¸nh x¹ ng­îc cña f th× f còng lµ ¸nh x¹ ng­îc cña g.

◊ VÝ dô 7. C¸c cÆp ¸nh x¹ sau ®©y lµ ¸nh x¹ ng­îc cña nhau. (1) f : R → R, x |→ 2x + 1 vµ g : R → R, x |→ (x − 1)/2 (2) f : {1, 2, 3}→{a, b, c}, f = {(1,a), (2,b), (3,c)} vµ g : {a, b, c}→{1, 2, 3}, g = {(a,1), (b,2), (c,3)} • §Þnh lý 4. ¸nh x¹ f lµ kh¶ nghÞch khi vµ chØ khi f lµ song ¸nh. Chøng minh Gi¶ sö f : X → Y cã ¸nh x¹ ng­îc lµ g : Y → X. Theo ®Þnh nghÜa ta cã

g o f = IdX (i) vµ f o g = IdY (ii) V× IdX lµ ®¬n ¸nh nªn tõ (i) vµ ®Þnh lý 3 suy ra f lµ ®¬n ¸nh. V× IdY lµ toµn ¸nh nªn tõ (ii) vµ ®Þnh lý 3 suy ra f lµ toµn ¸nh. VËy f lµ song ¸nh. Ng­îc l¹i, gi¶ sö f : X → Y lµ song ¸nh. Khi ®ã, víi mäi y ∈ Y, tËp hîp f−1(y) cã ®óng mét phÇn tö. LËp ¸nh x¹

g : Y → X, y α f-1(y)

DÔ dµng thÊy r»ng ¸nh x¹ g x¸c ®Þnh nh­ trªn lµ tháa m·n ®iÒu kiÖn (i) vµ (ii). Tøc lµ ¸nh x¹ f kh¶ nghÞch vµ g lµ ¸nh x¹ ng­îc cña f. • §Þnh lý 5. ¸nh x¹ ng­îc nÕu tån t¹i lµ duy nhÊt. Chøng minh Ta chøng minh r»ng, nÕu g, h : Y → X ®Òu lµ ¸nh x¹ ng­îc cña f : X → Y th× g ≡ h. ThËt vËy, ta cã

TrÇn Quèc ChiÕn To¸n rêi r¹c

Ch­¬ng 1 . C¬ së to¸n häc 1 − 13

g = g o IdY ( IdY lµ ¸nh x¹ ®ång nhÊt trªn Y) = g o (f o h) ( v× h lµ ¸nh x¹ ng­îc cña f) = (g o f) o h ( sö dông tÝnh kÕt hîp cña tÝch ¸nh x¹) = IdX o h (v× g lµ ¸nh x¹ ng­îc cña f) = h

• Bµi tËp 1. Cho c¸c tËp A = {0, 1, 2} vµ B = {a, b, c}. T×m tÊt c¶ ¸nh x¹ a) tõ A vµo B b) ®¬n ¸nh tõ A vµo B c) toµn ¸nh tõ A vµo B d) kh«ng ph¶i lµ ®¬n ¸nh vµ còng kh«ng ph¶i lµ toµn ¸nh tõ A vµo B. 2. Cho f : X → Y lµ ®¬n ¸nh, chøng minh c¸c tÝnh chÊt sau. a. ∀ A, B ⊂ X, f(A ∩ B) = f(A) ∩ f(B) b. ∀ A, B ⊂ X, f(A − B) = f(A) − f(B) c. ∀ A, B ⊂ X, f−1(A − B) = f−1(A) − f−1(B) d. ∀ A, B ⊂ X, A ⊂ B ⇒ f−1(A) ⊂ f−1(B) 3. Ta nãi r»ng X ≤ Y nÕu cã ®¬n ¸nh f : X → Y. Chøng minh r»ng quan hÖ trªn lµ quan hÖ thø tù theo nghÜa tæng qu¸t. 4. Líp häc cã 30 sinh viªn. Trong ®ã cã 15 sinh viªn ®¹t m«n To¸n , 10 sinh viªn ®¹t m«n tin vµ 10 sinh viªn ®¹t c¶ hai m«n. Hái cã bao nhiªu ng­êi kh«ng ®¹t c¶ hai m«n?

TrÇn Quèc ChiÕn To¸n rêi r¹c

Ch­¬ng 1 . C¬ së to¸n häc 1 − 14

1.5. c«ng thøc truy håi 1.5.1. Kh¸i niÖm c«ng thøc truy håi ◊ VÝ dô 1

XÐt bµi to¸n ®Õm sè tËp con P(X) cña tËp X. Gäi s(n) lµ sè tËp con cña tËp cã n phÇn tö . Cho x lµ phÇn tö cña X. T¸ch P(X) ra lµm hai nhãm, nhãm tËp con chøa x vµ nhãm tËp con kh«ng chøa x. Ta cã c«ng thøc

s(n) = 2.s(n−1) ∀n §©y lµ mét c«ng thøc truy håi. • §Þnh nghÜa.

C«ng thøc truy håi cña d·y s(0), s(1), s(2),... lµ ph­¬ng tr×nh x¸c ®Þnh s(n) b»ng c¸c phÇn tö s(0), s(1), s(2), ..., s(n−1) tr­íc nã.

s(n) = F(s(0), s(1), s(2),..., s(n−1)) §iÒu kiÖn ban ®Çu lµ c¸c gi¸ trÞ g¸n cho mét sè h÷u h¹n c¸c phÇn tö ®Çu. Trong vÝ dô trªn ta cã ®iÒu kiÖn ban ®Çu lµ s(0) = 1. 1.5.2. Gi¶i c«ng thøc truy håi b»ng ph­¬ng ph¸p lÆp Néi dung cña ph­¬ng ph¸p nµy lµ thay thÕ liªn tiÕp c«ng thøc truy håi vµo chÝnh nã, mçi lÇn thay bËc n gi¶m Ýt nhÊt 1 ®¬n vÞ, cho ®Õn khi ®¹t gi¸ trÞ ban ®Çu. ◊ VÝ dô 1: Quay l¹i bµi to¸n ®Õm sè tËp con cña tËp X. Ta cã

s(n) = 2.s(n − 1) & s(0) = 1 Ta cã

s(n) = 2.s(n − 1) = 2.2.s(n − 2) = ... = 2.2.....2.s(0) = 2n ◊ VÝ dô 2: Bµi to¸n th¸p Hµ néi. Ph­¬ng ph¸p di chuyÓn c¸c ®Üa nh­ sau:

ChuyÓn n−1 ®Üa tõ cäc 1 sang cäc 2, chuyÓn ®Üa lín nhÊt tõ cäc 1 sang cäc 3, vµ cuèi cïng chuyÓn n−1 ®Üa tõ cäc 2 sang cäc 3. Nh­ vËy ta cã c«ng thøc truy håi tÝnh sè lÇn di chuyÓn ® Üa :

s(n) = 2.s(n−1) + 1 & s(1) = 1 Ta cã s(n) = 2.s(n−1) + 1 = 2(2.s(n−2) + 1) + 1 = 22.s(n−2) + 2 + 1 = 2(2.s(n−3) + 1) + 2 + 1 = 23.s(n−3) + 22 + 2 + 1 ... = 2n-1.s(1) + 2n-2 + 2n-3 + ... + 2 + 1 = 2n-1 + 2n-2 + 2n-3 + ... + 2 + 1 = 2n - 1

TrÇn Quèc ChiÕn To¸n rêi r¹c

Ch­¬ng 1 . C¬ së to¸n häc 1 − 15

1.5.3. Gi¶i c«ng thøc truy håi b»ng ph­¬ng tr×nh ®Æc tr­ng C«ng thøc truy håi tuyÕn tÝnh thuÇn nhÊt bËc k cã d¹ng s(n) = c1.s(n−1) + c2.s(n−2) + ... + ck.s(n−k) , ck ≠ 0 (1) trong ®ã ci , i=1,...,k , lµ c¸c h»ng sè.

§iÒu kiÖn ban ®Çu lµ

s(0) = C0 , s(1) = C1 , ... , s(k−1) = Ck-1 . (2)

Ph­¬ng tr×nh ®Æc tr­ng cña c«ng thøc truy håi (1) lµ tk − c1.t

k−1 − c2.tk−2 − ... − ck = 0 (3)

• §Þnh lý 1: NÕu s1 vµ s2 lµ nghiÖm cña ph­¬ng tr×nh (1) th×

a.s1 + b.s2 còng lµ nghiÖm cña (1) víi mäi h»ng a, b. Chứng minh HiÓn nhiªn. • §Þnh lý 2: NÕu r lµ nghiÖm béi m cña ph­¬ng tr×nh ®Æc tr­ng (3) th×

rn , n.rn , ... , nm−1.rn lµ c¸c nghiÖm cña (1). • §Þnh lý 3: NÕu k = 2 vµ α vµ β lµ hai nghiÖm ph©n biÖt cña (3) th× tån t¹i h»ng a, b sao cho

s(n) = a.αn + b.βn lµ nghiÖm cña (1) vµ tho¶ m·n ®iÒu kiÖn ban ®Çu (2). • §Þnh lý 4: NÕu k = 2 vµ α lµ nghiÖm kÐp cña (3) th× tån t¹i h»ng a, b sao cho

s(n) = a.αn + b.n.αn lµ nghiÖm cña (1) vµ tho¶ m·n ®iÒu kiÖn ban ®Çu (2). ◊ VÝ dô 3: D·y Fibonacci ®­îc ®Þnh nghÜa nh­ sau :

fn = fn -1 + fn -2 & f0 = f1 = 1 Ph­¬ng tr×nh ®Æc tr­ng

t2 − t − 1 = 0 cã hai nghiÖm lµ

TrÇn Quèc ChiÕn To¸n rêi r¹c

Ch­¬ng 1 . C¬ së to¸n häc 1 − 16

α = 2

51+ vµ β =

251−

Nh­ vËy d·y Fibonacci cã d¹ng

fn = a.αn + b.βn Gi¶i hÖ

a + b = 1 & a.α + b.β = 1 ta ®­îc

a = 2

51.5

1 + & b = −

251.

51 −

Cuèi cïng ta cã c«ng thøc :

fn =

1

251.

51

+

+n

1

251.

51

+

−n

◊ Ghi chó: Mét ®iÒu thó vÞ lµ fn lµ sè tù nhiªn, nh­ng l¹i ®­îc biÓu diÔn b»ng mét biÓu thøc v« tØ. • Bµi tËp

Gi¶i c¸c c«ng thøc truy håi sau

1. a(n) = 2.n.a(n-1) víi n ≥ 1, a(0) = 1 2. a(n) = a(n-1) + n víi n ≥ 1, a(0) = 0 3. a(n) = 6.a(n-1) - 8.a(n-2) víi n ≥ 2, a(0) = 1, a(1) = 0 4. a(n) = 7.a(n-1) - 10.a(n-2) víi n ≥ 2, a(0) = 5, a(1) = 16 5. a(n) = a(n-1) + 6.a(n-2) víi n ≥ 2, a(0) = 3, a(1) = 6 6. a(n) = a(n-1) + 1 + 2n-1 víi n ≥ 1, a(0) = 0 7. a(n) = 6.a(n-1) - 9.a(n-2) víi n ≥ 2, a(0) = 1, a(1) = 1

8. )(na = )1( −na + 2. )2( −na víi a(0) = a(1) = 1 (®Æt b(n) = )(na )

9. a(n) = )1()2(

−−

nana

víi a(0) = 0, a(1) = 22

1 (lÊy l«ga hai vÕ vµ ®Æt b(n) =

lg(a(n)) 10. a(n) = - 2.n.a(n-1) + 3.n(n-1).a(n-2) víi n ≥ 2, a(0) = 1, a(2) = 2

TrÇn Quèc ChiÕn To¸n rêi r¹c

Ch­¬ng 1 . C¬ së to¸n häc 1 − 17

1.6. HÖ Sè NHÞ THøC • §Þnh nghÜa. Víi mäi n, k ∈ N, ta ®Þnh nghÜa hÖ sè nhÞ thøc

C(n, k) = )!!.(

!knk

n−

• C¸c tÝnh chÊt c¬ b¶n Víi mäi n, k ∈ N, k ≤ n (i) TÝnh ®èi xøng

C(n, k) = C(n, n−k) C(n, 0) = C(n, n) = 1

(ii) C«ng thøc tam gi¸c Pascal

C(n, k) = C(n−1, k−1) + C(n−1, k) (iii) C«ng thøc gi¶m bËc

k.C(n,k) = n.C(n−1,k−1) • NhÞ thøc Newton Víi n ∈ N, x, y ∈ C ta cã

(x+y)n = C(n,0).xn + C(n,1).xn-1.y +...+ C(n,n−1).x.yn-1 + C(n,n).yn Chứng minh Qui n¹p theo n. • HÖ qu¶. (i) C(n,0) + C(n,1) + ... + C(n,n) = 2n (sè c¸c tËp con cña n phÇn tö lµ 2n ) (ii) C(n,0) − C(n,1) + ... + (−1)nC(n,n) = 0

(iii)

[ ] [ ]∑∑

==

+=2

12

00

)12,()2,(nn

jjjnCjnC = 2n - 1

(sè tËp con ch½n b»ng sè tËp con lÎ). • C«ng thøc Vandermonde Cho a, b, n ∈ N. Ta cã

),(),().,(0

nbaCknbCkaCn

k+=−∑

=

Chứng minh Gäi E lµ tËp cã a+b phÇn tö, A, B ⊂ E rêi nhau, A cã a phÇn tö vµ B cã b phÇn tö. Khi ®ã mçi tæ hîp chËp n cña c¸c phÇn tö trong E lµ mét kÕt hîp cña mét tæ hîp chËp k cña c¸c phÇn tö trong A vµ tæ hîp chËp n−k cña c¸c phÇn tö trong B. Tõ ®ã suy ra c«ng thøc. ¸p dông c«ng thøc cho a = b = n suy ra

TrÇn Quèc ChiÕn To¸n rêi r¹c

Ch­¬ng 1 . C¬ së to¸n häc 1 − 18

• HÖ qu¶: Víi n ∈ N ta cã

),2(),(0

2 nnCknCn

k=∑

=

• Bµi tËp Chøng minh c¸c ®¼ng thøc sau

1. C(p,p) + C(p+1,p) + … + C(n,p) = C(n+1,p+1)

2. C(n,1) + 2.C(n,2) + … + n.C(n,n) = ∑=

n

kknCk

1),(. = n.2n−1

3. C(n,0) + 21

C(n,1) + 31

.C(n,2) + … + 1

1+n

.C(n,n) = ∑= +

n

kknC

k0),(

11

= ( )121

1 1 −+

+n

n

4. C(n,0) + 2.C(n,1) + 22.C(n,2) + … + 2n.C(n,n) = 3n 5. C(2n,2) + C(2n,4) + … + C(2n,2n) = 22n – 1 - 1

TrÇn Quèc ChiÕn To¸n rêi r¹c

Ch­¬ng 1 . C¬ së to¸n häc 1 − 19

• Test 1. Sö dông nguyªn lÝ qui n¹p to¸n häc ®Ó chøng minh mÖnh ®Ò S(n) ®óng víi mäi n ≥ n0 lµ A. Chøng minh trùc tiÕp S(n) víi n ≥ n0 bÊt k× B. (i) Chøng minh S(n0)

(ii) Cho n > n0 bÊt k×. Gi¶ thiÕt S(k) ®óng víi mäi k > n. Chøng minh S(n) ®óng. C. (i) Chøng minh S(n0)

(ii) Cho n > n0 bÊt k×. Gi¶ thiÕt S(k) ®óng víi mäi k < n. Chøng minh S(n) ®óng. D. (i) Chøng minh S(n0)

(ii) Cho n > n0 bÊt k×. Gi¶ thiÕt tån t¹i k < n sao cho S(k) ®óng. Chøng minh S(n) ®óng.

2. Lùc l­îng cña tËp A ∪ B ∪ C lµ

A. card(A) + card(B) + card(C) B. card(A) + card(B) + card(C) – card(A.B.C)

C. card(A) + card(B) + card(C) – card(A.B.C) + card(A.B) + card(B.C) + card(A.C)

D. card(A) + card(B) + card(C) – card(A.B) – card(B.C) – card(A.C) + card(A.B.C)

TrÇn Quèc ChiÕn To¸n rêi r¹c

Ch­¬ng 2 . NhËp m«n tæ hîp 2 − 1

Ch­¬ng 2 : nhËp m«n tæ hîp

2.1. s¬ l­îc lÞch sö Cã thÓ nãi t­ duy tæ hîp ra ®êi tõ rÊt sím. Vµo thêi nhµ Chu Trung quèc ng­êi ta ®· biÕt ®Õn nh÷ng h×nh vu«ng thÇn bÝ. Thêi cæ Hi-l¹p, thÕ kû thø 4 tr­íc C«ng nguyªn, nhµ triÕt häc Kxenokrat ®· biÕt c¸ch tÝnh sè c¸c tõ kh¸c nhau lËp tõ b¶ng ch÷ c¸i cho tr­íc. Nhµ to¸n häc Pitagor vµ häc trß ®· t×m ra ®­îc nhiÒu sè cã tÝnh chÊt ®Æc biÖt. Ch¼ng h¹n 36 kh«ng nh÷ng lµ tæng 4 sè ch½n vµ 4 sè lÎ ®Çu tiªn, mµ cßn lµ tæng lËp ph­¬ng cña 3 sè tù nhiªn ®Çu tiªn

36 = 1+ 2 + 3 + 4 + 5 + 6 + 7 + 8 = 13 + 23 + 33

Tõ ®Þnh lý Pitagor ng­êi ta còng ®· t×m ra nh÷ng sè mµ b×nh ph­¬ng cña nã b»ng tæng b×nh ph­¬ng cña 2 sè kh¸c. C¸c bµi to¸n nh­ vËy ®ßi hái ph¶i cã nghÖ thuËt tæ hîp nhÊt ®Þnh. Tuy nhiªn cã thÓ nãi r»ng, lý thuyÕt tæ hîp ®­îc h×nh thµnh nh­ mét ngµnh to¸n häc míi vµo thÕ kû 17 b»ng mét lo¹t c«ng tr×nh nghiªn cøu cña c¸c nhµ to¸n häc xuÊt s¾c nh­ Pascal, Fermat, Euler, Leibnitz,... C¸c bµi to¸n tæ hîp cã ®Æc tr­ng bïng næ tæ hîp víi sè cÊu h×nh tæ hîp khæng lå. ViÖc gi¶i chóng ®ßi hái mét khèi l­îng tÝnh to¸n khæng lå (cã tr­êng hîp mÊt hµng chôc n¨m). V× vËy trong thêi gian dµi, khi mµ c¸c ngµnh to¸n häc nh­ PhÐp tÝnh vi ph©n, PhÐp tÝnh tÝch ph©n, ph­¬ng tr×nh vi ph©n,.. ph¸t triÓn nh­ vò b·o, th× d­êng nh­ nã n»m ngoµi sù ph¸t triÓn vµ øng dông cña to¸n häc. T×nh thÕ thay ®æi tõ khi xuÊt hiÖn m¸y tÝnh vµ sù ph¸t triÓn cña to¸n häc h÷u h¹n. NhiÒu vÊn ®Ò tæ hîp ®· ®­îc gi¶i quyÕt trªn m¸y tÝnh. Tõ chç chØ nghiªn cøu c¸c trß ch¬i, tæ hîp ®· trë thµnh ngµnh to¸n häc ph¸t triÓn m¹nh mÏ, cã nhiÒu øng dông trong c¸c lÜnh vùc to¸n häc , tin häc ...

TrÇn Quèc ChiÕn To¸n rêi r¹c

Ch­¬ng 2 . NhËp m«n tæ hîp 2 − 2

2.2. bµi to¸n tæ hîp Lý thuyÕt tæ hîp nghiªn cøu viÖc ph©n bè c¸c phÇn tö vµo c¸c tËp hîp, tho¶ m·n yªu cÇu nµo ®ã. Mçi c¸ch ph©n bè nh­ thÕ gäi lµ mét cÊu h×nh tæ hîp. Ta th­êng gÆp c¸c d¹ng bµi to¸n sau: 1. Bµi to¸n tån t¹i: Chøng minh sù tån t¹i cÊu h×nh tæ hîp. 2. Bµi to¸n ®Õm : §Õm sè cÊu h×nh tæ hîp. 3. Bµi to¸n liÖt kª: LiÖt kª c¸c cÊu h×nh tæ hîp. 4. Bµi to¸n tèi ­u: T×m cÊu h×nh tèi ­u theo hµm môc tiªu nµo ®ã.

Chóng ta sÏ t×m hiÓu mét sè bµi to¸n tæ hîp tiªu biÓu. 2.2.1. Bµi to¸n th¸p Hµ néi Bµi to¸n nµy do Edouard Lucas ®­a ra ë cuèi thÕ kû 19 (¤ng còng lµ ng­êi ®­a ra d·y Fibonacci). Bµi to¸n ph¸t biÓu nh­ sau. Cã 3 cäc, cäc thø nhÊt cã n ®Üa kÝch th­íc kh¸c nhau xÕp chång nhau, ®Üa nhá n»m trªn ®Üa lín. H·y chuyÓn c¸c ®Üa tõ cäc thø nhÊt sang cäc thø ba, sö dông cäc trung gian thø hai, sao cho lu«n ®¶m b¶o ®Üa nhá trªn ®Üa lín. H·y ®Õm sè lÇn di chuyÓn ®Üa. T×m ph­¬ng ¸n di chuyÓn ®Üa tèi ­u. Sè lÇn di chuyÓn lµ 2n − 1. Khi n = 64, ta cã sè lÇn di chuyÓn lµ:

18 446 744 073 709 551 615 2.2.2. Bµi to¸n xÕp n cÆp vî chång Bµi to¸n nµy còng do Lucas ®­a ra n¨m 1891. Bµi to¸n ph¸t biÓu nh­ sau. Cã n cÆp vî chång cÇn xÕp vµo bµn trßn sao cho kh«ng cã cÆp nµo ngåi gÇn nhau. Cã bao nhiªu c¸ch xÕp nh­ vËy ? Bµi to¸n nµy dÉn ®Õn viÖc nghiªn cøu mét kh¸i niÖm quan träng lµ sè ph©n bè vµ m·i ®Õn n¨m 1934 míi cã lêi gi¶i . Sè c¸ch xÕp lµ

2.n!.Un trong ®ã Un lµ sè ph©n bè. B¶ng sau cho thÊy sù bïng næ tæ hîp ghª gím cña sè ph©n bè

n = 4 5 6 7 8 9 10 11

TrÇn Quèc ChiÕn To¸n rêi r¹c

Ch­¬ng 2 . NhËp m«n tæ hîp 2 − 3

Un = 2 13 80 579 4 738 43 387 439 792 4 890 741 2.2.3. Bµi to¸n ®­êng ®i qu©n ngùa trªn bµn cê Cho bµn cê vua víi kÝch th­íc 8 x 8 = 64 «. T×m ®­êng ®i cña qu©n ngùa qua tÊt c¶ c¸c «, mçi « chØ 1 lÇn, vµ quay vÒ « xuÊt ph¸t. Ng­êi ta chøng minh tæng qu¸t ®­îc r»ng: Trªn bµn cê vu«ng cã sè c¹nh ch½n lín h¬n hoÆc b»ng 6 bao giê còng tån t¹i ®­êng ®i. §­êng ®i cña Euler (1759) cã tÝnh chÊt: hiÖu c¸c « ®èi xøng qua t©m bµn cê b»ng 32.

37 62 43 56 35 60 41 50 44 55 36 61 42 49 34 59 63 38 53 46 57 40 51 48 54 45 64 39 52 47 58 33 1 26 15 20 7 32 13 22 16 19 8 25 14 21 6 31 27 2 17 10 29 4 23 12 13 9 28 3 24 11 30 5

§­êng ®i cña Beverle (1848) cã tÝnh chÊt : tæng c¸c « trªn cét vµ hµng b»ng 260.

1 30 47 52 5 28 43 54 48 51 2 29 44 53 6 27 31 46 49 4 25 8 55 42 50 3 32 45 56 41 26 7 33 62 15 20 9 24 39 58 16 19 34 61 40 57 10 23 63 14 17 36 21 12 59 38 18 35 64 13 60 37 22 11

TrÇn Quèc ChiÕn To¸n rêi r¹c

Ch­¬ng 2 . NhËp m«n tæ hîp 2 − 4

2.2.4. H×nh vu«ng la tinh H×nh vu«ng la tinh cÊp n lµ h×nh vu«ng gåm c¸c sè 1, 2, ..., n-1, n tho¶ m·n tæng mçi hµng vµ tæng mçi cét ®Òu b»ng nhau vµ b»ng

1 + 2 + ... + n = 2

)1( +nn

H×nh vu«ng la tinh chuÈn cÊp n lµ h×nh vu«ng la tinh cÊp n cã dßng ®Çu vµ cét ®Çu lµ 1, 2, ..., n. B¶ng sau ®©y lµ h×nh vu«ng la tinh chuÈn cÊp 7

1 2 3 4 5 6 7 2 3 4 5 6 7 1 3 4 5 6 7 1 2 4 5 6 7 1 2 3 5 6 7 1 2 3 4 6 7 1 2 3 4 5 7 1 2 3 4 5 6

C«ng thøc tÝnh sè h×nh vu«ng la tinh ®Õn nay vÉn cßn bá ngá. Tuy nhiªn ta cã thÓ lËp ch­¬ng tr×nh liÖt kª tÊt c¶ h×nh vu«ng la tinh chuÈn. D­íi ®©y lµ mét sè gi¸ trÞ

n = 1 2 3 4 5 6 7 ln = 1 1 1 4 56 9 408 16 942 080

( ln lµ sè h×nh vu«ng la tinh chuÈn cÊp n). 2.2.5. H×nh lôc gi¸c thÇn bÝ N¨m 1910 Clifford Adams ®­a ra bµi to¸n h×nh lôc gi¸c thÇn bÝ sau: Trªn 19 « lôc gi¸c h·y ®iÒn c¸c sè tõ 1 ®Õn 19 sao cho tæng theo s¸u h­íng cña lôc gi¸c b»ng nhau (= 38). Sau 47 n¨m trêi kiªn nhÉn cuèi cïng «ng ta ®· t×m ra lêi gi¶i. Nh­ng do s¬ ý ®¸nh mÊt b¶n th¶o «ng ®· tèn thªm 5 n¨m n÷a ®Ó kh«i phôc lêi gi¶i. N¨m 1962 Adams c«ng bè lêi gi¶i. §©y còng lµ lêi gi¶i duy nhÊt.

TrÇn Quèc ChiÕn To¸n rêi r¹c

Ch­¬ng 2 . NhËp m«n tæ hîp 2 − 5

15 14 13

9 8 10 6 4

11 5 12 1 2

18 7 16 17 19

3

TrÇn Quèc ChiÕn To¸n rêi r¹c

Ch­¬ng 3. Bµi to¸n tån t¹i 3 − 1

ch­¬ng 3. bµi to¸n tån t¹i Trong nhiÒu tr­êng hîp viÖc x¸c ®Þnh sù tån t¹i mét cÊu h×nh tho¶ m·n tÝnh chÊt nµo ®ã còng cã ý nghÜa quan träng vÒ mÆt lý thuyÕt còng nh­ thùc tÕ. V× thÕ mét d¹ng bµi to¸n tiÕp theo trong Tæ hîp lµ bµi to¸n tån t¹i : XÐt sù tån t¹i c¸c cÊu h×nh tæ hîp tho¶ m·n c¸c tÝnh chÊt cho tr­íc. Bµi to¸n tån t¹i ®­îc nghiªn cøu tõ rÊt l©u vµ gãp phÇn ®¸ng kÓ thóc ®Èy sù ph¸t triÓn cña lý thuyÕt tæ hîp còng nh­ nhiÒu ngµnh to¸n häc kh¸c. C¸c bµi to¸n sau mét phÇn nµo minh ho¹ ®iÒu ®ã. 3.1. mét sè vÝ dô 3.1.1. Bµi to¸n 36 sÜ quan Bµi to¸n nµy do nhµ to¸n häc Euler ®­a ra, néi dung cña nã nh­ sau. Ng­êi ta triÖu tËp tõ 6 trung ®oµn , mçi trung ®oµn 6 sÜ quan cã 6 cÊp bËc kh¸c nhau : thiÕu uý, trung uý, th­îng uý, ®¹i uý, thiÕu t¸ vµ trung t¸. Hái cã thÓ xÕp 36 sÜ quan nµy thµnh h×nh vu«ng 6 x 6 sao cho trong mçi hµng ngang còng nh­ hµng däc ®Òu cã ®¹i diÖn cña 6 trung ®oµn vµ cã c¶ 6 cÊp bËc kh¸c nhau ? §Ó ®¬n gi¶n ta dïng c¸c ch÷ c¸i lín A, B, C, D, E, F ®Ó chØ 6 trung ®oµn vµ c¸c ch÷ c¸i nhá a, b,c, d, e, f ®Ó chØ 6 cÊp bËc. Bµi to¸n cã thÓ tæng qu¸t ho¸ b»ng c¸ch thay sè 6 b»ng n. Tr­êng hîp n = 4 ta cã mét lêi gi¶i sau :

Ab Dd Ba Cc Bc Ca Ad Db Cd Bd Dc Aa Da Ac Cb Bd

Tr­êng hîp n = 5 ta cã mét lêi gi¶i sau :

Aa Bd Cc Dd Ee Cd De Ea Ab Bc Eb Ac Bd Ce Da Be Ca Db Ec Ad Dc Ed Ae Ba Cb

TrÇn Quèc ChiÕn To¸n rêi r¹c

Ch­¬ng 3. Bµi to¸n tån t¹i 3 − 2

Do lêi gi¶i cña bµi to¸n cã thÓ biÓu diÔn bëi hai h×nh vu«ng víi c¸c ch÷ c¸i hoa vµ th­êng xÕp c¹nh nhau nªn cßn cã tªn gäi lµ bµi to¸n h×nh vu«ng la tinh trùc giao. Euler ®· mÊt nhiÒu c«ng søc ®Ó t×m lêi gi¶i cho bµi to¸n nh­ng kh«ng thµnh c«ng. V× vËy «ng ®­a ra gi¶ thuyÕt r»ng c¸ch xÕp nh­ vËy kh«ng tån t¹i. Gi¶ thuyÕt nµy ®­îc nhµ to¸n häc Ph¸p Tarri chøng minh n¨m 1901 b»ng c¸ch duyÖt tÊt c¶ kh¶ n¨ng xÕp. Dùa trªn gi¶ thuyÕt kh«ng tån t¹i lêi gi¶i cho n = 2 vµ n = 6 Euler cßn ®­a ra gi¶ thuyÕt tæng qu¸t h¬n lµ : Kh«ng tån t¹i h×nh vu«ng la tinh trùc giao cÊp 4k + 2. Gi¶ thuyÕt nµy tån t¹i suèt 2 thÕ kû. M·i ®Õn n¨m 1960 ba nhµ to¸n häc Mü lµ Boce, Parker, Srikanda míi chØ ra mét lêi gi¶i víi n = 10 vµ sau ®ã ®­a ra ph­¬ng ph¸p x©y dùng h×nh vu«ng la tinh trùc giao cÊp 4k + 1 víi k >1. Bµi to¸n cã nhiÒu øng dông trong quy ho¹ch thùc nghiÖm, h×nh häc x¹ ¶nh ... 3.1.2. Bµi to¸n 2n ®iÓm trªn l­íi n x n ®iÓm Cho l­íi « vu«ng gåm n x n ®iÓm. Hái cã thÓ chän trong chóng 2n ®iÓm sao cho kh«ng cã 3 ®iÓm nµo th¼ng hµng hay kh«ng ? HiÖn nay ng­êi ta míi biÕt lêi gi¶i ®èi víi n ≤ 15. D­íi ®©y lµ lêi gi¶i víi n = 12.

• • • • • • • • • • • • • •

• • • • • • • •

• •

TrÇn Quèc ChiÕn To¸n rêi r¹c

Ch­¬ng 3. Bµi to¸n tån t¹i 3 − 3

3.2. Nguyªn lÝ Dirichlet 3.2.1. Nguyªn lÝ Dirichlet (nguyªn lý chim bå c©u) NÕu xÕp nhiÒu h¬n k ®èi t­îng vµo k c¸i hép th× tån t¹i hép chøa Ýt nhÊt 2 ®èi t­îng. ◊ VÝ dô 1. Trong 367 ng­êi bao giê còng cã hai ng­êi trïng ngµy sinh nhËt, bëi v× trong n¨m chØ cã nhiÒu nhÊt 366 ngµy. ◊ VÝ dô 2. M­êi ng­êi cã hä TrÇn, Lª, NguyÔn vµ tªn lµ Hïng, H­ng, Hai. Khi ®ã sÏ cã Ýt nhÊt 2 ng­êi trïng hä vµ tªn, bëi v× chØ cã 9 cÆp hä tªn kh¸c nhau. 3.2.2. Nguyªn lÝ Dirichlet tæng qu¸t NÕu xÕp nhiÒu h¬n N ®èi t­îng vµo k c¸i hép th× tån t¹i hép chøa Ýt nhÊt N/k ®èi t­îng( x lµ sè nguyªn nhá nhÊt ≥ x). ◊ VÝ dô 1. Trong 100 ng­êi bao giê còng cã Ýt nhÊt 9 ng­êi trïng th¸ng sinh. Chøng minh. ThËt vËy, xÕp nh÷ng ng­êi cïng th¸ng sinh vµo 1 nhãm. Cã 12 nhãm tÊt c¶. Ta cã 100/12 = 9. VËy theo nguyªn lý Dirichlet tån t¹i nhãm cã Ýt nhÊt 9 ng­êi. ◊ VÝ dô 2. Trong héi nghÞ cã n ng­êi bao giê còng cã 2 ng­êi cã sè ng­êi quen b»ng nhau. Chøng minh. ThËt vËy, xÕp nh÷ng ng­êi cã sè ng­êi quen b»ng nhau vµ b»ng i vµo cïng 1 nhãm, kÝ hiÖu nhãm i , 0 ≤ i ≤ n - 1. Ta ph©n 2 tr­êng hîp:

(i) Cã 1 ng­êi kh«ng quen ai c¶. Trong tr­êng hîp nµy kh«ng cã ai quen (n−1) ng­êi. V× vËy ta cã tèi ®a (n−1) nhãm 0,1,2,...,n−3,n−2. Nh­ vËy theo nguyªn lý Dirichlet ph¶i tån t¹i nhãm cã Ýt nhÊt 2 ng­êi.

(ii) Ai còng cã ng­êi quen. Ta cã tèi ®a (n−1) nhãm 1,2,...,n−2,n−1. Nh­ vËy

theo nguyªn lý Dirichlet ph¶i tån t¹i nhãm cã Ýt nhÊt 2 ng­êi. ◊ VÝ dô 3. Trong mÆt ph¼ng cho 6 ®iÓm . C¸c ®iÓm ®­îc nèi víi nhau tõng cÆp mét bëi c¸c c¹nh xanh hoÆc ®á. Chøng minh r»ng lu«n tån t¹i tam gi¸c cã c¸c c¹nh cïng mµu. Chøng minh.

TrÇn Quèc ChiÕn To¸n rêi r¹c

Ch­¬ng 3. Bµi to¸n tån t¹i 3 − 4

Chän ®iÓm P bÊt kú tõ 6 ®iÓm. Tõ P cã 5 c¹nh nèi ®Õn 5 ®iÓm cßn l¹i. Nh­ vËy theo nguyªn lý Dirichlet sÏ cã 3 = 5/2 c¹nh cïng mµu PP1,PP2,PP3 . Gi¶ sö ®ã lµ mµu xanh. NÕu c¸c c¹nh P1P2,P2P3,P3P1 mµu ®á th× ta cã ∆P1P2P3 cïng mµu. NÕu mét trong c¸c c¹nh P1P2,P2P3,P3P1 cã mµu xanh , gi¶ sö ®ã lµ PiPj th× ∆PPiPj cïng mµu xanh. ◊ VÝ dô 4. Chøng minh r»ng trong n+1 sè nguyªn d­¬ng ≤ 2n, bao giê còng t×m ®­îc 2 sè chia hÕt cho nhau. Chøng minh ViÕt mçi sè ai trong n+1 sè trªn d­íi d¹ng :

ai = ik2 .qi , i = 1,2,...,n+1 trong ®ã ki lµ sè nguyªn kh«ng ©m, qi lµ sè lÎ. C¸c sè q1,q2,...,qn+1 lµ c¸c sè lÎ ≤ 2n. Trong ®o¹n tõ 1 ®Õn 2n chØ cã n sè lÎ. Nh­ vËy theo nguyªn lý Dirichlet cã hai sè qi vµ qj b»ng nhau. Khi ®ã ai vµ aj sÏ chia hÕt cho nhau.

TrÇn Quèc ChiÕn To¸n rêi r¹c

Ch­¬ng 3. Bµi to¸n tån t¹i 3 − 5

Bµi tËp

1. Mét ®éi bãng thi ®Êu liªn tôc trong 30 ngµy cña th¸ng 9, mçi ngµy Ýt nhÊt 1 trËn vµ tæng sè trËn ®Êu kh«ng qu¸ 45 trËn. Chøng minh r»ng t×m ®­îc sè ngµy liªn tôc nµo ®ã trong th¸ng 9 mµ ®éi ch¬i ®óng 14 trËn. H­íng dÉn: Ký hiÖu ai lµ tæng sè trËn thi ®Êu ®Õn hÕt ngµy i. a1, a2, ..., a30 lµ d·y sè nguyªn d­¬ng t¨ng, 1 ≤ ai ≤ 45. Suy ra ta cã d·y 60 sè trong kho¶ng tõ 1 ®Õn 59.

1 ≤ a1, a2, ..., a30 , a1+14, a2 +14, ..., a30 +14 ≤ 59

Theo nguyªn lý Dirichlet tån t¹i i ≠ j sao cho aj = ai + 14, suy ra tõ ngµy i+1 ®Õn ngµy j cã ®óng 14 trËn. 2. Chøng minh r»ng trong nhãm 10 ng­êi cã 2 ng­êi cã tæng hoÆc hiÖu cña tuæi chia hÕt 16. H­íng dÉn. Ký hiÖu a1, a2, ..., a10 lµ tuæi cña 10 ng­êi. §Æt ri = ai mod 16 , si = ri nÕu ri ≤ 8 vµ si = 16 - ri nÕu ri > 8. s1 , ..., s10 lµ 10 sè tõ 0 ®Õn 8, suy ra tån t¹i i ≠ k tho¶ si = sk 3. Cho song ¸nh f tõ X = {1,2,...,n} vµo X. Ký hiÖu k lÇn hîp cña hµm f víi chÝnh nã lµ

fk = f o f o ... o f Chøng minh r»ng tån t¹i i vµ j tho¶

fi(x) = fj(x) víi mäi x ∈ X. Chøng minh r»ng tån t¹i k tho¶

fk(x) = x víi mäi x ∈ X. H­íng dÉn: f1, f2, ..., fk, ... lµ c¸c song ¸nh tõ X vµo X. Sè song ¸nh h÷u h¹n, suy ra tån t¹i i < j sao cho fi = fj . §Æt k=j-i ta cã fk(x) = x ∀x∈X. 4. Cho bµn cê kÝch th­íc 3 x 7 víi c¸c « ®­îc t« mµu xanh hoÆc ®á. Chøng minh r»ng bµn cê chøa h×nh ch÷ nhËt kh«ng tÇm th­êng (tøc kh«ng cã c¹nh b»ng 1) sao cho c¸c « ë bèn gãc cïng mµu. H­íng dÉn: Gäi 2 « cïng mµu trªn 1 cét lµ cÆp ®ång mµu. Mçi cét cã Ýt nhÊt 1 cÆp ®ång mµu. Cã 4 cét cã cÆp ®ång mµu ®á hoÆc xanh, ®á ch¼ng h¹n. Mçi cét chØ cã 3 tæ hîp cÆp. Suy ra cã hai cÆp ®ång mµu ®á, t¹o 4 ®Ønh h×nh ch÷ nhËt. 5. Cho p sè 1 vµ q sè 0 xÕp thµnh vßng trßn. Gi¶ sö p ≥ k.q , k nguyªn d­¬ng. Chøng minh r»ng cã Ýt nhÊt k sè 1 xÕp liÒn nhau. H­íng dÉn: ¸p dông trùc tiÕp nguyªn lý Dirichlet tæng qu¸t, hoÆc ph¶n chøng. 6. Cho X lµ tËp con n+2 phÇn tö cña tËp {1,2,...,2n+1} vµ m lµ phÇn tö lín nhÊt cña X. Chøng minh r»ng tån t¹i i ≠ j trong X tho¶ i+j = m.

TrÇn Quèc ChiÕn To¸n rêi r¹c

Ch­¬ng 3. Bµi to¸n tån t¹i 3 − 6

H­íng dÉn: §Æt ak = k nÕu k ≤ m/2 vµ ak = m-k nÕu k > m/2 ∀k∈X\{m}. Tån t¹i i ≠ j tho¶ ai = aj . Suy ra i+j=m. 7. Chøng minh r»ng d·y n2 +1 sè kh¸c nhau ai , i =1,...,n2 +1, chøa d·y con n+1 phÇn tö t¨ng hoÆc gi¶m . H­íng dÉn: Gäi bi lµ sè phÇn tö cña chuçi t¨ng dµi nhÊt, ci lµ sè phÇn tö cña chuçi gi¶m dµi nhÊt xuÊt ph¸t tõ ai. Ta cã n2 +1 cÆp (bi,ci) kh¸c nhau. Suy ra ph¶i cã bi > n hoÆc ci > n. 8. ViÕt ch­¬ng tr×nh t×m d·y t¨ng dµi nhÊt cña mét d·y cho tr­íc. 9. Cho 5 ®iÓm trªn mÆt ph¼ng cã to¹ ®é nguyªn. Chøng minh r»ng tån t¹i hai ®iÓm cã trung ®é nguyªn. H­íng dÉn: XÕp lo¹i c¸c ®iÓm theo tÝnh ch¼n lÎ cña to¹ ®é th× cã 4 lo¹i (c,c), (c,l), (l,c) vµ (l,l) (c lµ sè ch½n, l lµ sè lÎ). Khi ®ã cã hai ®iÓm cïng lo¹i, cã trung ®iÓm nguyªn.

TrÇn Quèc ChiÕn To¸n rêi r¹c

Ch­¬ng 4. Bµi to¸n ®Õm 4 − 1

Ch­¬ng 4. bµi to¸n ®Õm 4.1. c¸c nguyªn lý c¬ b¶n 4.1.1. Nguyªn lÝ nh©n ◊ VÝ dô: Mét nhµ hµng cã thùc ®¬n sau:

C Khai vÞ :

1. Salad 2. Sóp C Mãn ¨n chÝnh : 1. ThÞt bß 2. ThÞt lîn 3. C¸ C §å uèng: 1. Trµ 2. S÷a 3. Bia 4. Cola

Cã bao nhiªu c¸ch chän b÷a ¨n gåm 1 mãn khai vÞ, 1 mãn ¨n chÝnh, 1 lo¹i ®å uèng ? Sè c¸ch chän cã thÓ tÝnh nh­ sau:

2 c¸ch chän khai vÞ * 3 c¸ch chän mãn chÝnh * 4 c¸ch chän ®å uèng = 24 • Nguyªn lý nh©n: Gi¶ sö mét cÊu h×nh tæ hîp ®­îc x©y dùng qua k b­íc, b­íc 1 cã thÓ ®­îc thùc hiÖn n1 c¸ch , b­íc 2 cã thÓ ®­îc thùc hiÖn n2 c¸ch ,..., b­íc k cã thÓ ®­îc thùc hiÖn nk c¸ch. Khi ®ã sè cÊu h×nh lµ:

n1. n2 . ... . nk ◊ VÝ dô 1:

i) Cã bao nhiªu chuçi 4 ký tù ®­îc t¹o thµnh tõ c¸c ch÷ c¸i A,B,C,D,E nÕu kh«ng cho phÐp lÆp l¹i ? Ký tù thø nhÊt cã 5 c¸ch chän. Ký tù thø hai cã 4 c¸ch chän. Ký tù thø ba cã 3 c¸ch chän. Ký tù thø t­ cã 2 c¸ch chän.

TrÇn Quèc ChiÕn To¸n rêi r¹c

Ch­¬ng 4. Bµi to¸n ®Õm 4 − 2

Nh­ vËy theo nguyªn lý nh©n sè x©u ký tù kh¸c nhau lµ

5 * 4 * 3 * 2 = 120

ii) Cã bao nhiªu chuçi trong c©u i) b¾t ®Çu b»ng ch÷ B ?

1 * 4 * 3 * 2 = 24

iii) Cã bao nhiªu chuçi trong c©u i) b¾t ®Çu kh«ng ph¶i b»ng ch÷ B ?

120 − 24 = 96 ◊ VÝ dô 2 : Cho tËp S = {x1 , x2 , ..., xn}. §Õm sè tËp con cña S. Gi¶i: Mét tËp con cña S cã thÓ ®­îc x©y dùng trong n b­íc kÕ tiÕp nh­ sau : NhÆt hoÆc kh«ng nhÆt x1 , NhÆt hoÆc kh«ng nhÆt x2 ,..., NhÆt hoÆc kh«ng nhÆt xn . Mçi b­íc ®­îc thùc hiÖn nhiÒu nhÊt lµ 2 c¸ch. Nh­ vËy sè tËp con lµ

2 . 2 . 2 ..... 2 = 2n n lÇn

4.1.2. Nguyªn lý céng ◊ VÝ dô: Cã bao nhiªu chuçi 8 bit b¾t ®Çu b»ng 101 hoÆc 111 ? Gi¶i: Theo nguyªn lý nh©n cã 25 chuçi b¾t ®Çu b»ng 101 vµ cã 25 chuçi b¾t ®Çu b»ng 111. V× hai lo¹i chuçi nµy kh¸c nhau nªn ta cã

2 . 25 = 64 chuçi 8 bit b¾t ®Çu b»ng 101 hoÆc 111. • Nguyªn lý céng: Gi¶ sö { X1, X2, ... , Xn } lµ mét ph©n ho¹ch cña tËp S. Khi ®ã

S= X1+ X2 + ... + Xn ◊ VÝ dô 1: §Õm sè c¸ch chän 2 quyÓn s¸ch chuyªn ngµnh kh¸c nhau tõ 5 quyÓn tin häc kh¸c nhau, 3 quyÓn to¸n kh¸c nhau, vµ 2 quyÓn kinh tÕ kh¸c nhau.

Gi¶i: Theo nguyªn lý nh©n ta cã : 5 . 3 = 15 c¸ch chän 1 quyÓn tin häc, 1 quyÓn to¸n häc 5 . 2 = 10 c¸ch chän 1 quyÓn tin häc, 1 quyÓn kinh tÕ 3 . 2 = 6 c¸ch chän 1 quyÓn to¸n häc, 1 quyÓn kinh tÕ Theo nguyªn lý céng ta cã :

TrÇn Quèc ChiÕn To¸n rêi r¹c

Ch­¬ng 4. Bµi to¸n ®Õm 4 − 3

15 + 10 + 6 = 31

c¸ch chän s¸ch. ◊ VÝ dô 2: Mét héi ®ång gåm cã 6 ng­êi lµ : A, B, C, D, E, F. CÇn chän 1 chñ tÞch, 1 th­ ký vµ 1 thñ quü.

i) Cã bao nhiªu c¸ch chän. ii) Cã bao nhiªu c¸ch chän mµ A hoÆc B lµ chñ tÞch. iii) Cã bao nhiªu c¸ch chän mµ E gi÷ 1 trong c¸c chøc vô trªn. iv) Cã bao nhiªu c¸ch chän mµ c¶ D vµ F ®Òu gi÷ chøc vô.

Gi¶i.

i) Theo nguyªn lý nh©n sè c¸ch chän lµ 6 . 5 . 4 = 120 ii) NÕu A lµ chñ tÞch th× cã 5 . 4 = 20 c¸ch. NÕu B lµ chñ tÞch th× cã 5 . 4 =

20 c¸ch. Theo nguyªn lý céng cã tÊt c¶ 20 + 20 = 40 c¸ch.

iii) NÕu E lµ chñ tÞch th× cã 20 c¸ch. NÕu E lµ th­ ký th× cã 20 c¸ch, NÕu E lµ thñ quü th× cã 20 c¸ch. Theo nguyªn lý céng cã tÊt c¶ 20 + 20 + 20 = 60 c¸ch.

iv) Cã 3 c¸ch g¸n D vµo c¸c chøc vô, 2 c¸ch g¸n F, vµ 4 c¸ch g¸n 1 trong 4

ng­êi cßn l¹i vµo chøc vô cuèi cïng. Theo nguyªn lý nh©n ta cã 3 . 2 . 4 = 24 c¸ch.

TrÇn Quèc ChiÕn To¸n rêi r¹c

Ch­¬ng 4. Bµi to¸n ®Õm 4 − 4

4.2. C¸c cÊu h×nh tæ hîp 4.2.1. ChØnh hîp lÆp • §Þnh nghÜa: Mét chØnh hîp lÆp chËp k cña n phÇn tö lµ mét bé cã thø tù gåm k thµnh phÇn lÊy tõ n phÇn tö ®· cho. C¸c thµnh phÇn cã thÓ ®­îc lÆp l¹i. Mét chØnh hîp lÆp chËp k cña n cã thÓ xem nh­ mét phÇn tö cña tÝch §Ò-c¸c Xk , víi X lµ tËp n phÇn tö. Nh­ vËy sè tÊt c¶ c¸c chØnh hîp lÆp chËp k cña n lµ

nk ◊ VÝ dô: TÝnh sè hµm tõ tËp X cã k phÇn tö ®Õn tËp Y cã n phÇn tö. Mçi hµm tõ X vµo Y t­¬ng øng víi mét bé cã thø tù k thµnh phÇn cña n phÇn tö cña Y, c¸c phÇn tö cã thÓ lÆp l¹i . Nh­ vËy sè hµm tõ X vµo Y lµ nk . 4.2.2. ChØnh hîp kh«ng lÆp • §Þnh nghÜa: Mét chØnh hîp kh«ng lÆp chËp k cña n phÇn tö lµ mét bé cã thø tù gåm k thµnh phÇn lÊy tõ n phÇn tö ®· cho. C¸c thµnh phÇn kh«ng ®­îc lÆp l¹i. Mét chØnh hîp kh«ng lÆp chËp k cña n cã thÓ ®­îc x©y dùng qua k b­íc kÕ tiÕp nh­ sau : Chän thµnh phÇn®Çu : cã n kh¶ n¨ng. Chän thµnh phÇn thø hai : cã n - 1 kh¶ n¨ng. ... Chän thµnh phÇn thø k : cã n - k + 1 kh¶ n¨ng. Nh­ vËy, theo nguyªn lý nh©n, sè tÊt c¶ chØnh hîp kh«ng lÆp chËp k cña n phÇn tö lµ

A(n,k) = n.(n − 1).....(n − k + 1) = )!(!kn

n−

◊ VÝ dô: TÝnh sè hµm ®¬n ¸nh tõ tËp X cã k phÇn tö ®Õn tËp Y cã n phÇn tö. Gi¶i : Mçi hµm ®¬n ¸nh tõ X vµo Y t­¬ng øng víi mét chØnh hîp kh«ng lÆp chËp k cña n phÇn tö cña Y. Nh­ vËy sè cÇn t×m lµ A(n,k) = n.(n−1).....(n−k+1).

TrÇn Quèc ChiÕn To¸n rêi r¹c

Ch­¬ng 4. Bµi to¸n ®Õm 4 − 5

4.2.3. Ho¸n vÞ • §Þnh nghÜa : Mét ho¸n vÞ cña n phÇn tö lµ mét c¸ch s¾p xÕp thø tù c¸c phÇn tö ®ã. Ho¸n vÞ cã thÓ coi nh­ tr­êng hîp riªng cña chØnh hîp kh«ng lÆp chËp k cña n trong ®ã k = n. Ta cã sè ho¸n vÞ lµ

P(n) = n! ◊ VÝ dô: Cã 6 ng­êi xÕp thµnh hµng ngang ®Ó chôp ¶nh. Hái cã thÓ bè trÝ bao nhiªu kiÓu kh¸c nhau ? Gi¶i: Mçi kiÓu ¶nh lµ mét ho¸n vÞ cña 6 ng­êi. VËy sè kiÓu ¶nh lµ 6! = 720. 4.2.4. Tæ hîp ◊ §Þnh nghÜa: Mét tæ hîp chËp k cña n phÇn tö lµ mét bé kh«ng kÓ thø tù gåm k thµnh phÇn kh¸c nhau lÊy tõ n phÇn tö ®· cho. Nãi c¸ch kh¸c ta cã thÓ coi mét tæ hîp chËp k cña n phÇn tö lµ mét tËp con cã k phÇn tö tõ n phÇn tö ®· cho. Gäi sè tæ hîp chËp k cña n phÇn tö lµ C(n,k) ta cã :

A(n,k) = C(n,k) * k! Suy ra

C(n,k) = )!!.(!

knkn−

◊ VÝ dô: Cã n ®éi bãng thi ®Êu vßng trßn. Ph¶i tæ chøc bao nhiªu trËn ®Êu bãng tÊt c¶ ? Gi¶i : Mçi trËn øng víi mét tæ hîp chËp 2 cña n. VËy cã C(n,2) trËn ®Êu. • HÖ qu¶ : TÝch k sè tù nhiªn liªn tiÕp chia hÕt k! Chøng minh. V× C(n,k) = (n−k+1).(n−k+2).....n / k! lµ sè nguyªn. • TÝnh chÊt : - C(n,k) = C(n,n-k) - C(n,0) = C(n,n) = 1 - C(n,k) = C(n−1,k−1) + C(n−1,k) • NhÞ thøc Newton: (x+y)n = C(n,0).xn + C(n,1).xn−1.y +...+ C(n,n−1).x.yn−1 + C(n,n).yn

TrÇn Quèc ChiÕn To¸n rêi r¹c

Ch­¬ng 4. Bµi to¸n ®Õm 4 − 6

• HÖ qu¶ nhÞ thøc Newton: • C(n,0) + C(n,1) + ... + C(n,n) = 2n (sè c¸c tËp con cña n phÇn tö lµ 2n ) • C(n,0) − C(n,1) + ... + (−1)nC(n,n) = 0 (sè tËp con ch½n b»ng sè tËp con lÎ). 4.2.5. Ho¸n vÞ lÆp ◊ VÝ dô: Cã 3 viªn bi ®á, 2 viªn bÞ xanh vµ 4 viªn bi tr¾ng. Hái cã bao nhiªu c¸ch s¾p c¸c viªn bi trªn theo hµng ngang. Ta cã tÊt c¶ 9 chç trèng ®Ó xÕp c¸c viªn bi. Ta cã C(9,3) kh¶ n¨ng xÕp 3 viªn bi ®á, C(6,2) kh¶ n¨ng xÕp 2 viªn bi xanh, cßn l¹i 1 kh¶ n¨ng xÕp c¸c viªn bi tr¾ng. Theo nguyªn lý nh©n ta cã

C(9,3).C(6,2) = !4!.2!.3

!9!4!.2

!6.!6!.3

!9=

c¸ch xÕp. • §Þnh nghÜa: Ho¸n vÞ lÆp lµ ho¸n vÞ trong ®ã mçi phÇn tö ®­îc Ên ®Þnh mét sè lÇn lÆp l¹i cho tr­íc. • §Þnh lý: Gi¶ sö tËp S cã n phÇn tö, trong ®ã cã n1 kiÓu 1, n2 kiÓu 2, ..., nk kiÓu k. Khi ®ã sè c¸c ho¸n vÞ n phÇn tö cña S lµ

( )!!...!.

!,...,,21

21k

kn nnnnnnnC =

4.2.6. Tæ hîp lÆp ◊ VÝ dô: Gi¶ sö ta cã 3 quyÓn s¸ch : To¸n, Tin, Lý vµ mçi quyÓn cã Ýt nhÊt 6 b¶n photocopy. Hái cã bao nhiªu c¸ch chän ra 6 quyÓn. Gi¶i: Bµi to¸n ®Æt ra lµ chän 6 phÇn tö, kh«ng kÓ thø tù vµ cho phÐp lÆp l¹i. Mçi c¸ch chän ®­îc x¸c ®Þnh duy nhÊt bëi sè l­îng cña mçi lo¹i s¸ch. Nh­ vËy ta cã thÓ biÓu diÔn mçi c¸ch chän nh­ sau

To¸n Tin Lý x x x | x x | x

TrÇn Quèc ChiÕn To¸n rêi r¹c

Ch­¬ng 4. Bµi to¸n ®Õm 4 − 7

trong ®ã 6 dÊu x chØ quyÓn s¸ch chän vµ 2 dÊu | chØ ph©n c¸ch gi÷a c¸c lo¹i s¸ch. Nh­ vËy mçi c¸ch chän t­¬ng ®­¬ng víi tæ hîp chËp 2 (dÊu |) tõ 8 phÇn tö. Ta cã sè c¸ch chän lµ

C(8,2) = 28 • §Þnh nghÜa: Tæ hîp lÆp chËp k tõ n phÇn tö lµ mét nhãm kh«ng ph©n biÖt thø tù gåm k phÇn tö trÝch tõ n phÇn tö ®· cho, trong ®ã c¸c phÇn tö cã thÓ ®­îc lÆp l¹i. • §Þnh lý: Gi¶ sö X cã n phÇn tö. Khi ®ã sè tæ hîp lÆp chËp k tõ n phÇn tö cña X lµ

C(k + n − 1, n − 1) = C(k + n − 1, k). ◊ VÝ dô: Ph­¬ng tr×nh

x1 + x2 + x3 + x4 = 10 cã bao nhiªu bé nghiÖm nguyªn kh«ng ©m ? Gi¶i : Mçi béi nghiÖm nguyªn kh«ng ©m cña ph­¬ng tr×nh t­¬ng øng 1-1 víi mét c¸ch chän 10 phÇn tö, trong ®ã phÇn tö kiÓu i lÆp l¹i xi lÇn, i=1,…,4. VËy sè bé nghiÖm lµ sè tæ hîp lÆp chËp 10 cña 4. VËy ta cã sè nghiÖm lµ

C(10 + 4 -1 , 4 - 1) = C(13, 3) = 286

TrÇn Quèc ChiÕn To¸n rêi r¹c

Ch­¬ng 4. Bµi to¸n ®Õm 4 − 8

4.3. mét sè bµi tËp øng dông 4.3.1. Bµi to¸n ®Õm c¸ch xÕp chç ◊ Bµi 1: Cã bao nhiªu c¸ch xÕp c¸c ch÷ c¸i A,B,C,D,E,F chøa x©u DEF ? Gi¶i. X©u DEF ®i liÒn nhau coi nh­ 1 phÇn tö. VËy cã tÊt c¶ 4! = 24 c¸ch xÕp. ◊ Bµi 2: Cã bao nhiªu c¸ch xÕp c¸c ch÷ c¸i A,B,C,D,E,F chøa c¸c ch÷ D,E,F kÒ nhau ? Gi¶i. øng víi mçi ho¸n vÞ con cña D,E,F ta cã 4! ho¸n vÞ cña A,B,C,D,E,F chøa ho¸n vÞ con. VËy ta cã tÊt c¶ 3! * 4! = 144 c¸ch xÕp. ◊ Bµi 3: Cã bao nhiªu c¸ch xÕp c¸c ch÷ c¸i A,B,C,D,E,F sao cho A,B kh«ng ®øng kÒ nhau ? Gi¶i. Sè c¸ch xÕp A, B ®øng gÇn nhau lµ 2! * 5! = 2 . 120 = 240. VËy , theo nguyªn lý céng, sè c¸ch xÕp mµ A,B kh«ng kÒ nhau lµ

6! − ( 2! * 5!) = 720 - 240 = 480 ◊ Bµi 4: Mét tæ sinh viªn cã 7 nam vµ 5 n÷ xÕp thµnh hµng däc. Hái cã bao nhiªu c¸ch xÕp hµng ®Ó kh«ng cã hai sinh viªn n÷ ®øng gÇn nhau ? Gi¶i. Mçi c¸ch xÕp hµng t­¬ng øng víi mét ho¸n vÞ cña 7 (SV nam A1,A2,...,A7) vµ mét chØnh hîp chËp 5 (SV n÷) cña 8 (kho¶ng trèng ký hiÖu b»ng dÊu g¹ch ngang):

_A1_A2_A3_A4_A5_A6_A7_ Nh­ vËy ta cã tÊt c¶

7! * P(8,5) = 5040 . 6720 = 33 868 800 c¸ch xÕp hµng. ◊ Bµi 5: Cã bao nhiªu c¸ch xÕp k bit 0 vµ m bit 1 trªn hµng ngang sao cho kh«ng cã 2 bit 0 kÒ nhau. Gi¶i. C¸c bit 0 ®­îc xÕp chen vµo m+1 kho¶ng trèng gi÷a c¸c bit 1. Nh­ vËy ta cã sè c¸ch xÕp lµ :

C(m+1,k). Chó ý r»ng ®Ó cã lêi gi¶i ph¶i tho¶ m ≥ k .

TrÇn Quèc ChiÕn To¸n rêi r¹c

Ch­¬ng 4. Bµi to¸n ®Õm 4 − 9

◊ Bµi 6: Cã bao nhiªu c¸ch xÕp k bit 0 vµ m bit 1 trªn vßng trßn ®­îc ®¸nh sè tõ 1 ®Õn m+k (vÞ trÝ m+k kÒ víi vÞ trÝ 1) sao cho kh«ng cã 2 bit 0 kÒ nhau. Gi¶i. Cè ®Þnh vÞ trÝ 1. Ta cã 2 tr­êng hîp. • Tr­êng hîp vÞ trÝ 1 lµ bit 0. Lóc nµy ph¶i xÕp bit 1 vµo vÞ trÝ 2 vµ vÞ trÝ m+k. Ta cßn m−2 bit 1 vµ k−1 bit 0 vµ quay l¹i bµi to¸n trªn. Nh­ vËy sè c¸ch xÕp trong tr­êng hîp nµy lµ :

C(m−1,k−1) • Tr­êng hîp vÞ trÝ 1 lµ bit 1. Ta cßn m−1 bit 1 vµ k bit 0 vµ quay l¹i bµi to¸n trªn. Nh­ vËy sè c¸ch xÕp trong tr­êng hîp nµy lµ :

C(m,k) Tæng céng sè c¸ch xÕp lµ

C(m−1,k−1) + C(m,k) = )!)!.(1(

)!1(kmk

m−−

− + C(m,k)

= mk

. C(m,k) + C(m,k) = m

mk +. C(m,k)

Chó ý r»ng ®Ó cã lêi gi¶i ph¶i tho¶ m ≥ k . 4.3.2. Bµi to¸n ®Õm sè ®­êng ®i ◊ Bµi 1: Cho l­íi c¸c « vu«ng sau : (0,m) (n,m)

(0,0) (n,0) Hái cã bao nhiªu ®­êng ®i kh¸c nhau tõ nót (0,0) ®Õn nót (n,m) nÕu chØ cho phÐp ®i trªn c¹nh c¸c « vu«ng theo chiÒu sang ph¶i hoÆc lªn trªn. Gi¶i : Mçi ®­êng ®i gåm m + n ®o¹n, trong ®ã cã m ®o¹n lªn trªn, vµ n ®o¹n sang ph¶i, t­¬ng øng tæ hîp chËp m (lªn trªn) tõ m + n phÇn tö , vÝ dô :

→↑→→↑→→↑→→↑→→↑

TrÇn Quèc ChiÕn To¸n rêi r¹c

Ch­¬ng 4. Bµi to¸n ®Õm 4 − 10

VËy sè ®­êng ®i lµ C(n+m,m) ◊ Bµi 2: Cho l­íi c¸c « vu«ng kÝch th­íc n * n sau : (0,n) (n,n)

(0,0) (n,0) C¸c ®­êng ®i tõ nót (0,0) ®Õn (n,n) (xem vÝ dô 1) gäi lµ tèt nÕu nã kh«ng v­ît lªn trªn ®­êng chÐo chÝnh. C¸c ®­êng kh¸c gäi lµ xÊu. H·y ®Õm sè ®­êng ®i tèt. Gi¶i : Ký hiÖu Gn lµ tæng sè ®­êng ®i tèt, Bn lµ sè ®­êng ®i xÊu. Ta cã, theo vÝ dô 1,

Gn + Bn = C(2n,n) Víi mçi ®­êng ®i xÊu P ta ký hiÖu (x,y) lµ nót ®Çu tiªn n»m bªn trªn ®­êng chÐo chÝnh. Ta x©y dùng mét ®­êng ®i P’ tõ nót (0, 0) ®Õn nót (n−1,n+1) nh­ sau : §o¹n tõ (0,0) ®Õn (x,y) P’ trïng víi P. §o¹n tõ (x,y) ®Õn (n−1,n+1) ®èi xøng víi P qua ®­êng th¼ng nèi hai ®iÓm (0,1) vµ (n−1,n). Gi÷a P vµ P’ lµ quan hÖ 1-1. Nh­ vËy ta cã

Bn = C(2n,n−1) Suy ra

Gn = C(2n,n) − C(2n,n−1) =

!!.)!2(

nnn −

)!1)!.(1()!2(

+− nnn =

)1(),2(

+nnnC

Sè Gn gäi lµ sè Catalan (mang tªn nhµ to¸n häc BØ Eugene Catalan ,1814-1894). 4.3.3. ¸p dông c«ng thøc truy håi

TrÇn Quèc ChiÕn To¸n rêi r¹c

Ch­¬ng 4. Bµi to¸n ®Õm 4 − 11

◊ Bµi 1: Trªn mÆt ph¼ng kÎ n ®­êng th¼ng sao cho kh«ng cã ba ®­êng nµo ®ång qui vµ kh«ng cã hai ®­êng nµo song song. Hái mÆt ph¼ng ®­îc chia lµm mÊy phÇn ? Gi¶i : Gäi sè phÇn mÆt ph¼ng chia bëi n ®­êng th¼ng lµ s(n). Gi¶ sö ®· kÎ n−1 ®­êng th¼ng. B©y giê kÎ thªm ®­êng th¼ng thø n th× sè phÇn mÆt ph¼ng ®­îc thªm sÏ b»ng sè giao ®iÓm céng 1 (n−1 + 1 = n). V× vËy ta cã c«ng thøc truy håi sau

s(n) = s(n−1) + n & s(1) = 2

Gi¶i c«ng thøc trªn b»ng ph­¬ng ph¸p lÆp ta cã

s(n) = s(n-1) + n = s(n-2) + (n-1) + n ... = s(1) + 2 + 3 + ... + (n-1) + n = 1 + 1 + 2 + 3 + ... + (n-1) + n = 1 + n(n+1)/2 ◊ Bµi 2: TÝnh sè mÊt thø tù Dn . Gi¶i : Mçi kÕt hîp mÊt thø tù lµ mét ho¸n vÞ a1 , a2 , ... , an cña n sè 1, 2, ..., n sao cho ai ≠ i ∀i=1,..,n. Thµnh phÇn a1 cã thÓ nhËn (n−1) gi¸ trÞ . Víi mçi gi¸ trÞ k = a1 ta cã 2 tr­êng hîp : i) ak = 1 : Khi ®ã c¸c thµnh phÇn cßn l¹i t¹o thµnh mét mÊt thø tù cña n-2 phÇn tö. Nh­ vËy sè mÊt thø tù trong tr­êng hîp nµy lµ (n-1).Dn-2 . ii) ak ≠ 1 : Khi ®ã, coi 1 nh­ lµ k, c¸c thµnh phÇn cßn l¹i t¹o thµnh mét mÊt thø tù cña n-1 phÇn tö. Nh­ vËy sè mÊt thø tù trong tr­êng hîp nµy lµ (n-1).Dn-1 . Cuèi cïng ta cã c«ng thøc

Dn = (n−1).(Dn-1 + Dn-2 ) §Ó t×m c«ng thøc t­êng minh ta biÕn ®æi c«ng thøc trªn thµnh

Dn − n.Dn-1 = − (Dn-1 − (n−1).Dn-2 ) §Æt Vn = Dn − n.Dn−1 ta cã

Vn = − Vn-1 = ... = (−1)n - 2 V2 = (−1)n-2 = (−1)n Suy ra

TrÇn Quèc ChiÕn To¸n rêi r¹c

Ch­¬ng 4. Bµi to¸n ®Õm 4 − 12

!nDn −

)!1(1

−−

nDn =

!)1(

n

n−

Céng c¸c hÖ thøc trªn , víi n=1,...,n, ta ®­îc

!nDn = 1 −

!11

+ !2

1 + ... +

!)1(

n

n−

Tõ ®ã ta cã

Dn = n! . (1 − !1

1 + !2

1 + ... + !)1(

n

n− )

D­íi ®©y lµ mét vµi gi¸ trÞ cña Dn

n = 4 5 6 7 8 9 10 11 Dn = 9 44 256 1 854 14 833 133 496 1 334 961 14 684 570

TrÇn Quèc ChiÕn To¸n rêi r¹c

Ch­¬ng 4. Bµi to¸n ®Õm 4 − 13

4.4. nguyªn lý bï trõ 4.4.1. Nguyªn lý Cho hai tËp A , B. Theo nguyªn lý céng ta cã • C«ng thøc 1: A ∪ B = A + B - A ∩ B Cho n tËp X1, X2 ,..., Xn b»ng quy n¹p ta cã • C«ng thøc 2:

X1 ∪ X2 ∪... ∪ Xn = ∑=

−−n

k

k knS1

1 ),()1(

trong ®ã

S(n,k) = ∑≤≤≤≤

∩∩∩nii

iiik

kxxx

...1 1

21...

Trong tæng S(n,k) (i1 , i2 , ... , ik ) lÊy tÊt c¶ c¸c tæ hîp chËp k cña n vµ nh­

vËy S(n,k) lµ tæng cña C(n,k) sè h¹ng. Nãi riªng ta cã

S(n,1) = X1 + X2 + ... + Xn vµ

S(n,n) = X1 ∩ X2 ∩... ∩ Xn B©y giê ta cho c¸c tÝnh chÊt α1 ,..., αn trªn tËp X. XÐt bµi to¸n : • §Õm sè phÇn tö trong X kh«ng tho¶ m·n mét tÝnh chÊt αk nµo c¶. Gi¶i. Víi mäi k = 1, ..., n, ta ký hiÖu:

Xk = { x ∈ X x tho¶ m·n αk } Nh­ vËy phÇn bï cña Xk lµ

kX = { x ∈ X x kh«ng tho¶ m·n αk } Ký hiÖu N lµ sè cÇn ®Õm, ta cã :

TrÇn Quèc ChiÕn To¸n rêi r¹c

Ch­¬ng 4. Bµi to¸n ®Õm 4 − 14

N = 1X ∩ 2X ∩... ∩ nX = nXXX ∪∪∪ ...21

= X − X1 ∪ X2 ∪... ∪ Xn

Tõ c«ng thøc 2 suy ra

N = X − ∑=

−−n

k

k knS1

1 ),()1( = X + ∑=

−n

k

k knS1

),()1(

Tõ ®ã ta cã • Nguyªn lý bï trõ :

N = ∑=

−n

k

k knS0

),()1(

trong ®ã

S(n,0) = X vµ

S(n,k) = ∑≤≤≤≤

∩∩∩nii

iiik

kXXX

...1 1

21... ∀k=1, ..., n

4.4.2. Bµi to¸n øng dông ◊ Bµi to¸n bá th­: Cã n l¸ th­ vµ n phong b× ghi s½n ®Þa chØ. Bá ngÉu nhiªn c¸c l¸ th­ vµo phong b×.

i) Hái s¸c xuÊt ®Ó kh«ng l¸ th­ nµo ®óng ®Þa chØ lµ bao nhiªu ? ii) Hái s¸c xuÊt ®Ó ®óng r l¸ th­ ®óng ®Þa chØ lµ bao nhiªu (r ≤ n) ?

Gi¶i . i) Gäi X lµ tËp hîp tÊt c¶ c¸c c¸ch bá th­. Ta cã X = n! . Gäi αk lµ tÝnh

chÊt l¸ th­ k göi ®óng ®Þa chØ, Xk lµ tËp hîp c¸ch bá th­ sao cho l¸ th­ th­ k kh«ng göi ®óng ®Þa chØ (k=1, ..., n). KÝ hiÖu N(n, k) lµ sè c¸ch bá th­ sao cho cã ®óng k l¸ th­ ®óng ®Þa chØ. Nh­ vËy theo nguyªn lý bï trõ sè c¸ch bá th­ sao cho kh«ng cã l¸ th­ nµo göi ®óng ®Þa chØ lµ.

N(n,0) = ∑=

−n

k

k knS0

),()1(

trong ®ã

TrÇn Quèc ChiÕn To¸n rêi r¹c

Ch­¬ng 4. Bµi to¸n ®Õm 4 − 15

S(n,0) = X = n!

S(n,k) = ∑≤≤≤≤

∩∩∩nii

iiik

kXXX

...1 1

21... ∀k=1, ..., n

Víi mçi bé k l¸ th­ i1, i2, ..., ik ta cã (n−k)! c¸ch bá th­, tøc ho¸n vÞ c¸c l¸ th­ cßn l¹i, sao cho c¸c l¸ th­ i1, i2, ..., ik bá ®óng ®Þa chØ. Nh­ vËy ta cã

S(n,k) = C(n,k). (n−k)! = !!

kn

Suy ra

N(n,0) = n!.( )

−+++−

!1...

!21

!111

n

n

Nh­ vËy s¸c xuÊt cÇn t×m lµ

( )

−+++−

!1...

!21

!111

n

n

Mét ®iÒu lý thó lµ s¸c xuÊt trªn tiÕn ®Õn 1/ e khi n → ∞ . Sè N(n,0) trªn còng chÝnh lµ tæng sè ho¸n vÞ f(i) cña tËp {1,2,...,n} tho¶ m·n f(i) ≠ i . V× vËy N(n,0) ®­îc gäi lµ sè mÊt thø tù vµ ®­îc ký hiÖu lµ Dn .

ii) Cho tæ hîp i1, i2, ..., ir . Sè c¸ch bá th­ ®Ó chØ c¸c l¸ th­ i1, i2, ..., ir göi ®óng ®Þa chØ ®óng b»ng N(n−r,0). Nh­ vËy sè c¸ch bá th­ ®Ó cã ®óng r l¸ th­ göi ®óng ®Þa chØ lµ

N(n,r) = C(n,r). N(n−r,0) = C(n,r). (n−r)!. ( )

−+++−

)!(1...

!21

!111

rn

rn

◊ Bµi to¸n xÕp kh¸ch (Lucas) : Mét bµn trßn cã 2n ghÕ . CÇn s¾p n cÆp vî chång sao cho ®µn «ng ngåi xen kÏ ®µn bµ vµ kh«ng cã cÆp nµo ngåi c¹nh nhau (cã tÝnh ®Õn vÞ trÝ ghÕ vµ thø tù chç ngåi) . Hái cã bao nhiªu c¸ch xÕp ?

TrÇn Quèc ChiÕn To¸n rêi r¹c

Ch­¬ng 4. Bµi to¸n ®Õm 4 − 16

Gi¶i. Gäi sè ph¶i t×m lµ Mn . XÕp c¸c bµ tr­íc (cø mét ghÕ xÕp th× mét ghÕ ®Ó trèng dµnh cho c¸c «ng). Víi n ghÕ ch½n ta cã n! c¸ch xÕp vµ víi n ghÕ lÎ ta còng cã n! c¸ch xÕp. Nh­ vËy sè c¸ch xÕp c¸c bµ lµ 2.n! . Gäi sè c¸ch xÕp c¸c «ng øng víi mét c¸ch xÕp c¸c bµ lµ Un , ta cã

Mn = 2.n!.Un B©y giê ta ®i tÝnh Un.

§¸nh sè c¸c bµ (®· xÕp ) tõ 1 ®Õn n. §¸nh sè c¸c «ng t­¬ng øng víi c¸c bµ («ng i lµ chång bµ i ). §¸nh sè c¸c ghÕ trèng theo nguyªn t¾c : ghÕ sè i n»m gi÷a bµ i vµ i+1 (quy ­íc n+1 = 1). Mçi c¸ch xÕp c¸c «ng ®­îc biÓu diÔn b»ng mét ho¸n vÞ f(i) cña tËp {1,2,...,n}, tøc lµ ghÕ i ®­îc xÕp cho «ng f(i). §Ó tho¶ m·n yªu cÇu bµi to¸n f(i) ph¶i tho¶ m·n f(i) ≠ i & f(i) ≠ i + 1 (*)

Nh­ vËy sè Un lµ sè c¸c ho¸n vÞ tho¶ m·n (*). Sè Un gäi lµ sè ph©n bè. XÐt tËp hîp X c¸c ho¸n vÞ f cña {1,2,...,n}. Ta gäi

Pi lµ tÝnh chÊt f(i) = i vµ

Qi lµ tÝnh chÊt f(i) = i + 1 KÝ hiÖu Pn+i lµ tÝnh chÊt Qi . TiÕp theo kÝ hiÖu Xi lµ sè c¸c ho¸n vÞ cña trong X tho¶ tÝnh chÊt Pi , i = 1, ..., 2n. Nh­ vËy theo nguyªn lý bï trõ sè c¸ch xÕp chç lµ

Un = ∑=

−n

k

k knS2

0),2()1(

trong ®ã

S(2n,0) = X = n! vµ

S(2n,k) = ∑≤≤≤≤

∩∩∩nii

iiik

kXXX

2...1 1

21... ∀k=1, ..., 2n

Chó ý r»ng kh«ng thÓ x¶y ra ®ång thêi Pi vµ Qi hoÆc ®ång thêi Pi+1 vµ Qi , tøc lµ :

TrÇn Quèc ChiÕn To¸n rêi r¹c

Ch­¬ng 4. Bµi to¸n ®Õm 4 − 17

Xi ∩ Xn+i = ∅ & Xi+1 ∩ Xn+i = ∅ ∀i=1,...,n

Nh­ vËy ta cã

S(2n,k) = 0 ∀k > n vµ kÐo theo

Un = ∑=

−n

k

k knS0

),2(.)1(

Gäi g(2n,k) lµ sè c¸ch lÊy ra k tÝnh chÊt tho¶ m·n kh«ng thÓ x¶y ra ®ång

thêi Pi vµ Qi hoÆc ®ång thêi Pi+1 vµ Qi ( g(2n,0) = 1). Vµ víi mçi c¸ch lÊy k tÝnh chÊt nh­ vËy ta cã (n-k)! c¸ch ph©n bè c¸c tÝnh chÊt cßn l¹i. Nh­ vËy ta cã

Un = ∑=

−−−n

k

nk knkng0

)!.()1).(,2(.)1(

B©y giê ta cßn ph¶i tÝnh g(2n,k). NÕu xÕp theo vßng trßn P1 , Q1 , P2 , Q2 ,..., Pn , Qn , ta thÊy g(2n,k) chÝnh lµ sè c¸ch lÊy ra k phÇn tö sao cho kh«ng cã hai phÇn tö kÒ nhau. §©y chÝnh lµ sè c¸ch xÕp k sè 0 víi (2n-k) sè 1 sao cho kh«ng cã hai sè 0 kÒ nhau. Theo bµi to¸n 6 ë phÇn III ta cã

g(2n,k) = kn

n−2

2 C(2n - k , k)

Nh­ vËy sè ph©n bè

Un = ∑ (-1)k . kn

n−2

2 C(2n - k,k).(n-k)!

(Touchard J. 1934 France ).

TrÇn Quèc ChiÕn To¸n rêi r¹c

Ch­¬ng 4. Bµi to¸n ®Õm 4 − 18

bµi tËp

Bµi 1. H·y ®Õm sè c¸ch chän n cÆp tõ 2n phÇn tö kh¸c nhau. (§¸p sè: Sn = Sn-1 + 2(n-1)Sn-1 = (2n-1)Sn-1 = (2n-1)!!) Bµi 2. Cã bao nhiªu c¸ch chän 2 sè nguyªn tõ 1,2,...,2n sao cho tæng cña chóng lµ sè lÎ, sè ch½n. (§¸p sè: Tæng lÎ = n.n = n2 , Tæng ch½n = C(2n,2) - n2 = 2.C(n,2)2 ) Bµi 3. Trong 2n phÇn tö cã ®óng n phÇn tö gièng hÖt nhau. Cã bao nhiªu c¸ch chän n phÇn tö. (§¸p sè: C(n,0) + C(n,1) + C(n,2) +...+ C(n,n) = 2n ) Bµi 4. Cã bao nhiªu cÆp sè tõ 1,2,...,n-1 cã tæng lín h¬n n . (§¸p sè: n=2k : 1+3+...+(2k-1) = (k-1)2 n =2k+1 : 2+4+...+(2k-2) = k(k-1) ; h­íng dÉn: vÏ h×nh)

TrÇn Quèc ChiÕn To¸n rêi r¹c

Ch­¬ng 5. Bµi to¸n liÖt kª 5 − 1

Ch­¬ng 5. bµi to¸n liÖt kª 5.1. Ph¸t biÓu bµi to¸n liÖt kª • VÝ dô

Mét b¨ng video cã thÓ ghi ®­îc C gi©y. Ta cã n phim video víi thêi gian

t­¬ng øng lµ t1, t2 , ... , tn . Ta ph¶i chän k phim i1, i2 , ... , ik sao cho tæng

∑∈ },...,,{ 21 kiiii

it

kh«ng v­ît qu¸ C vµ lín nhÊt. Mét c¸ch gi¶i ch©n ph­¬ng lµ liÖt kª tÊt c¸c tËp con {i1, i2 , ... , ik} ⊂ {1, 2 , ... ,

n} vµ chän tËp cã tæng trªn lµ lín nhÊt kh«ng v­ît qu¸ C. V× vËy trong nhiÒu tr­êng hîp, khi kh«ng cã thuËt to¸n hiÖu qu¶ ®Ó gi¶i quyÕt nh÷ng bµi to¸n nh­ trªn, th× ph­¬ng ph¸p liÖt kª vãi sù trî gióp cña m¸y tÝnh vÉn lµ gi¶i ph¸p kh¶ dÜ.

• Ph¸t biÓu bµi to¸n liÖt kª

X¸c ®Þnh thuËt to¸n x©y dùng lÇn l­ît cÊu h×nh quan t©m. ThuËt to¸n cÇn ®¶m

b¶o c¸c yªu cÇu sau: - Kh«ng lÆp l¹i cÊu h×nh - Kh«ng bá sãt cÊu h×nh

TrÇn Quèc ChiÕn To¸n rêi r¹c

Ch­¬ng 5. Bµi to¸n liÖt kª 5 − 2

5.2. Ph­¬ng ph¸p sinh §Ó cã thÓ liÖt kª tÊt c¶ cÊu h×nh ta cÇn s¾p xÕp c¸c cÊu h×nh theo thø tù nµo ®ã, sau ®ã liÖt kª chóng theo thø tù tõ nhá ®Õn lín th× ®¶m b¶o hai yªu cÇu cña bµi to¸n liÖt kª. Mét lo¹i thø tù hay ®­îc dïng lµ thø tù tõ ®iÓn. Trong bµi nµy chóng ta sÏ nghiªn cøu ph­¬ng ph¸p t¹o ra mét cÊu h×nh tæ hîp liÒn kÒ ngay sau mét cÊu h×nh ®· cã theo thø tù tõ ®iÓn. §ã chÝnh lµ ý t­ëng ph­¬ng ph¸p sinh. 5.2.1. Thø tù tõ ®iÓn vµ ph­¬ng ph¸p sinh • Thø tù tõ ®iÓn

Cho α = s1 s2 ... sp vµ β = t1 t2 ... tq lµ c¸c d·y sè hoÆc ký tù. Ta nãi r»ng α nhá h¬n β (theo kiÓu tõ ®iÓn) , ký hiÖu α < β, nÕu hoÆc

(i) p < q vµ si = ti víi mäi i = 1,2,...,p

hoÆc

(ii) Tån t¹i k ≤ min{p, q} sao cho si = ti víi mäi i = 1,2,...,k-1 vµ sk < tk ◊ VÝ dô: - BANANA < BANDIT - AN < ANH - 132 < 1324 - 13246 < 1342 Trong c¸c thuËt to¸n liÖt kª tiÕp theo chóng ta sÏ liÖt kª c¸c cÊu h×nh theo thø tù tõ ®iÓn. ThuËt to¸n cã c¸c b­íc chung nh­ sau: • ThuËt to¸n sinh tæng qu¸t KÝ hiÖu s lµ cÊu h×nh hiÖn hµnh, s0 lµ cÊu h×nh ®Çu tiªn (theo thø tù tõ ®iÓn). B­íc 1. Khëi t¹o, g¸n s := s0. B­íc 2. KÕt xuÊt s. B­íc 3. KiÓm tra tiªu chuÈn kÕt thóc. NÕu s lµ cÊu h×nh cuèi cïng th× kÕt thóc, ng­îc l¹i sang b­íc 4. B­íc 4. T×m cÊu h×nh t ®øng kÒ sau s theo thø tù tõ ®iÓn. G¸n s := t vµ quay l¹i b­íc 2. ◊ L­u ý. Tuú theo bµi to¸n cô thÓ cã thÓ gép b­íc 3 vµ 4 thµnh 1 b­íc ®Ó t¨ng hiÖu qu¶ thuËt to¸n. ¸p dông thuËt to¸n tæng qu¸t cho c¸c bµi to¸n cô thÓ, ta chØ cÇn x¸c ®Þnh cÊu h×nh ®Çu tiªn s0 vµ ph­¬ng ph¸p t×m cÊu h×nh t kÕ tiÕp sau cÊu h×nh s.

TrÇn Quèc ChiÕn To¸n rêi r¹c

Ch­¬ng 5. Bµi to¸n liÖt kª 5 − 3

Sau ®©y chóng ta sÏ nghiªn cøu mét sè bµi to¸n liÖt kª quan träng. 5.2.2. LiÖt kª tÊt c¶ c¸c d·y nhÞ ph©n cã ®é dµi b»ng n • Ph¸t biÓu bµi to¸n. Cho n ∈ N. H·y liÖt kª, theo thø tù tõ ®iÓn, tÊt c¶ c¸c d·y nhÞ ph©n ®é dµi n, tøc lµ c¸c d·y [ b1, ..., bn], trong ®ã bi ∈ {0, 1} ∀ i=1, ..., n. Sè d·y nhÞ ph©n lµ 2n vµ d·y ®Çu tiªn s0 = [0, 0, ..., 0]. Ph­¬ng ph¸p t×m d·y kÕ tiÕp nh­ sau. • Ph­¬ng ph¸p t×m d·y kÕ tiÕp: Cho d·y nhÞ ph©n s = [s1 , s2 ,... ,sn ], ta t×m d·y tiÕp theo t = [t1 , t2 ,... ,tn ]. XuÊt ph¸t tõ sn, ®i tõ ph¶i sang tr¸i, ta t×m phÇn tö ®Çu tiªn sm , 1 ≤ m ≤ n, tho¶ sm = 0.

NÕu kh«ng t×m thÊy th× s = [1, 1, ..., 1] lµ d·y cuèi cïng, kÕt thóc t×m kiÕm. NÕu t×m thÊy ta x©y dùng d·y t = [t1 , t2 ,... ,tn ] nh­ sau:

ti = si víi mäi i = 1,2,...,m-1 tm = 1 ti = 0 víi mäi i = m+1,m+2,...,n Tõ ®ã suy ra thuËt to¸n liÖt kª d·y nhÞ ph©n sau. • ThuËt to¸n:

- §Çu vµo: n - §Çu ra : Danh s¸ch tÊt c¶ d·y nhÞ ph©n ®é dµi n theo thø tù tõ ®iÓn t¨ng dÇn. - C¸c b­íc: 1. Khëi t¹o d·y xuÊt ph¸t : G¸n si := 0 víi mäi i = 1,2,..., n .

2. KÕt xuÊt s. 3. T×m m tho¶

m = max{i si = 0 } NÕu kh«ng t×m thÊy th× s = [1, 1, ..., 1] lµ d·y cuèi cïng, kÕt thóc. NÕu t×m thÊy ta ®Æt

sm := 1 si := 0 víi mäi i = m+1,m+2,...,n Quay l¹i b­íc 2.

TrÇn Quèc ChiÕn To¸n rêi r¹c

Ch­¬ng 5. Bµi to¸n liÖt kª 5 − 4

5.2.3. LiÖt kª tæ hîp chËp r tõ n phÇn tö XÐt bµi to¸n liÖt kª tÊt c¶ tæ hîp chËp r tõ n phÇn tö {1,2,...,n}. V× tæ hîp lµ tËp hîp c¸c phÇn tö, kh«ng kÓ thø tù, nªn ta qui ­íc mçi tæ hîp sÏ ®­îc biÓu diÔn b»ng danh s¸ch [s1, s2 ,... , sr ] víi s1 < s2 <... < sr . Nh­ vËy, theo thø tù tõ ®iÓn, tæ hîp ®Çu tiªn lµ [1, 2,..., r] vµ tæ hîp cuèi cïng lµ [n−r+1, n−r+2,..., n]. ◊ VÝ dô: XÐt tæ hîp chËp 5 cña [1,2,3,4,5,6,7]. Tæ hîp ®Çu lµ [1,2,3,4,5]. Tæ hîp tiÕp theo lµ [1,2,3,4,6] vµ [1,2,3,4,7]. Tæ hîp liÒn sau sÏ lµ [1,2,3,5,6] vµ [1,2,3,5,7]. Tæ hîp cuèi lµ [3,4,5,6,7]. T×m tæ hîp ®i sau [1,3,4,6,7]. Kh«ng d·y 5 sè nµo b¾t ®Çu b»ng 1,3,4 v­ît qua [1,3,4,6,7]. V× vËy d·y ph¶i t×m b¾t buéc b¾t ®Çu b»ng 1,3,5 . V× vËy tæ hîp tiÕp theo lµ [1,3,5,6,7]. • Ph­¬ng ph¸p t×m tæ hîp kÕ tiÕp: Cho tæ hîp α = {s1 , s2 ,... ,sr } , ta t×m tæ hîp tiÕp theo β = {t1 , t2 ,... ,tr }. Tr­íc hÕt ta nhËn xÐt thÊy r»ng thµnh phÇn thø i trong tæ hîp kh«ng thÓ v­ît qu¸ n−r+i . Gi¸ trÞ nµy gäi lµ trÞ cùc ®¹i cña thµnh phÇn thø i . Ta t×m

m = max{i si < n-r+i } Sau ®ã ta ®Æt ti = si víi mäi i = 1, 2,..., m−1 tm = sm + 1 tm+i = sm + i + 1 víi mäi i =1, 2, ..., r−m • ThuËt to¸n:

- §Çu vµo: r, n - §Çu ra : Danh s¸ch tÊt c¶ tæ hîp chËp r cña [1,2,...,n] theo thø tù tõ ®iÓn t¨ng dÇn. - C¸c b­íc: 1. Khëi t¹o d·y xuÊt ph¸t : G¸n si := i víi mäi i = 1,2,...,r .

2. KÕt xuÊt s. 3. NÕu tho¶ ®iÒu kiÖn kÕt thóc

s1 = n − r+1

TrÇn Quèc ChiÕn To¸n rêi r¹c

Ch­¬ng 5. Bµi to¸n liÖt kª 5 − 5

thuËt to¸n kÕt thóc. Ng­îc l¹i sang b­íc 4.

4. T×m m tho¶ m = max{i si < n−r+i } §Æt sm := sm + 1 si := si−1 + 1 víi mäi i = m+1,m+2,...,r Quay l¹i b­íc 2. 5.2.4. LiÖt kª ho¸n vÞ XÐt bµi to¸n liÖt kª tÊt c¶ ho¸n vÞ cña n phÇn tö {1,2,...,n}. Mçi ho¸n vÞ sÏ ®­îc biÓu diÔn nh­ lµ d·y s1, s2 ,... , sn . Nh­ vËy ho¸n vÞ ®Çu tiªn lµ [1,2,...,n] vµ ho¸n vÞ cuèi cïng lµ [n,n-1,...,1]. ◊ VÝ dô: Gi¶ sö ta ph¶i t×m ho¸n vÞ t = [t1 , t2 , t3 , t4 , t5 , t6 ] cña {1,2,3,4,5,6} tiÕp theo sau ho¸n vÞ s = [1,6,3,5,4,2]. LiÖu 4 sè ®Çu cña t cã thÓ lµ 1,6,3,5 ®­îc kh«ng ? Kh«ng ! Bëi v× trong c¸c ho¸n vÞ b¾t ®Çu b»ng 1,6,3,5 (chØ cã 2 ho¸n vÞ lµ s vµ [1,6,3,5,2,4]) th× s cã thø tù lín nhÊt. LiÖu 3 sè ®Çu cña t cã thÓ lµ 1,6,3 ®­îc kh«ng ? Kh«ng ! ThËt vËy, c¸c ho¸n vÞ b¾t ®Çu b»ng 1,6,3 gåm : s=[1,6,3,5,4,2], [1,6,3,5,2,4], [1,6,3,4,2,5], [1,6,3,4,5,2], [1,6,3,2,5,4], [1,6,3,2,4,5] Vµ hiÓn nhiªn lµ trong c¸c ho¸n vÞ ®ã s lµ ho¸n vÞ cã sè thø tù lín nhÊt.

Lý do viÖc t kh«ng thÓ b¾t ®Çu bëi 1,6,3,5 hoÆc 1,6,3 lµ v× trong c¶ hai

tr­êng hîp nµy c¸c sè cßn l¹i cña s t¹o thµnh d·y gi¶m dÇn. Nh­ vËy xuÊt ph¸t tõ bªn ph¶i ta t×m sè ®Çu tiªn d nhá h¬n sè bªn ph¶i nã.

Trong vÝ dô nµy ®ã lµ sè 3. Vµ t b¾t ®Çu b»ng 1,6 . Sè tiÕp theo cña t ph¶i lín h¬n 3. V× ta muèn t lµ ho¸n vÞ nhá nhÊt trong c¸c ho¸n vÞ lín h¬n s, sè tiÕp theo ph¶i lµ sè nhá nhÊt trong c¸c sè cßn l¹i (tøc trõ 1 vµ 6) lín h¬n 3. Sè ®ã ph¶i lµ 4. Ba sè cßn l¹i ph¶i t¨ng dÇn. Nh­ vËy ta cã t = [1,6,4,2,3,5]. • Ph­¬ng ph¸p t×m ho¸n vÞ kÕ tiÕp:

TrÇn Quèc ChiÕn To¸n rêi r¹c

Ch­¬ng 5. Bµi to¸n liÖt kª 5 − 6

Cho ho¸n vÞ s = [s1 , s2 ,... ,sn ] , ta t×m ho¸n vÞ tiÕp theo t = [t1 , t2 ,... ,tn ]. §i tõ ph¶i sang tr¸i ta t×m phÇn tö ®Çu tiªn sm tho¶ sm < sm+1 . Sau ®ã t×m chØ sè k lín nhÊt tho¶ sm < sk . Sau ®ã ta ®Æt ti = si víi mäi i = 1,2,...,m−1 tm = sk n−m phÇn tö tiÕp theo lµ c¸c sè cßn l¹i s¾p xÕp theo thø tù t¨ng dÇn. • ThuËt to¸n: - §Çu vµo: n - §Çu ra : Danh s¸ch tÊt c¶ ho¸n vÞ cña {1,2,...,n} theo thø tù t¨ng dÇn. - C¸c b­íc: 1. Khëi t¹o d·y xuÊt ph¸t : G¸n si := i víi mäi i = 1, 2,..., n. 2. KÕt xuÊt s 3. NÕu tho¶ ®iÒu kiÖn kÕt thóc

s = [n, n−1,..., 2,1] thuËt to¸n kÕt thóc. Ng­îc l¹i sang b­íc 4. 4. T×m m lµ chØ sè lín nhÊt tho¶ sm < sm+1 . T×m k lµ chØ sè lín nhÊt tho¶ sm < sk . Ho¸n vÞ sm vµ sk S¾p xÕp l¹i sm+1 , sm+2 ,..., sn theo thø tù t¨ng dÇn. Quay l¹i b­íc 2.

TrÇn Quèc ChiÕn To¸n rêi r¹c

Ch­¬ng 5. Bµi to¸n liÖt kª 5 − 7

5.3. ThuËt to¸n quay lui 5.3.1. Néi dung thuËt to¸n ý t­ëng chÝnh cña thuËt to¸n nµy lµ x©y dùng dÇn c¸c thµnh phÇn cña cÊu h×nh b»ng c¸ch thö tÊt c¶ c¸c kh¶ n¨ng. Gi¶ thiÕt cÊu h×nh cÇn ®­îc m« t¶ b»ng mét bé gåm n thµnh phÇn x1, x2, ..., xn . Gi¶ sö ®· x¸c ®Þnh ®­îc i−1 thµnh phÇn x1 , x2 , ... , xi-1 . Ta x¸c ®Þnh thµnh phÇn thø i b»ng c¸ch duyÖt tÊt c¶ kh¶ n¨ng cã thÓ ®Ò cö cho nã (®¸nh sè c¸c kh¶ n¨ng tõ 1 ®Õn ni ). Víi mçi kh¶ n¨ng j, kiÓm tra xem kh¶ n¨ng j cã chÊp nhËn ®­îc kh«ng. Cã thÓ x¶y ra 2 tr­êng hîp : - NÕu chÊp nhËn j th× x¸c ®Þnh xi theo j , sau ®ã nÕu i = n , th× ta ®­îc mét cÊu h×nh, cßn tr¸i l¹i ta tiÕn hµnh x¸c ®Þnh xi+1. - NÕu thö tÊt c¶ kh¶ n¨ng mµ kh«ng kh¶ n¨ng nµo ®­îc chÊp nhËn th× quay l¹i b­íc tr­íc ®Ó x¸c ®Þnh l¹i xi−1. §iÒu quan träng cña thuËt to¸n lµ ph¶i ghi nhí , t¹i mçi b­íc ®· ®i qua, nh÷ng kh¶ n¨ng ®· thö ®Ó tr¸nh trïng lÆp. Râ rµng nh÷ng th«ng tin nµy cÇn ®­îc l­u tr÷ theo c¬ cÊu ng¨n xÕp (stack - vµo sau ra tr­íc). V× thÕ thñ tôc ®Ö qui rÊt phï hîp víi thuËt to¸n nµy. B­íc x¸c ®Þnh xi cã thÓ diÔn t¶ qua thñ tôc Procedure Try(i:integer); var j:integer; begin for j:=1 to ni do if <chÊp nhËn j> then begin <x¸c ®Þnh xi theo j> if i = n then <ghi nhËn cÊu h×nh> else Try(i+1); end; end; PhÇn quan träng nhÊt trong thñ tôc trªn lµ viÖc ®­a ra ®­îc mét danh s¸ch c¸c kh¶ n¨ng ®Ò cö vµ viÖc x¸c ®Þnh gi¸ trÞ cña biÓu thøc logic <chÊp nhËn j>. Th«ng th­êng gi¸ trÞ nµy ngoµi viÖc phô thuéc j cßn phô thuéc vµo kh¶ n¨ng ®­îc chän ë c¸c b­íc tr­íc. V× thÕ cÇn ghi nhí tr¹ng th¸i míi cña qu¸ tr×nh t×m kiÕm sau khi <x¸c ®Þnh xi theo j> vµ tr¶ l¹i tr¹ng th¸i cò sau lêi gäi Try(i+1) . C¸c tr¹ng th¸i nµy ®­îc ghi nhËn nhê mét sè biÕn toµn côc (global), gäi lµ biÕn tr¹ng th¸i.

TrÇn Quèc ChiÕn To¸n rêi r¹c

Ch­¬ng 5. Bµi to¸n liÖt kª 5 − 8

Sau khi x©y dùng thñ tôc ®Ö qui Try , ch­¬ng tr×nh chÝnh gi¶i bµi to¸n liÖt kª cã d¹ng : Begin Init ; Try(1) ; End. trong ®ã Init lµ thñ tôc khëi t¹o c¸c gi¸ trÞ ban ®Çu (nhËp c¸c gi¸ trÞ tham sè cña bµi to¸n, khëi g¸n c¸c biÕn tr¹ng th¸i, biÕn ®Õm ...). 5.3.2. LiÖt kª c¸c d·y nhÞ ph©n cã ®é dµi n

Ta biÓu diÔn d·y nhÞ ph©n d­íi d¹ng x1, x2,.. xn , trong ®ã xi ∈ {0,1}. Thñ tôc Try(i) x¸c ®Þnh xi ∈ {0,1}. C¸c gi¸ trÞ nµy ®­îc mÆc nhiªn chÊp nhËn mµ kh«ng cÇn ph¶i tho¶ m·n ®iÒu kiÖn g× (v× thÕ bµi to¸n kh«ng cÇn biÕn tr¹ng th¸i). Thñ tôc Init nhÊp gi¸ trÞ n vµ khëi g¸n biÕn ®Õm count . Thñ tôc Result ®­a ra d·y t×m ®­îc. Var n:integer; x:array[1..20] of 0..1; count:integer; Procedure Init; begin write('n = ');readln(n); count := 0; end; Procedure Result; var i:integer; begin count := count + 1; write(count : 5, '.'); for i := 1 to n do write(x[i] : 2); writeln; end; Procedure Try(i:integer); var j : integer; begin for j := 0 to 1 do

TrÇn Quèc ChiÕn To¸n rêi r¹c

Ch­¬ng 5. Bµi to¸n liÖt kª 5 − 9

begin x[i] := j; if i = n then Result else Try(i+1); end; end; Begin Init ; Try(1) ; End. 5.3.3. LiÖt kª c¸c ho¸n vÞ cña {1,2,..., n}

BiÓu diÔn ho¸n vÞ d­íi d¹ng x1, x2,.. xn , trong ®ã xi ∈ {1,2,...,n} vµ xi ≠ xj víi i ≠ j . C¸c gi¸ trÞ j ch¹y tõ 1 ®Õn n ®­îc lÇn l­ît ®Ò cö cho xi vµ j ®­îc chÊp nhËn nÕu nã ch­a ®­îc dïng. V× thÕ cÇn ghi nhí ®èi víi mçi gi¸ trÞ j xem nã ®· ®­îc dïng hay ch­a. §iÒu nµy ®­îc thùc hiÖn nhê d·y biÕn logic b[j] , trong ®ã b[j] = true nÕu j ch­a ®­îc dïng. C¸c biÕn nµy cÇn ph¶i ®­îc g¸n gi¸ trÞ true trong thñ tôc Init. Sau khi g¸n j cho xi cÇn g¸n false cho b[j] vµ g¸n l¹i true khi thùc hiÖn xong thñ tôc Result hay Try(i+1). Var n:integer; x:array[1..20] of integer; b:array[1..20] of boolean; count:integer; Procedure Init; begin write('n = ');readln(n); for i := 1 to n do b[i] := true; count := 0; end; Procedure Result; var i:integer; begin count := count + 1; write(count : 5, '.'); for i := 1 to n do write(x[i] : 3); writeln; end;

TrÇn Quèc ChiÕn To¸n rêi r¹c

Ch­¬ng 5. Bµi to¸n liÖt kª 5 − 10

Procedure Try(i:integer); var j : integer; begin for j :=1 to n do if b[j] then {chÊp nhËn} begin x[i] := j; b[j] := false; {ghi nhËn tr¹ng th¸i míi} if i = n then Result else Try(i+1); b[j] := true; {tr¶ l¹i tr¹ng th¸i cò} end; end; Begin Init ; Try(1) ; End. 5.3.4. Tæ hîp chËp r tõ n phÇn tö XÐt bµi to¸n liÖt kª tÊt c¶ tæ hîp chËp r tõ n phÇn tö {1,2,...,n}. Mçi tæ hîp sÏ ®­îc biÓu diÔn nh­ lµ d·y [x1, x2 ,... , xr ] víi x1 < x2 <... < xr . Nh­ vËy c¸c gi¸ trÞ ®Ò cö cho xi lµ tõ xi-1 + 1 ®Õn n−r+i . §Ó ®iÒu nµy ®óng cho c¶ tr­êng hîp i = 1 ta thªm vµo x0 víi x0 = 0. Var n, r : integer; x : array[0..20] of integer; count : integer; Procedure Init; begin write('n, r = '); readln(n, r); x[0] := 0; count := 0; end; Procedure Result; var i:integer; begin count := count + 1; write(count : 5, '.');

TrÇn Quèc ChiÕn To¸n rêi r¹c

Ch­¬ng 5. Bµi to¸n liÖt kª 5 − 11

for i := 1 to r do write(x[i] : 3); writeln; end; Procedure Try(i:integer); var j : integer; begin for j := x[i-1]+1 to n-r+i do begin x[i] := j; if i = r then Result else Try(i+1); end; end; Begin Init ; Try(1) ; End. 5.3.5. Bµi to¸n xÕp HËu XÐt bµi to¸n liÖt kª tÊt c¶ c¸c c¸ch xÕp n qu©n HËu trªn bµn cê n x n sao cho chóng kh«ng ¨n lÉn nhau.

§¸nh sè cét vµ dßng cña bµn cê tõ 1 ®Õn n. Mçi dßng xÕp ®óng 1 qu©n HËu. Tõ ®ã dÉn ®Õn viÖc biÓu diÔn mçi c¸ch xÕp b»ng mét ho¸n vÞ x1 , x2 ,... , xn cña {1,2,...,n}, trong ®ã xi = j nghÜa lµ qu©n HËu dßng i ®­îc xÕp vµo cét j . Nh­ vËy c¸c gi¸ trÞ ®Ò cö cho xi lµ tõ 1 ®Õn n . Gi¸ trÞ j ®­îc chÊp nhËn nÕu « (i,j) ch­a bÞ c¸c qu©n HËu tr­íc chiÕu ®Õn. §Ó kiÓm so¸t ®iÒu nµy ta cÇn ghi nhËn tr¹ng th¸i cña bµn cê tr­íc còng nh­ sau khi xÕp ®­îc mét qu©n HËu. Chó ý r»ng qu©n HËu cã thÓ chiÕu ngang, däc vµ chÐo. ViÖc kiÓm so¸t chiÒu ngang lµ kh«ng cÇn thiÕt v× mçi dßng ®­îc xÕp ®óng 1 qu©n HËu. ViÖc kiÓm so¸t chiÒu däc ®­îc thùc hiÖn nhê d·y biÕn logic a[j] víi quy ­íc a[j] = true nÕu cét j cßn trèng. Ph­¬ng tr×nh mét ®­êng chÐo lµ i + j = const (2 ≤ i+j ≤ 2n) vµ cña ®­êng chÐo thø hai lµ i - j = const (1-n ≤ i-j ≤ n-1). Tõ ®ã ®­êng chÐo thø nhÊt ®­îc ghi nhËn nhê d·y biÕn logic b[t] (2 ≤ t ≤ 2n) vµ ®­êng chÐo thø 2 nhê d·y biÕn logic c[h] (1-n ≤ h ≤ n-1) víi qui ­íc c¸c ®­êng chÐo nµy ch­a bÞ qu©n HËu khèng chÕ nÕu biÕn t­¬ng øng cã gi¸ trÞ true. Nh­ vËy gi¸ trÞ j ®­îc chÊp nhËn khi vµ chØ khi c¶ 3 biÕn a[j] , b[i+j] vµ c[i-j] cã gi¸ trÞ true. C¸c biÕn nµy cÇn g¸n false sau khi xÕp qu©n HËu dßng i vµ tr¶ l¹i true sau khi gäi Result hay Try(i+1).

TrÇn Quèc ChiÕn To¸n rêi r¹c

Ch­¬ng 5. Bµi to¸n liÖt kª 5 − 12

Const Max = 20; Var n : integer; x : array[1..Max] of integer; a : array[1..Max] of boolean; b : array[2.. 2*Max] of boolean; c : array[1-Max..Max-1] of boolean; count:integer; Procedure Init; var i : integer; begin write('n = ');readln(n); for i := 1 to n do a[i] := true; for i := 2 to 2*n do b[i] := true; for i := 1-n to n-1 do c[i] := true; count := 0; end; Procedure Result; var i:integer; begin count := count + 1; write(count : 5, '.'); for i := 1 to n do write(x[i] : 3); writeln; end; Procedure Try(i:integer); var j : integer; begin for j := 1 to n do if a[j] and b[i+j] and c[i-j] then begin {chÊp nhËn} x[i] := j; {ghi nhËn tr¹ng th¸i míi} a[j] := false; b[i+j] := false; c[i-j] := false; if i = n then Result else Try(i+1); {tr¶ l¹i tr¹ng th¸i cò} a[j] := true; b[i+j] := true; c[i-j] := true;

TrÇn Quèc ChiÕn To¸n rêi r¹c

Ch­¬ng 5. Bµi to¸n liÖt kª 5 − 13

end; end; Begin Init ; Try(1) ; End. D­íi ®©y lµ sè c¸ch xÕp HËu øng víi mét sè gi¸ trÞ n:

n = 4 5 6 7 8 9 10 11 12 13 14 Sn = 2 10 4 40 92 352 724 2680 14200 73712 365596

5.3.6. Bµi to¸n h×nh ch÷ nhËt La tinh Gi¶ sö S = {1,2,...,n}. Mét h×nh ch÷ nhËt La tinh trªn S lµ b¶ng p dßng q cét sao cho mçi dßng lµ mét chØnh hîp kh«ng lÆp chËp q cña S vµ mçi cét lµ mét chØnh hîp kh«ng lÆp chËp p cña S. Theo ®Þnh nghÜa ta cã p ≤ n vµ q ≤ n . §Æc biÖt trong tr­êng hîp q = n, mçi dßng lµ mét ho¸n vÞ cña S sao cho kh«ng cét nµo chøa phÇn tö lÆp l¹i. H×nh ch÷ nhËt La tinh d¹ng p x n gäi lµ chuÈn nÕu dßng ®Çu lµ 1,2,...,n. ThÝ dô

1 2 3 4 5 6 7 2 3 4 5 6 7 1 3 4 5 6 7 1 2

lµ mét h×nh ch÷ nhËt la tinh chuÈn trªn S = {1,2,3,4,5,6,7}. Gäi L(p,n) lµ sè h×nh ch÷ nhËt la tinh p x n vµ K(p,n) lµ sè h×nh ch÷ nhËt la tinh chuÈn p x n. Ta cã

L(p,n) = n! . K(p,n) vµ

K(2,n) = Dn trong ®ã Dn lµ sè mÊt thø tù. Sè ph©n bè Un lµ sè c¸c h×nh ch÷ nhËt la tinh chuÈn 3 x n víi 2 dßng ®Çu cè ®Þnh lµ

TrÇn Quèc ChiÕn To¸n rêi r¹c

Ch­¬ng 5. Bµi to¸n liÖt kª 5 − 14

1 2 .............. n − 1 n 2 3 .............. n n − 1 Nhµ to¸n häc Riordan J. (1946) ®· chøng minh

K(3,n) = ∑=

−−

m

kknkkn UDDknC

02..).,(

trong ®ã m = [n/2] ( [x] ký hiÖu sè nguyªn lín nhÊt ≤ x ) vµ U0 = 1. Bµi to¸n ®Õm sè h×nh ch÷ nhËt la tinh víi sè dßng nhiÒu h¬n ®Õn nay vÉn ch­a ®­îc gi¶i quyÕt. Ng­êi ta míi chØ ®­a ra mét vµi d¹ng tiÖm cËn cho L(p,n). NÕu p = q = n th× ta cã h×nh vu«ng la tinh. Mét h×nh vu«ng la tinh gäi lµ chuÈn nÕu dßng ®Çu vµ cét ®Çu lµ ho¸n vÞ 1,2,...,n−1,n. VÝ dô sau ®©y lµ h×nh vu«ng la tinh chuÈn cÊp 7

1 2 3 4 5 6 7 2 3 4 5 6 7 1 3 4 5 6 7 1 2 4 5 6 7 1 2 3 5 6 7 1 2 3 4 6 7 1 2 3 4 5 7 1 2 3 4 5 6

Gäi ln lµ sè h×nh vu«ng la tinh chuÈn ta cã

L(n,n) = n! . (n-1)! . ln

C«ng thøc tÝnh ln ®Õn nay vÉn cßn bá ngá. Tuy nhiªn ta cã thÓ lËp ch­¬ng tr×nh liÖt kª tÊt c¶ h×nh vu«ng la tinh chuÈn. D­íi ®©y lµ mét sè gi¸ trÞ

n = 1 2 3 4 5 6 7 ln = 1 1 1 4 56 9 408 16 942 080

TrÇn Quèc ChiÕn To¸n rêi r¹c

Ch­¬ng 5. Bµi to¸n liÖt kª 5 − 15

bµi tËp

1. ViÕt ch­¬ng tr×nh liÖt kª tæ hîp chËp r tõ n phÇn tö theo ph­¬ng ph¸p sinh. 2. ViÕt ch­¬ng tr×nh liÖt kª ho¸n vÞ cña n phÇn tö theo ph­¬ng ph¸p sinh. 3. ViÕt ch­¬ng tr×nh liÖt kª c¸c d·y nhÞ ph©n ®é dµi n theo ph­¬ng ph¸p sinh. 4. ViÕt ch­¬ng tr×nh liÖt kª ho¸n vÞ tæng qu¸t theo thuËt to¸n quay lui. 5. ViÕt ch­¬ng tr×nh liÖt kª tæ hîp tæng qu¸t theo thuËt to¸n quay lui. 6. ViÕt ch­¬ng tr×nh liÖt kª h×nh vu«ng la tinh chuÈn theo thuËt to¸n quay lui. 7. ViÕt ch­¬ng tr×nh liÖt kª ®­êng ®i qu©n ngùa trªn bµn cê theo thuËt to¸n quay lui.

TrÇn Quèc ChiÕn To¸n rêi r¹c

Ch­¬ng 6. Bµi to¸n tèi ­u 6 − 1

ch­¬ng 6. bµi to¸n tèi ­u

6.1. Giíi thiÖu 6.1.1. Ph¸t biÓu bµi to¸n tèi ­u Trong rÊt nhiÒu vÊn ®Ò øng dông thùc tÕ cña bµi to¸n tæ hîp c¸c cÊu h×nh tæ hîp ®­îc g¸n gi¸ trÞ b»ng sè ®¸nh gi¸ gi¸ trÞ sö dông (hay chi phÝ sö dông) cña cÊu h×nh ®èi víi môc ®Ých sö dông cô thÓ nµo ®ã. Khi ®ã xuÊt hiÖn bµi to¸n : Lùa chän trong c¸c cÊu h×nh tæ hîp chÊp nhËn cÊu h×nh tèt nhÊt ( cã gi¸ trÞ sö dông lín nhÊt hay chi phÝ sö dông thÊp nhÊt ). C¸c bµi to¸n nh­ vËy gäi lµ c¸c bµi to¸n tèi ­u tæ hîp. Ta cã thÓ ph¸t biÓu bµi to¸n tèi ­u tæ hîp mét c¸ch tæng qu¸t nh­ sau :

min (max ) {f(x) : x ∈ D} trong ®ã D lµ tËp h÷u h¹n phÇn tö. Hµm f(x) gäi lµ hµm môc tiªu, mçi phÇn tö x ∈ D gäi lµ mét ph­¬ng ¸n, cßn tËp D gäi lµ tËp c¸c ph­¬ng ¸n cña bµi to¸n. Trong bµi to¸n tèi ­u tæ hîp, tËp ph­¬ng ¸n D ®­îc m« t¶ nh­ lµ tËp c¸c cÊu h×nh tæ hîp tho¶ m·n mét sè tÝnh chÊt cho tr­íc nµo ®ã. Ph­¬ng ¸n x* ®em l¹i gi¸ trÞ nhá nhÊt (lín nhÊt) cho hµm môc tiªu gäi lµ ph­¬ng ¸n tèi ­u. Khi ®ã trÞ f(x*) gäi lµ trÞ tèi ­u cña bµi to¸n. 6.1.2. Mét sè vÝ dô a) Bµi to¸n ng­êi du lÞch Mét ng­êi du lÞch muèn ®i tham quan n thµnh phè 1,2,...,n. XuÊt ph¸t tõ thµnh phè nµo ®ã ng­êi du lÞch ®i qua tÊt c¶ c¸c thµnh phè, mçi thµnh phè chØ qua ®óng mét lÇn, sau ®ã quay vÒ n¬i xuÊt ph¸t. Chi phÝ ®i tõ thµnh phè i ®Õn thµnh phè j lµ cij . H·y t×m ®­êng ®i sao cho tæng chi phÝ lµ nhá nhÊt. Râ rµng ta cã thÓ thiÕt lËp quan hÖ t­¬ng øng 1-1 gi÷a hµnh tr×nh

v = v(1)→ v(2)→ ... → v(n−1)→ v(n)→ v(1) trong ®ã v(i) ký hiÖu thµnh phè thø i trªn hµnh tr×nh, víi ho¸n vÞ (v(1), v(2), ... , v(n) ). §Æt

TrÇn Quèc ChiÕn To¸n rêi r¹c

Ch­¬ng 6. Bµi to¸n tèi ­u 6 − 2

f(v) = cv(1),v(2) + cv(2),v(3) + ... + cv(n-1),v(n) + cv(n),v(1)

Ký hiÖu P lµ tËp tÊt c¶ c¸c ho¸n vÞ cña {1,2,...,n}. Khi ®ã bµi to¸n ng­êi du

lÞch cã thÓ ph¸t biÓu d­íi d¹ng bµi to¸n tèi ­u tæ hîp sau :

min { f(v) : v ∈ P } Cã thÓ thÊy r»ng tæng sè hµnh tr×nh cña ng­êi du lÞch lµ n!, trong ®ã chØ cã (n−1)! hµnh tr×nh thùc sù kh¸c nhau ( bëi v× cã thÓ xuÊt ph¸t tõ thµnh phè bÊt kú nªn cã thÓ cè ®Þnh mét thµnh phè nµo ®ã). b) Bµi to¸n ba-l« Mét nhµ th¸m hiÓm cÇn mang theo mét sè ®å vËt cho mét chuyªn ®i xa. Cã n vËt víi träng l­îng t­¬ng øng lµ a1, a2, ..., an (kg) vµ gi¸ trÞ sö dông t­¬ng øng lµ c1, c2, ..., cn . ChiÕc ba l« chØ cho phÐp ®ùng tèi ®a b (kg). Bµi to¸n ®Æt ra lµ nhµ th¸m hiÓm cÇn chän nh÷ng vËt nµo mang theo ®Ó tæng gi¸ trÞ sö dông lµ lín nhÊt ? Mét ph­¬ng ¸n ®em ®å cña nhµ th¸m hiÓm ®­îc xem nh­ mét vect¬ nhÞ ph©n ®é dµi n : x = (x1, x2, ..., xn), trong ®ã xi = 1 cã nghÜa lµ ®å vËt thø i ®­îc mang theo vµ xj = 0 cã nghÜa lµ ®å vËt thø j kh«ng ®­îc mang theo. Víi ph­¬ng ¸n x, gi¸ trÞ sö dông lµ

f(x) = ∑=

n

iii xc

1.

vµ tæng träng l­îng lµ

g(x) = ∑=

n

iii xa

1.

Gäi S lµ tËp c¸c vecto nhÞ ph©n ®é dµi n, bµi to¸n cã thÓ ph¸t biÓu d­íi d¹ng

max { f(x) : x ∈ S & g(x) ≤ b } c) Bµi to¸n cho thuª phßng Mét kh¸ch s¹n cã mét phßng (häp, héi th¶o, ...) ®Ó cho thuª. §Çu n¨m kh¸ch s¹n nhËn ®­îc c¸c yªu cÇu thuª phßng cña n kh¸ch hµng. Mçi kh¸ch hµng i sÏ cho biÕt tËp Ni c¸c ngµy thuª (trän ngµy). Kh¸ch s¹n chØ cã quyÒn nhËn hoÆc tõ chèi yªu cÇu cña kh¸ch hµng. Hái kh¸ch s¹n ph¶i chän kh¸ch hµng nh­ thÕ nµo ®Ó tæng sè ngµy sö dông phßng lµ cao nhÊt. Ký hiÖu I = {1,2,...,n} lµ tËp c¸c chØ sè kh¸ch hµng. Khi ®ã tËp c¸c ph­¬ng

TrÇn Quèc ChiÕn To¸n rêi r¹c

Ch­¬ng 6. Bµi to¸n tèi ­u 6 − 3

¸n cho thuª phßng lµ

D = { J ⊂ I : Nk ∩ Nl = ∅ ∀ k, l ∈ J , k ≠ l } Víi mçi ph­¬ng ¸n J ∈ D , sè ngµy sö dông phßng lµ

f(J) = ∑∈Jj

jN

Bµi to¸n ®­a vÒ d¹ng

max { f(J) : J ∈ D } d) Bµi to¸n lËp lÞch gia c«ng Gi¶ sö mçi chi tiÕt trong n chi tiÕt 1, 2, ... , n, cÇn ph¶i gia c«ng lÇn l­ît trªn m m¸y 1, 2, ... , m. Thêi gian gia c«ng chi tiÕt j trªn m¸y i lµ tij . H·y t×m lÞch (tr×nh tù gia c«ng) c¸c chi tiÕt trªn c¸c m¸y sao cho hoµn tÊt gia c«ng tÊt c¶ chi tiÕt lµ sím nhÊt. Ta sÏ xÐt bµi to¸n nµy víi thªm gi¶ thiÕt lµ c¸c chi tiÕt ph¶i ®­îc gia c«ng mét c¸ch liªn tôc, tøc lµ qu¸ tr×nh gia c«ng cña mçi chi tiÕt ph¶i ®­îc tiÕn hµnh liªn tôc, kh«ng cho phÐp thêi gian dõng chê ®îi khi chuyÓn tõ m¸y nµy sang m¸y kh¸c. T×nh huèng nh­ vËy rÊt hay gÆp trong s¶n xuÊt c«ng nghiÖp. Ch¼ng h¹n trong d©y chuyÒn luyÖn thÐp vËt liÖu ph¶i ®­îc gia c«ng liªn tôc tr¸nh gi¸n ®o¹n lµm gi¶m nhiÖt ®é c¶n trë viÖc gia c«ng tiÕp theo. Râ rµng mçi lÞch gia c«ng t­¬ng øng mét ho¸n vÞ v = (v(1),v(2),...,v(n)) cña n sè tù nhiªn 1,2,...,n. Thêi gian hoµn thµnh theo lÞch v ®­îc tÝnh theo c«ng thøc:

f(v) = ∑−

=+

1

1)1(),(

n

jjvjvc + ∑

=

m

knvkt

1)(,

trong ®ã cij = Sj − Si , Sj lµ thêi ®iÓm b¾t ®Çu thùc hiÖn gia c«ng chi tiÕt j (i,j = 1,2,...,n). ý nghÜa cña cij cã thÓ gi¶i thÝch ®©y lµ tæng thêi gian gi¸n ®o¹n (®­îc tÝnh kÓ tõ khi b¾t ®Çu gia c«ng chi tiÕt i) g©y ra bëi chi tiÕt j khi nã ®­îc gia c«ng sau chi tiÕt i. V× vËy cij ®­îc tÝnh theo c«ng thøc:

cij =

− ∑∑−

==≤≤

1

111max

k

llj

k

llimk

tt , i, j = 1, 2, ..., n

Ký hiÖu P lµ tËp tÊt c¶ ho¸n vÞ cña {1,2,...,n} ta cã bµi to¸n tèi ­u sau:

TrÇn Quèc ChiÕn To¸n rêi r¹c

Ch­¬ng 6. Bµi to¸n tèi ­u 6 − 4

min {f(v) : v ∈ P } e. Ph­¬ng ph¸p ®iÓm diÖn Mét trong nh÷ng ph­¬ng ph¸p hiÓn nhiªn nhÊt ®Ó gi¶i bµi to¸n tèi ­u tæ hîp lµ: Trªn c¬ së c¸c thuËt to¸n liÖt kª tæ hîp ta duyÖt tõng ph­¬ng ¸n vµ tÝnh trÞ hµm môc tiªu t¹i nã, so s¸nh víi gi¸ trÞ tèt nhÊt ®Õn thêi ®iÓm ®ã. Ph­¬ng ph¸p nµy gäi lµ ph­¬ng ph¸p ®iÓm diÖn. Ph­¬ng ph¸p nµy hiÖu qu¶ thÊp. VÝ dô ta ph¶i liÖt kª

15! = 1 307 674 368 000 ho¸n vÞ víi tèc ®é tÝnh to¸n 1 tû phÐp tÝnh trªn gi©y. Gi¶ sö mçi ho¸n vÞ cÇn 100 phÐp to¸n. Khi ®ã ph¶i hÕt 130 767 gi©y ( 36 giê) míi liÖt kª hÕt.

TrÇn Quèc ChiÕn To¸n rêi r¹c

Ch­¬ng 6. Bµi to¸n tèi ­u 6 − 5

6.2. ThuËt to¸n Johnson gi¶i bµi to¸n lËp lÞch gia c«ng trªn 2 m¸y XÐt bµi to¸n lËp lÞch gia c«ng trªn 2 m¸y A, B. §©y lµ tr­êng hîp riªng cña bµi to¸n lËp lÞch nªu trong môc tr­íc. Mçi chi tiÕt i cÇn gia c«ng lÇn l­ît trªn m¸y A vµ B víi thêi gian t­¬ng øng lµ ai vµ bi (i=1,...,n). H·y t×m lÞch (tr×nh tù) gia c«ng sao cho hoµn thµnh gia c«ng tÊt c¶ chi tiÕt lµ sím nhÊt. Gi¶ thiÕt r»ng tr×nh tù gia c«ng trªn 2 m¸y lµ nh­ nhau (tr­êng hîp tr×nh tù kh¸c nhau còng quy vÒ tr­êng hîp nµy). Khi ®ã mçi lÞch gia c«ng sÏ t­¬ng øng mét ho¸n vÞ

v = (v(1), v(2),..., v(n)) cña n sè tù nhiªn 1,2,...,n. Víi i = 1,2,...,n vµ x = A, B ta ký hiÖu s(i,x) lµ thêi ®iÓm b¾t ®Çu gia c«ng chi tiÕt i trªn m¸y x f(i,x) lµ thêi ®iÓm kÕt thóc gia c«ng chi tiÕt i trªn m¸y x Víi mäi lÞch gia c«ng v, ta cã thÓ gi¶ thiÕt m¸y A b¾t ®Çu gia c«ng ë thêi ®iÓm s(v(1),A) = 0 (1) Râ rµng lµ f(i,A) = s(i,A) + ai ; f(i,B) = s(i,B) + bi ,∀i=1, 2,..., n (2) M¸y A b¾t ®Çu gia c«ng chi tiÕt v(i) chØ sau khi gia c«ng xong chi tiÕt v(i−1), tøc lµ s(v(i),A) ≥ f(v(i-1),A) , i = 2,3,...,n. (3) M¸y B cã thÓ b¾t ®Çu gia c«ng chi tiÕt v(1) sau khi m¸y A gia c«ng xong chi tiÕt v(1)

TrÇn Quèc ChiÕn To¸n rêi r¹c

Ch­¬ng 6. Bµi to¸n tèi ­u 6 − 6

s(v(1),B) ≥ f(v(1),A) (4) M¸y B b¾t ®Çu gia c«ng chi tiÕt v(i) chØ sau khi gia c«ng xong chi tiÕt v(i−1) vµ sau khi m¸y A gia c«ng xong chi tiÕt v(i), tøc lµ s(v(i),B) ≥ max{ f (v(i-1),B), f (v(i),A)} , i = 2,3,...,n. (5) Thêi gian ®Ó hoµn thµnh lÞch gia c«ng v lµ T(v) = f(v(n),B) (6) Râ rµng lµ víi mçi lÞch gia c«ng v, T(v) ®¹t gi¸ trÞ nhá nhÊt khi tÊt c¶ c¸c dÊu bÊt ®¼ng thøc ë (3) , (4), (5) trªn trë thµnh ®¼ng thøc, tøc lµ s(v(i),A) = f(v(i-1),A) , i = 2,3,...,n. (3') s(v(1),B) = f(v(1),A) (4') s(v(i),B) = max{ f (v(i-1),B), f(v(i),A)} , i = 2,3,...,n. (5') ◊ VÝ dô: XÐt bµi to¸n víi 5 chi tiÕt. Thêi gian gia c«ng c¸c chi tiÕt cho ë b¶ng sau

Chi tiÕt M¸y 1 2 3 4 5 A 3 4 6 5 6 B 3 3 2 7 3

Gi¶ sö lÞch gia c«ng lµ

v = (1,2,3,4,5,6) Khi ®ã theo c¸c c«ng thøc (1),(2),(3'),(4'), (5') ta tÝnh ®­îc s(1,A) = 0 ; f(1,A) = s(1,A) + a1 = 0 + 3 = 3 s(2,A) = f(1,A) = 3 ; f(2,A) = s(2,A) + a2 = 3 + 4 = 7 s(3,A) = f(2,A) = 7 ; f(3,A) = s(3,A) + a3 = 7 + 6 = 13 s(4,A) = f(3,A) = 13; f(4,A) = s(4,A) + a4 = 13 + 5 = 18 s(5,A) = f(4,A) = 18; f(5,A) = s(5,A) + a5 = 18 + 6 = 24 s(1,B) = f(1,A) = 3; f(1,B) = s(1,B) + b1 = 3 + 3 = 6 s(2,B) = max {f(1,B), f(2,A) }= max {3, 7} = 7 ; f(2,B) = s(2,B) + b2 = 7 + 3 = 10 s(3,B) = max {f(2,B), f(3,A) }= max {13, 10} = 13 ; f(3,B) = s(3,B) + b3 = 13 + 2 = 15

TrÇn Quèc ChiÕn To¸n rêi r¹c

Ch­¬ng 6. Bµi to¸n tèi ­u 6 − 7

s(4,B) = max {f(3,B), f(4,A) }= max {15, 18} = 18 ; f(4,B) = s(4,B) + b4 = 18 + 7 = 25 s(5,B) = max {f(4,B), f(5,A) }= max {25, 24} = 25 ; f(5,B) = s(5,B) + b5 = 25 + 3 = 28 §Ó biÓu diÔn lÞch gia c«ng ng­êi ta th­êng sö dông s¬ ®å Gant. KÕt qu¶ trong vÝ dô trªn ta cã thÓ biÓu diÔn nh­ sau:

1 2 3 4 5 1 2 3 4 5

B A 0 1 2 3 4 5 6 7 8 9 10 11 12 13 14 15 16 17 18 19 20 21 22 23 24 25 26 27 28 29

Thêi gian hoµn thµnh lÞch gia c«ng lµ T(v) = f(5,B) = 28. Ta còng nhËn xÐt r»ng m¸y B cã c¸c kho¶ng thêi gian chÕt ®Ó chê gia c«ng chi tiÕt tiÕp theo. Cè ®Þnh thêi ®iÓm f(5,B), ta cã thÓ ®Èy thêi gian gia c«ng trªn m¸y B sang ph¶i sao cho kh«ng cã thêi gian chÕt gi÷a chõng, tøc lµ m¸y B ho¹t ®éng liªn tôc. KÕt qu¶ ta cã s¬ ®å sau

1 2 3 4 5 1 2 3 4 5

B A 0 1 2 3 4 5 6 7 8 9 10 11 12 13 14 15 16 17 18 19 20 21 22 23 24 25 26 27 28 29

Nh­ vËy ®Ó c¸c m¸y ho¹t ®éng liªn tôc c¸c thêi ®iÓm sÏ thay ®æi nh­ sau: s(1,B) = 10; s(2,B) = 13 ; s(3,B) = 16. Trong tr­êng hîp tæng qu¸t ta còng cã thÓ gi¶ thiÕt hai m¸y lµm viÖc liªn tôc. Gäi dB(v) lµ thêi ®iÓm m¸y B b¾t ®Çu thùc hiÖn gia c«ng theo lÞch v (trong vÝ dô trªn dB(v) = 10). Khi ®ã ta cã

T(v) = dB(v) + ∑=

n

iib

1

trong ®ã sè h¹ng thø 2 kh«ng phô thuéc lÞch gia c«ng v. VÊn ®Ò cßn l¹i lµ t×m lÞch gia c«ng cho dB(v) nhá nhÊt. Ta cÇn t×m c«ng thøc tÝnh dB(v). DÔ thÊy r»ng dB(v) lµ tæng av(1) vµ c¸c kho¶ng thêi gian chÕt cña m¸y B, nÕu ta bè trÝ m¸y B theo c¸c c«ng thøc (1), (2), (3'), (4') vµ (5'). Ta cã c«ng thøc sau

TrÇn Quèc ChiÕn To¸n rêi r¹c

Ch­¬ng 6. Bµi to¸n tèi ­u 6 − 8

dB(v) = ( )vknk∆

≤≤1max

trong ®ã ∆1(v) = av(1) (7)

∆k(v) = ∑=

k

iiva

1)( − ∑

=

1

1)(

k

iivb = ∆k−1(v) + av(k) − bv(k−1)

k = 2, 3,..., n.

Trong vÝ dô trªn ta cã ∆1(v) = 3 ∆2(v) = 3 + 4 - 3 = 4 ∆3(v) = 4 + 6 - 3 = 7 ∆4(v) = 7 + 5 - 2 = 10 ∆5(v) = 10 + 6 - 7 = 9 VËy dB(v) = 10. Bµi to¸n ®­îc quy vÒ min { dB(v) : v ∈ P } (*) • Bæ ®Ò 1. Gi¶ sö lÞch gia c«ng v’ thu ®­îc tõ lÞch gia c«ng v b»ng c¸ch ho¸n vÞ 2 phÇn tö v(k) vµ v(k+1):

v’ = ( v(1),..., v(k-1), v(k+1), v(k), v(k+2), ..., v(n)). Khi ®ã nÕu

min { av(k), bv(k+1) } ≤ min { bv(k), av(k+1) } (8) th× dB(v) ≤ dB(v') (9) Chøng minh. Do v vµ v' chØ kh¸c nhau ë vÞ trÝ thø k vµ k+1 nªn ta cã

TrÇn Quèc ChiÕn To¸n rêi r¹c

Ch­¬ng 6. Bµi to¸n tèi ­u 6 − 9

∆i(v) = ∆i(v') , ∀i = 1, 2, ..., k−1, k+2, ..., n. Tõ ®ã ®Ó chøng minh (9), theo (7) ta chØ cÇn chøng minh max {∆k(v), ∆k+1(v) } ≤ max {∆k(v’), ∆k+1(v’) } (10) ThËt vËy (10) t­¬ng ®­¬ng víi max {∆k(v) - x , ∆k+1(v) – x } ≤ max {∆k(v’) – x, ∆k+1(v’) – x } trong ®ã x lµ gi¸ trÞ bÊt kú. Chän

x = ∑+

=

1

1)(

k

iiva − ∑

=

1

1)(

k

iivb

ta nhËn ®­îc bÊt ®¼ng thøc t­¬ng ®­¬ng max { −av(k+1), −bv(k) } ≤ max { −av(k), −bv(k+1) } ⇔ − min { av(k+1), bv(k) } ≤ − min { av(k), bv(k+1) } ⇔ min { av(k), bv(k+1) } ≤ min { av(k+1), bv(k) } NghÜa lµ (10) t­¬ng ®­¬ng (8). Ta cã ®pcm. • Bæ ®Ò 2. NÕu i, j , k lµ ba chØ sè tho¶ m·n min { ai , bj } ≤ min { aj , bi } (11) vµ min { aj , bk } ≤ min { ak , bj } (12) th× min { ai , bk } ≤ min { ak , bi } (13) Chøng minh. Gi¶ sö trong (11) ta cã ai ≤ bj vµ aj ≤ bi , cßn trong (12) ta cã aj ≤ bk vµ ak ≤ bj . Khi ®ã tõ (11) suy ra ai ≤ aj vµ tõ (12) suy ra aj ≤ ak . Tøc lµ ta cã ai ≤ ak vµ tõ ®ã suy ra (13). C¸c tr­êng hîp kh¸c chøng minh t­¬ng tù. • PhÐp ho¸n vÞ. Cho lÞch gia c«ng

v = (v(1), v(2), ..., v(n))

TrÇn Quèc ChiÕn To¸n rêi r¹c

Ch­¬ng 6. Bµi to¸n tèi ­u 6 − 10

PhÐp ho¸n vÞ i↔j lªn v cho ta lÞch gia c«ng v' thu ®­îc tõ v b»ng c¸ch ho¸n vÞ hai phÇn tö v(i) vµ v(j). • §Þnh lý Johnson (1954). T(v) ®¹t gi¸ trÞ nhá nhÊt khi lÞch gia c«ng

v = (v(1), v(2), ..., v(n)) tho¶ m·n min { av(k), bv(k+1) } ≤ min { bv(k), av(k+1) }, ∀ k = 1,2,...,n-1 (14) Chøng minh. Cho lÞch gia c«ng

v' = (v'(1), v'(2), ..., v'(n)) bÊt kú. Ta x©y dùng d·y ho¸n vÞ

v = v0 , v1, v2,..., vk, vk+1, ..., vm = v' nh­ sau: §Æt v0 = v. Gi¶ sö ®· cã vk vµ vk ≠ v' ta x©y dùng vk+1 qua c¸c b­íc sau: - T×m tham sè p nhá nhÊt (p < n) cã vk(p) ≤ v'(p). - T×m tham sè q tho¶ vk(q) = v'(p) . HiÓn nhiªn lµ q > p, v× vk(j) = v'(j),∀j <p - Thùc hiÖn liªn tiÕp (q−p) phÐp ho¸n vÞ q↔q−1, q−1↔q−2, ... , p↔p+1 lªn vk . §Æt vk+1 lµ ho¸n vÞ kÕt qu¶. Nh­ vËy sau k b­íc ta cã Ýt nhÊt k phÈn tö ®Çu cña vk vµ v' trïng nhau. B»ng c¸ch nµy sau nhiÒu nhÊt n b­íc ta nhËn ®­îc v'. V× ta chØ thùc hiÖn c¸c phÐp ho¸n vÞ hai phÇn tö kÒ nhau vµ Bæ ®Ò 2 lu«n ®¶m b¶o tho¶ m·n ®iÒu kiÖn (8) cña Bæ ®Ò 1, nªn , ¸p dông liªn tiÕp Bæ ®Ò 1, ta cã

dB(v) = dB(v0) ≤ dB(v1) ≤ dB(v2) ≤ ... ≤ dB(vk) ≤ dB(vk+1) ≤ ... ≤ dB(vm) = dB(v') Nh­ vËy ta chøng minh ®­îc

dB(v) ≤ dB(v') , ∀ v' tøc lµ v lµ lÞch gia c«ng tèi ­u. Tõ ®Þnh lý trªn ta nhËn ®­îc thuËt to¸n sau.

TrÇn Quèc ChiÕn To¸n rêi r¹c

Ch­¬ng 6. Bµi to¸n tèi ­u 6 − 11

• ThuËt to¸n Johnson (i) Chia c¸c chi tiÕt thµnh 2 nhãm:

N1 = { i : ai < bi } vµ

N2 = { i : ai > bi } C¸c chi tiÕt cã ai = bi xÕp vµo nhãm nµo còng ®­îc. (ii) S¾p xÕp N1 theo chiÒu t¨ng cña ai vµ N2 theo chiÒu gi¶m cña bi. (iii) Nèi N2 vµo ®u«i N1 . D·y thu ®­îc lµ lÞch gia c«ng tèi ­u. ◊ VÝ dô. Quay l¹i vÝ dô trªn. Ta cã b¶ng thêi gian Chi tiÕt

M¸y 1 2 3 4 5 A 3 4 6 5 6 B 3 3 2 7 3

Thùc hiÖn thuËt to¸n Johnson theo tõng b­íc (i) Chia nhãm :

N1 = {1, 4} & N2 = {2, 3, 5} (ii) S¾p xÕp : N1 theo chiÒu t¨ng cña ai vµ N2 theo chiÒu gi¶m cña bi

N1 = {1, 4} & N2 = {2, 5, 3} (iii) Nèi N2 vµo ®u«i N1 :

v = (1, 4, 2, 5, 3) S¬ ®å Gant cña lÞch gia c«ng tèi ­u thu ®­îc nh­ sau:

1 4 2 5 3 1 4 2 5 3

B A 0 1 2 3 4 5 6 7 8 9 10 11 12 13 14 15 16 17 18 19 20 21 22 23 24 25 26 27 28 29

hoÆc víi ph­¬ng ¸n m¸y B ho¹t ®éng liªn tôc

TrÇn Quèc ChiÕn To¸n rêi r¹c

Ch­¬ng 6. Bµi to¸n tèi ­u 6 − 12

1 4 2 5 3 1 4 2 5 3

B A 0 1 2 3 4 5 6 7 8 9 10 11 12 13 14 15 16 17 18 19 20 21 22 23 24 25 26 27 28 29

vµ gi¸ trÞ tèi ­u T(v) = 26.

TrÇn Quèc ChiÕn To¸n rêi r¹c

Ch­¬ng 7. §¹i sè Boole 7 − 1

ch­¬ng 7. ®¹i sè boole

Tªn tuæi nhµ to¸n häc thÕ kû 19 George Boole g¾n liÒn víi nhiÒu kh¸i niÖm to¸n häc quan träng nh­ §¹i sè Boole, hµm Boole, BiÓu thøc Boole, Vµnh Boole, .... Boole lµ mét trong nh÷ng nhµ khoa häc tiªn phong nghiªn cøu c¬ chÕ biÓu diÔn qu¸ tr×nh t­ duy l«gic. N¨m 1854 «ng viÕt cuèn C¸c qui luËt t­ duy. §ãng gãp lín nhÊt cña Boole lµ ph¸t triÓn lý thuyÕt l«gic b»ng ký hiÖu thay cho tõ ng÷. GÇn 100 n¨m sau, n¨m 1938, C.E. Shannon ®· ph¸t hiÖn ra r»ng cã thÓ sö dông §¹i sè Boole ®Ó nghiªn cøu m¹ch ®iÖn. Trong ch­¬ng nµy chóng ta sÏ nghiªn cøu c¸c tÝnh chÊt c¬ b¶n cña ®¹i sè Boole. 7.1. ®¹i sè boole 7.1.1. §¹i sè Boole • §Þnh nghÜa 1. §¹i sè Boole lµ hÖ thèng {S, +, ⋅ , , 0, 1} , trong ®ã tËp S chøa phÇn tö 0 vµ 1, phÐp lÊy tæng Boole + vµ phÐp lÊy tÝch Boole ⋅ lµ c¸c phÐp to¸n 2 ng«i trªn S vµ phÐp bï Boole lµ phÐp to¸n 1 ng«i trªn S tho¶ m·n c¸c tÝnh chÊt sau ®©y. (1) LuËt kÕt hîp

∀ x, y, z ∈ S : (x + y) + z = x + (y + z) & (x ⋅ y) ⋅ z = x ⋅ (y ⋅ z) (2) LuËt giao ho¸n

∀ x, y ∈ S : x + y = y + x & x ⋅ y = y ⋅ x (3) LuËt ph©n phèi

∀ x, y, z ∈ S : x ⋅ (y + z) = (x ⋅ y) + (x ⋅ z) & x + (y ⋅ z) = (x + y) ⋅ (x + z)

(4) LuËt ®ång nhÊt

∀ x ∈ S : x + 0 = x & x ⋅ 1 = x (5) LuËt bï trõ

∀ x ∈ S ∃x ∈ S : x +x = 1 & x .x = 0

TrÇn Quèc ChiÕn To¸n rêi r¹c

Ch­¬ng 7. §¹i sè Boole 7 − 2

◊ VÝ dô 1. Cho U lµ tËp vò trô vµ S lµ tËp tÊt c¶ tËp con cña U. Ta ®Þnh nghÜa c¸c phÐp to¸n trªn S nh­ sau

∀ X, Y ∈ S : X + Y = X ∪ Y & X ⋅ Y = X ∩ Y & X = U \ X TËp rçng ®ãng vai trß phÇn tö 0 vµ tËp U ∈ S ®ãng vai trß phÇn tö 1. Khi ®ã hÖ thèng { S, ∪, ∩, , ∅, U} lµ §¹i sè Boole. ◊ VÝ dô 2. Cho tËp B = {0, 1} víi c¸c phÐp to¸n sau PhÐp bï : 0 = 1, 1 = 0 PhÐp + : 1 + 1 = 1, 1 + 0 = 1, 0 + 1 = 1, 0 + 0 = 0 PhÐp ⋅ : 1 ⋅ 1 = 1, 1 ⋅ 0 = 0, 0 ⋅ 1 = 0, 0 ⋅ 0 = 0 lµ mét ®¹i sè Boole. • §Þnh lÝ 1. Trong §¹i sè Boole, phÇn tö bï lµ duy nhÊt. §Æc biÖt, nÕu

x + y = 1 & x ⋅ y = 0 th× y = x. Chøng minh y = y ⋅ 1 = y ⋅ (x + x ) = y ⋅ x + y ⋅ x = x ⋅ y + y ⋅ x = 0 + y ⋅ x = x ⋅x + y ⋅ x = x ⋅ x + x ⋅ y = x ⋅ ( x + y ) = x ⋅ 1

= x • §Þnh nghÜa 2. PhÇn tö x gäi lµ phÇn bï cña x • §Þnh lý 2. Cho §¹i sè Boole {S, +, ⋅ , , 0, 1} . Khi ®ã ta cã c¸c tÝnh chÊt sau (6) LuËt luü ®¼ng (Idempotent)

∀ x ∈ S : x + x = x & x ⋅ x = x (7) LuËt giíi néi (Bound)

∀ x ∈ S : x + 1 = 1 & x ⋅ 0 = 0 (8) LuËt hÊp thô (Absortion)

TrÇn Quèc ChiÕn To¸n rêi r¹c

Ch­¬ng 7. §¹i sè Boole 7 − 3

∀ x, y ∈ S : x + x ⋅ y = x & x ⋅ (x + y) = x (9) LuËt bï kÐp (Involution)

∀ x ∈ S : x = x (10) LuËt 0 vµ 1

0 = 1 & 1 = 0 (11) LuËt de Morgan

∀ x, y ∈ S : yx + = x ⋅y & yx ⋅ = x + y Chøng minh (6) x = x + 0 , luËt ®ång nhÊt (4)

= x + x.x , luËt bï trõ (5) = (x + x).(x + x) , luËt ph©n phèi (3) = (x + x).1 , luËt bï trõ (5) = x + x , luËt ®ång nhÊt (4)

x = x . 1 , luËt ®ång nhÊt (4) = x .(x + x) , luËt bï trõ (5) = x . x + x .x , luËt ph©n phèi (3) = x . x , luËt bï trõ (5)

(7) x + 1 = (x + 1).1 , luËt ®ång nhÊt (4) = (x + 1).(x +x) , luËt bï trõ (5) = x + 1.x , luËt ph©n phèi (3) = x +x.1 , luËt giao ho¸n (2) = x +x , luËt ®ång nhÊt (4) = 1 , luËt bï trõ (5)

x . 0 = x . 0 + 0 , luËt ®ång nhÊt (4) = x . 0 + x .x , luËt bï trõ (5) = x.( 0 + x) , luËt ph©n phèi (3) = x.x , luËt ®ång nhÊt (4) = 0 , luËt bï trõ (5)

(8) x + x.y = x.1 + x.y , luËt ®ång nhÊt (4) = x.(1 + y) , luËt ph©n phèi (3) = x.1 , luËt giíi néi (7) = x , luËt ®ång nhÊt (4)

x.(x+y) = x.x + x.y , luËt ph©n phèi (3)

TrÇn Quèc ChiÕn To¸n rêi r¹c

Ch­¬ng 7. §¹i sè Boole 7 − 4

= x + x.y , luËt luü ®¼ng (6) = x , ®¼ng thøc trªn

(9) x = x + 0 , luËt ®ång nhÊt (4) = x + (x +x) , luËt bï trõ (5) = x + (x + x) , luËt giao ho¸n (2) = ( x + x ) + x , luËt kÕt hîp (1) = (x + x ) + x , luËt giao ho¸n (2) = x , luËt bï trõ (5)

(10) 0 = 0 + 0 , luËt ®ång nhÊt (4) = 1 , luËt bï trõ (5)

1 = 1.1 , luËt ®ång nhÊt (4) = 0 , luËt bï trõ (5)

(11) LuËt de Morgan

§Ó chøng minh: yx + = x ⋅y

theo ®Þnh lÝ 1 ta chØ cÇn chøng minh (x + y).x ⋅y = 0 (*)

vµ (x + y) +x ⋅y = 1 (**)

Ta cã (x + y).x ⋅y = x ⋅y . (x + y) , luËt giao ho¸n (2) = (x ⋅y) . x + (x ⋅y) . y , luËt ph©n phèi (3) = (y .x ) . x + (x ⋅y) . y , luËt giao ho¸n (2) = y .(x . x) + x ⋅(y . y) , luËt kÕt hîp (1) = y .(x.x ) + x ⋅(y.y ) , luËt giao ho¸n (2) = y . 0 + x ⋅ 0 , luËt bï trõ (5) = 0 + 0 , luËt giíi néi (7)

= 0 , luËt ®ång nhÊt (4)

vµ (x + y) +x ⋅y = ((x + y) +x )⋅((x + y) +y ) , luËt ph©n phèi (3)

= ((y + x) +x )⋅((x + y) +y ) , luËt giao ho¸n (2) = (y + (x +x ))⋅(x + (y +y )) , luËt kÕt hîp (1) = (y + 1 )⋅(x + 1 ) , luËt bï trõ (5) = 1 + 1 , luËt giíi néi (7)

= 1 , luËt ®ång nhÊt (4) TiÕp theo, theo ®¼ng thøc trªn ta cã

TrÇn Quèc ChiÕn To¸n rêi r¹c

Ch­¬ng 7. §¹i sè Boole 7 − 5

yx + = x . y = x.y Tõ ®ã , sö dông luËt bï kÐp, suy ra

yx ⋅ = yx + =x + y • §Þnh nghÜa 3. §èi ngÉu cña mét biÓu thøc Boole lµ biÓu thøc nhËn ®­îc tõ biÓu thøc ®· cho b»ng c¸ch thÕ 0 b»ng 1, 1 b»ng 0, + b»ng . vµ . b»ng +. ◊ VÝ dô 3. §èi ngÉu cña biÓu thøc

yx + = x ⋅y lµ biÓu thøc

yx ⋅ = x + y • §Þnh lý 3 (Nguyªn lý ®èi ngÉu) . §èi ngÉu cña mét ®¼ng thøc Boole còng lµ ®¼ng thøc Boole. Chøng minh Gi¶ sö T lµ ®¼ng thøc Boole vµ P lµ chøng minh cña T. P bao gåm d·y c¸c ®¼ng thøc c¬ b¶n cho ë ®Þnh nghÜa 1. Ký hiÖu P’ lµ d·y c¸c ®¼ng thøc ®èi ngÉu cña P (chó ý r»ng mçi ®¼ng thøc ë ®Þnh nghÜa 1 ®Òu cã ®¼ng thøc ®èi ngÉu). Khi ®ã P’ chÝnh lµ chøng minh cña ®èi ngÉu T’ cña T.

TrÇn Quèc ChiÕn To¸n rêi r¹c

Ch­¬ng 7. §¹i sè Boole 7 − 6

7.1.2. Hµm Boole • §Þnh nghÜa 1. Cho B = {0, 1}. BiÕn x gäi lµ biÕn Boole, nÕu nã chØ nhËn c¸c gi¸ trÞ trong B.

Mét hµm tõ Bn vµo B, f(x1, ..., xn), gäi lµ hµm Boole bËc n. C¸c hµm Boole th­êng ®­îc biÓu diÔn b»ng b¶ng. ◊ VÝ dô 1. Hµm f : B2 → B , f(x,y) = 1 khi x = 1, y = 0 vµ f(x,y) = 0 trong c¸c tr­êng hîp kh¸c ®­îc biÓu diÔn b»ng b¶ng sau

x y f(x,y) 1 1 0 0

1 0 1 0

0 1 0 0

C¸c hµm Boole còng cã thÓ ®­îc biÓu diÔn bëi c¸c biÓu thøc Boole. • §Þnh nghÜa 2. Cho B = {0, 1} víi c¸c phÐp to¸n bï, tæng vµ tÝch Boole. BiÓu thøc Boole víi c¸c biÕn x1, ..., xn, ®­îc ®Þnh nghÜa ®Ö quy nh­ sau

(1) 0, 1, x1, ..., xn lµ biÓu thøc Boole (2) NÕu E lµ biÓu thøc Boole, th× E còng lµ biÓu thøc Boole

(3) NÕu E1 vµ E2 lµ c¸c biÓu thøc Boole, th× E1 + E2 vµ E1⋅E2 còng lµ biÓu thøc Boole. ◊ VÝ dô 2. T×m b¶ng gi¸ trÞ cña hµm f(x,y,z) cho bëi biÓu thøc sau

f(x,y,z) = x ⋅ y + z

Gi¶i C¸c gi¸ trÞ cña hµm cho bëi b¶ng sau

x y z x ⋅ y z f(x,y,z) = x ( y +( z

1 1 1 1 0 0 0 0

1 1 0 0 1 1 0 0

1 0 1 0 1 0 1 0

1 1 0 0 0 0 0 0

0 1 0 1 0 1 0 1

1 1 0 1 0 1 0 1

• §Þnh nghÜa 3. Hai hµm Boole f(x1, ..., xn) vµ g(x1, ..., xn) gäi lµ b»ng nhau nÕu

TrÇn Quèc ChiÕn To¸n rêi r¹c

Ch­¬ng 7. §¹i sè Boole 7 − 7

f(x1, ..., xn) = g(x1, ..., xn) ∀(x1, ..., xn) ∈ Bn

Hai biÓu thøc Boole gäi lµ t­¬ng ®­¬ng , nÕu chóng cïng biÓu diÔn mét hµm. ◊ VÝ dô 3. C¸c biÓu thøc x ⋅ y, x ⋅ y + 0, x ⋅ y ⋅ 1 t­¬ng ®­¬ng nhau. • §Þnh nghÜa 4. PhÇn bï cña hµm f(x1, ..., xn) lµ hµm f ®Þnh nghÜa nh­ sau

f(x1, ..., xn) = ( )nxxf ,...,1 ∀(x1, ..., xn) ∈ Bn

Tæng Boole cña hµm f(x1, ..., xn) vµ hµm g(x1, ..., xn) lµ hµm f + g ®Þnh nghÜa nh­ sau

(f + g)(x1, ..., xn) = f(x1, ..., xn) + g(x1, ..., xn) ∀(x1, ..., xn) ∈ Bn

TÝch Boole cña hµm f(x1, ..., xn) vµ hµm g(x1, ..., xn) lµ hµm f ⋅ g ®Þnh nghÜa nh­ sau

(f ⋅ g)(x1, ..., xn) = f(x1, ..., xn) ⋅ g(x1, ..., xn) ∀(x1, ..., xn) ∈ Bn

• §Þnh lý 1. Sè hµm Boole bËc n lµ n22

Chøng minh Theo quy t¾c nh©n cã 2n bé n phÇn tö kh¸c nhau gåm c¸c sè 0 vµ sè 1. V× hµm Boole lµ sù g¸n 0 hoÆc 1 cho mçi bé trong sè 2n bé n phÇn tö ®ã, nªn l¹i theo

quy t¾c nh©n sÏ cã n22 c¸c hµm Boole kh¸c nhau. ®pcm.

B¶ng sau cho sè c¸c hµm Boole kh¸c nhau tõ bËc 1 ®Õn bËc 6

BËc Sè hµm Boole 1 4 2 16 3 256 4 65 536 5 4 294 967 296 6 18 446 744 073 709 551 616

TrÇn Quèc ChiÕn To¸n rêi r¹c

Ch­¬ng 7. §¹i sè Boole 7 − 8

7.2. biÓu diÔn c¸c hµm boole Hai bµi to¸n quan träng trong ®¹i sè Boole sÏ ®­îc nghiªn cøu trong bµi nµy. Bµi to¸n thø nhÊt lµ: cho c¸c gi¸ trÞ hµm Boole, lµm thÕ nµo t×m ®­îc biÓu thøc Boole biÓu diÔn hµm ®ã. Bµi to¸n nµy ®­îc gi¶i b»ng c¸ch chøng minh r»ng mäi hµm Boole ®Òu cã thÓ ®­îc biÓu diÔn b»ng tæng c¸c tÝch Boole cña c¸c biÕn vµ phÇn bï cña chóng. Lêi gi¶i bµi to¸n nµy chøng tá r»ng mäi hµm Boole ®Òu cã thÓ ®­îc biÓu diÔn b»ng c¸ch dïng ba to¸n tö Boole lµ tÝch ( ⋅ ), tæng ( + ) vµ bï ( ). Bµi to¸n thø hai lµ: liÖu cã thÓ dïng mét tËp to¸n tö nhá h¬n ®Ó biÓu diÔn c¸c hµm Boole hay kh«ng. Ta sÏ thÊy r»ng mäi hµm Boole ®Òu cã thÓ ®­îc biÓu diÔn b»ng c¸ch dïng chØ mét to¸n tö. C¸c bµi to¸n nµy cã tÇm quan träng thùc tiÔn trong viÖc thiÕt kÕ c¸c m¹ch. 7.2.1. C¸c d¹ng chuÈn t¾c XÐt vÝ dô sau. ◊ VÝ dô 1. T×m c¸c biÓu thøc Boole biÓu diÔn c¸c hµm f(x,y,z) vµ g(x,y,z) cã c¸c gi¸ trÞ cho trong b¶ng sau

x y z f g 1 1 1 1 0 0 0 0

1 1 0 0 1 1 0 0

1 0 1 0 1 0 1 0

0 0 1 0 0 0 0 0

0 1 0 0 0 1 0 0

Gi¶i CÇn ph¶i lËp biÓu thøc cã gi¸ trÞ 1 khi

x = z =1 & y = 0, vµ cã gi¸ trÞ 0 trong c¸c tr­êng hîp cßn l¹i, ®Ó biÓu diÔn hµm f. BiÓu thøc

x.y. z tho¶ m·n yªu cÇu nµy. §Ó biÓu diÔn hµm g ta cÇn biÓu thøc b»ng 1 khi

x = y = 1 & z = 0 hoÆc x = z = 0 & y = 1

TrÇn Quèc ChiÕn To¸n rêi r¹c

Ch­¬ng 7. §¹i sè Boole 7 − 9

vµ cã gi¸ trÞ 0 trong c¸c tr­êng hîp cßn l¹i. BiÓu thøc

x.y.z + x.y. z tho¶ m·n yªu cÇu nµy. • §Þnh nghÜa 1. Mét biÕn Boole hoÆc bï cña nã gäi lµ mét tôc biÕn. TÝch Boole

y1.y2.....yn trong ®ã

yi = xi hoÆc yi = xi ∀ i = 1, ..., n víi xi lµ c¸c biÕn Boole, ®­îc gäi lµ mét tiÓu h¹ng (minterm) Mét tiÓu h¹ng cã gi¸ trÞ 1 chØ khi

yi = 1 ∀ i = 1, ..., n tøc lµ

xi = 1, nÕu yi = xi & xi = 0, nÕu yi = xi ∀ i = 1, ..., n ◊ VÝ dô 2. T×m tiÓu h¹ng cã gi¸ trÞ b»ng 1 nÕu x1 = x3 = 0 vµ x2 = x4 = x5 = 1 vµ b»ng 0 trong c¸c tr­êng hîp cßn l¹i. Gi¶i Theo trªn, tiÓu h¹ng cÇn t×m lµ

x1. x2 . x3 . x4 . x5

B»ng c¸ch lÊy tæng Boole cña c¸c tiÓu h¹ng ph©n biÖt ta cã thÓ lËp ®­îc biÓu thøc Boole víi tËp c¸c gi¸ trÞ cho tr­íc. §Þnh lý sau kh¼ng ®Þnh mäi hµm Boole ®Òu cã thÓ biÓu diÔn b»ng tæng c¸c tiÓu h¹ng. • §Þnh lý 1. Cho hµm Boole cÊp n f(x1, ..., xn) ≠ 0. Gi¶ sö A1, ..., Ak ∈ Bn , B = {0, 1}, lµ c¸c bé gi¸ trÞ tho¶ f(Ai) = 1 ∀i=1, ..., k. Víi mçi Ai = (a1, ..., an), ta ®Æt

mi = y1 . ... . yn trong ®ã

yj =

==

0,1,

jj

jj

axax

, ∀j = 1, ..., n

Khi ®ã

f(x1, ..., xn) = m1 + m2 + ... + mk

Chøng minh

TrÇn Quèc ChiÕn To¸n rêi r¹c

Ch­¬ng 7. §¹i sè Boole 7 − 10

Víi mäi i =1, ..., k, ta ký hiÖu mi(a1, ..., an) gi¸ trÞ cña mi sau khi thay xj b»ng aj víi mçi j = 1, ..., n. Ta cã

mi(A) =

≠=

i

i

AAAA

,0,1

, ∀ i = 1, ..., k

Khi ®ã víi mäi A ∈ Bn ta cã

m1(A) + m2(A) + ... + mk(A) = 1, nÕu ∃i: A = Ai vµ

m1(A) + m2(A) + ... + mk(A) = 0, nÕu ∀i: A ≠ Ai Tõ ®ã suy ra ®Þnh lý. • §Þnh nghÜa 2. BiÓu diÔn

f(x1, ..., xn) = m1 + m2 + ... + mk

ë ®Þnh lý 1 gäi lµ d¹ng tuyÓn chuÈn t¾c cña hµm Boole f. ◊ VÝ dô 3. T×m d¹ng tuyÓn chuÈn t¾c cña hµm f(x,y,z) = (x + y). z . Gi¶i. Tr­íc tiªn ta t×m c¸c gi¸ trÞ cña hµm f. C¸c gi¸ trÞ cho ë b¶ng sau

x y z x + y z (x + y). z 1 1 1 1 0 0 0 0

1 1 0 0 1 1 0 0

1 0 1 0 1 0 1 0

1 1 1 1 1 1 0 0

0 1 0 1 0 1 0 1

0 1 0 1 0 1 0 0

TiÕp theo ta t×m c¸c tiÓu h¹ng t­¬ng øng víi c¸c bé gi¸ trÞ cho biÓu thøc gi¸ trÞ 1. Ta cã

m1 = x.y. z , øng víi hµng thø 2 m2 = x. y . z , øng víi hµng thø 4 m3 = x .y. z , øng víi hµng thø 6 VËy

f(x,y,z) = x.y. z + x. y . z + x .y. z • §Þnh nghÜa 3. Tæng Boole

TrÇn Quèc ChiÕn To¸n rêi r¹c

Ch­¬ng 7. §¹i sè Boole 7 − 11

y1 + y2 + ... + yn

trong ®ã yi = xi hoÆc yi = xi ∀ i = 1, ..., n

víi xi lµ c¸c biÕn Boole, ®­îc gäi lµ mét ®¹i h¹ng (maxterm) Mét ®¹i h¹ng cã gi¸ trÞ 0 chØ khi

yi = 0 ∀ i = 1, ..., n tøc lµ

xi = 0, nÕu yi = xi & xi = 1, nÕu yi = xi ∀ i = 1, ..., n §Þnh lý sau kh¼ng ®Þnh mäi hµm Boole ®Òu cã thÓ biÓu diÔn b»ng tÝch c¸c ®¹i h¹ng. • §Þnh lý 2. Cho hµm Boole cÊp n f(x1, ..., xn) ≠ 0. Gi¶ sö A1, ..., Ak ∈ Bn , B = {0, 1}, lµ c¸c bé gi¸ trÞ tho¶ f(Ai) = 0 ∀i=1, ..., k. Víi mçi Ai = (a1, ..., an), ta ®Æt

Mi = y1 + y2 + ... + yn trong ®ã

yj =

==

1,0,

jj

jj

axax

, ∀j = 1, ..., n

Khi ®ã

f(x1, ..., xn) = M1 . M2 . ... . Mk

Chøng minh Víi mäi i =1, ..., k, ta ký hiÖu Mi(a1, ..., an) gi¸ trÞ cña Mi sau khi thay xj b»ng aj víi mçi j = 1, ..., n. Ta cã

Mi(A) =

≠=

i

i

AAAA

,1,0

, ∀ i = 1, ..., k

Khi ®ã víi mäi A ∈ Bn ta cã

M1(A) . M2(A) . ... . Mk(A) = 0, nÕu ∃i: A = Ai vµ

M1(A) . M2(A) . ... . Mk(A) = 1, nÕu ∀i: A ≠ Ai Tõ ®ã suy ra ®Þnh lý. • §Þnh nghÜa 4. BiÓu diÔn

f(x1, ..., xn) = M2 . ... . Mk

ë ®Þnh lý 2 gäi lµ d¹ng héi chuÈn t¾c cña hµm Boole f.

TrÇn Quèc ChiÕn To¸n rêi r¹c

Ch­¬ng 7. §¹i sè Boole 7 − 12

◊ VÝ dô 4. T×m d¹ng héi chuÈn t¾c cña hµm f(x,y,z) = (x + y). z . Gi¶i. Tr­íc tiªn ta t×m c¸c gi¸ trÞ cña hµm f. C¸c gi¸ trÞ cho ë b¶ng sau

x y z x + y z (x + y). z 1 1 1 1 0 0 0 0

1 1 0 0 1 1 0 0

1 0 1 0 1 0 1 0

1 1 1 1 1 1 0 0

0 1 0 1 0 1 0 1

0 1 0 1 0 1 0 0

TiÕp theo ta t×m c¸c ®¹i h¹ng t­¬ng øng víi c¸c bé gi¸ trÞ cho biÓu thøc gi¸ trÞ 0. Ta cã

M1 = x + y + z , øng víi hµng thø 1 M2 = x + y + z , øng víi hµng thø 3 M3 = x + y + z , øng víi hµng thø 5 M4 = x + y + z , øng víi hµng thø 7 M5 = x + y + z , øng víi hµng thø 8 VËy

f(x,y,z) = ( x + y + z ).( x + y + z ).(x + y + z ).(x + y + z ).(x + y + z) 7.2.2. TÝnh ®Çy ®ñ KÕt qu¶ cña môc trªn cho thÊy mäi hµm Boole cã thÓ biÓu diÔn b»ng c¸c phÐp to¸n Boole +, . , . • §Þnh nghÜa 1. Mét tËp hîp c¸c phÐp to¸n Boole gäi lµ ®Çy ®ñ nÕu mäi hµm Boole ®Òu cã thÓ biÓu diÔn b»ng c¸c phÐp to¸n cña nã.

Nh­ vËy, ta cã • §Þnh lý 1. TËp hîp 3 phÐp to¸n { +, . , } lµ ®Çy ®ñ. Ta cã thÓ t×m ®­îc tËp ®Çy ®ñ c¸c phÐp to¸n nhá h¬n kh«ng ? Ta cã thÓ lµm ®­îc ®iÒu ®ã nÕu mét trong ba phÐp to¸n trªn cã thÓ biÓu diÔn qua hai phÐp to¸n cßn l¹i. §iÒu nµy cã thÓ lµm ®­îc nÕu ta sö dông luËt de Morgan.

Tr­íc tiªn, ta cã thÓ lo¹i bá tæng Boole + b»ng c¸ch dïng ®¼ng thøc

x + y = yx.

TrÇn Quèc ChiÕn To¸n rêi r¹c

Ch­¬ng 7. §¹i sè Boole 7 − 13

(suy ra tõ ®¼ng thøc de Morgan thø nhÊt yx + = x ⋅y ) Nh­ vËy, ta cã

• §Þnh lý 2. TËp hîp 2 phÐp to¸n { . , } lµ ®Çy ®ñ.

T­¬ng tù, ta cã thÓ lo¹i bá tÝch Boole . b»ng c¸ch dïng ®¼ng thøc

x • y = yx + (suy ra tõ ®¼ng thøc de Morgan thø nhÊt yx ⋅ = x + y )

§iÒu nµy chøng minh

• §Þnh lý 3. TËp hîp 2 phÐp to¸n { + , } lµ ®Çy ®ñ. ◊ Chó ý: TËp { +, • } kh«ng ®Çy ®ñ, v× kh«ng thÓ biÓu diÔn x b»ng hai phÐp to¸n +, • (bµi tËp). LiÖu cã tån t¹i tËp ®Çy ®ñ chØ cã 1 phÐp to¸n kh«ng ? C©u tr¶ lêi lµ tån t¹i. Ta sÏ x©y dùng c¸c phÐp to¸n nh­ vËy. • §Þnh nghÜa 2. PhÐp to¸n ↑ hay NAND:

x1 ↑ x2 = x1 NAND x2 = ( )

≠∀=∀

)1,1(,,1)1,1(),(,0

21

21

xxxx

PhÐp to¸n ↓ hay NOR:

x1 ↓ x2 = x1 NOR x2 = ( )

≠∀=∀

)0,0(,,0)0,0(),(,1

21

21

xxxx

• §Þnh lý 4. C¸c tËp hîp 1 phÐp to¸n { ↑} vµ { ↓ } lµ ®Çy ®ñ. Chøng minh LËp b¶ng gi¸ trÞ cña c¸c biÓu thøc ta thÊy

x = x ↑ x vµ x.y = (x ↑ y) ↑ (x ↑ y) Nh­ vËy phÐp to¸n bï vµ nh©n • biÓu diÔn ®­îc b»ng phÐp to¸n ↑. MÆt

kh¸c tËp { • , } ®Çy ®ñ, suy ra tËp hîp 1 phÐp to¸n { ↑ } còng ®Çy ®ñ. T­¬ng tù ta cã thÓ chøng minh

x = x ↓ x , x.y = (x ↓ x) ↓ (y ↓ y) vµ x+y = (x ↓ y) ↓ (x ↓ y)

TrÇn Quèc ChiÕn To¸n rêi r¹c

Ch­¬ng 7. §¹i sè Boole 7 − 14

vµ suy ra tËp hîp 1 phÐp to¸n { ↓ } còng ®Çy ®ñ. bµi tËp

1. T×m c¸c gi¸ trÞ cña c¸c biÓu thøc sau

a) 1. 0 b) 1 + 1 c) 0 .0 d) 01+ 2. T×m c¸c gi¸ trÞ cña biÕn Boole x tho¶ c¸c ph­¬ng tr×nh sau a) x.1 = 0 b) x + x = 0 c) x.1 = x d) x. x = 1 3. T×m c¸c gi¸ trÞ cña c¸c biÕn Boole x vµ y tho¶

x.y = x + y 4. Cã bao nhiªu hµm Boole bËc 7 kh¸c nhau 5. Chøng minh a) x + x.y = x b) x. y + y. z + z. x = x .y + y .z + z .x To¸n tö Boole ⊕ , ®­îc gäi lµ to¸n tö XOR, ®­îc ®Þnh nghÜa nh­ sau

1 ⊕ 1 = 0 ⊕ 0 = 1, 1 ⊕ 0 = 0 ⊕ 1 =1 6. Rót gän c¸c biÓu thøc sau a) x ⊕ 0 b) x ⊕ 1 c) x ⊕ x d) x ⊕ x 7. Chøng minh a) x ⊕ y = (x + y). yx. b) x ⊕ y = x. y + x .y 8. C¸c ®¼ng thøc sau ®óng hay kh«ng ? a) x ⊕ y = y ⊕ x b) x ⊕ (y ⊕ z) = (x ⊕ y) ⊕ z c) x + (y ⊕ z) = (x + y) ⊕ (x + z) d) x ⊕ (y + z) = (x ⊕ y) + (x ⊕ z) 9. T×m c¸c ®èi ngÉu cña c¸c biÓu thøc sau a) x + y b) x . y c) x.y.z + x . y . z d) x. z + x.0 + x .1 10*. Cho hµm Boole F(x1, ..., xn). Chøng minh

Fd(x1, ..., xn) = ( )nxxF ,...,1 11*. Cho hµm Boole F(x1, ..., xn), G(x1, ..., xn). Chøng minh

F = D ⇒ Fd = Gd

12*. Cã bao nhiªu hµm Boole F(x,y,z) kh¸c nhau sao cho

F( x , y , z ) = F(x, y, z) ∀ x, y, z 13*. Cã bao nhiªu hµm Boole F(x,y,z) kh¸c nhau sao cho

TrÇn Quèc ChiÕn To¸n rêi r¹c

Ch­¬ng 7. §¹i sè Boole 7 − 15

F( x , y, z) = F(x, y , z) = F(x, y, z ) ∀ x, y, z 14. T×m tÝch Boole cña c¸c biÕn x, y, z hoÆc phÇn bï cña chóng, biÕt r»ng tÝch ®ã cã gi¸ trÞ 1 nÕu vµ chØ nÕu a) x = y = 0, z = 1 b) x = 0, y = 1, z = 0 c) x = 0, y = z = 1 d) x = y = z = 0 15. T×m khai triÓn tæng c¸c tÝch cña c¸c hµm Boole hai biÕn x, y a) x + y b) x. y c) 1 d) y 16. T×m khai triÓn tæng c¸c tÝch cña c¸c hµm Boole ba biÕn x, y, z a) x + y + z b) (x + z).y c) x d) x. y 17. T×m khai triÓn tæng c¸c tÝch cña c¸c hµm Boole F(x, y, z) biÕt F = 1 nÕu vµ chØ nÕu a) x = 0 b) x.y = 0 c) x + y = 0 d) x.y.z = 0 18. T×m khai triÓn tæng c¸c tÝch cña c¸c hµm Boole F(w, x, y, z) biÕt F = 1 nÕu vµ chØ nÕu mét sè lÎ cña w, x, y, z cã gi¸ trÞ 1. 20. T×m khai triÓn tæng c¸c tÝch cña c¸c hµm Boole F(v, w, x, y, z) biÕt F = 1 nÕu vµ chØ nÕu sè biÕn cã gi¸ trÞ 1 lín h¬n hoÆc b»ng 3.

TrÇn Quèc ChiÕn To¸n rêi r¹c

Môc lôc i

MôC LôC Ch­¬ng 1 : ng«n ng÷ to¸n häc 1 - 1 1.1. quy n¹p to¸n häc 1 - 1 1.1.1. Nguyªn lý quy n¹p to¸n häc 1 - 1 1.1.2. Bµi to¸n xÕp Tromino 1 - 1 • Bµi tËp 1 - 2 1.2. tËp hîp 1 - 3 1.2.1. C¸c kh¸i niÖm c¬ b¶n 1 - 3 1.2.2. C¸c phÐp to¸n tËp hîp 1 - 4 1.2.3. TÝch §Ò-c¸c 1 - 6 • Bµi tËp 1 - 6 1.3. quan hÖ 1 - 7 1.3.1. §Þnh nghÜa 1 - 7 1.3.2. Quan hÖ t­¬ng ®­¬ng vµ ph©n ho¹ch 1 - 7 1.3.3. Quan hÖ thø tù 1 - 8 • Bµi tËp 1 - 10 1.4. ¸nh x¹ 1 - 11 1.4.1. §Þnh nghÜa 1 - 11 1.4.2. C¸c phÐp to¸n ¸nh x¹ 1 - 13 • Bµi tËp 1 - 15 1.5. c«ng thøc truy håi 1 - 16 1.5.1. C«ng thøc truy håi 1 - 16 1.5.3. Gi¶i c«ng thøc truy håi b»ng ph­¬ng ph¸p lÆp 1 - 16 1.5.4. Gi¶i c«ng thøc truy håi b»ng ph­¬ng tr×nh ®Æc tr­ng 1 - 17 • Bµi tËp 1 - 19 1.6. hÖ sè nhÞ thøc 1 - 20 • Bµi tËp 1 - 21

Ch­¬ng 2 : NHẬP M¤N Tæ HîP 2 - 1 2.1. S¥ L¦îC LÞCH Sö 2 - 1 2.2. BµI TO¸N Tæ HîP 2 - 2 2.2.1. Bµi to¸n th¸p Hµ néi 2 - 2 2.2.2. Bµi to¸n n cÆp vî chång 2 - 2 2.2.3. Bµi to¸n ®­êng ®i qu©n ngùa 2 - 3 2.2.4. Bµi to¸n h×nh vu«ng La-tinh 2 - 4 2.2.5. Bµi to¸n h×nh lôc gi¸c thÇn bÝ 2 - 4 Ch­¬ng 3: bµi to¸n tån t¹i 3 - 1 3.1. mét sè vÝ dô 3 - 1 3.1.1. Bµi to¸n 36 sÜ quan 3 - 1 3.1.2. Bµi to¸n 2n ®iÓm trªn l­íi n x n ®iÓm 3 - 2 3.2. nguyªn lý dirichlet 3 - 3 3.2.1. Nguyªn lý Dirichlet 3 - 3 3.2.2. Nguyªn lý Dirichlet tæng qu¸t 3 - 3 BµI TËP 3 - 5

TrÇn Quèc ChiÕn To¸n rêi r¹c

Môc lôc ii

Ch­¬ng 4 : bµi to¸n ®Õm 4 - 1 4.1. C¸C nguyªn lý c¬ b¶n 4 - 1 4.1.1. Nguyªn lý nh©n 4 - 1 4.1.2. Nguyªn lý céng 4 - 2 4.2. c¸c cÊu h×nh tæ hîp c¬ b¶n 4 - 4 4.2.1. ChØnh hîp lÆp 4 - 4 4.2.2. ChØnh hîp kh«ng lÆp 4 - 4 4.2.3. Ho¸n vÞ 4 - 5 4.2.4. Tæ hîp 4 - 5 4.2.5. Ho¸n vÞ lÆp 4 - 6 4.2.6. Tæ hîp lÆp 4 - 6 4.3. mét sè bµi tËp øng dông 4 - 8 4.3.1. Bµi to¸n ®Õm c¸ch xÕp chç 4 - 8 4.3.2. Bµi to¸n ®Õm sè ®­êng ®i 4 - 9 4.3.3. ¸p dông c«ng thøc truy håi 4 - 11 4.4. nguyªn lý bï trõ 4 - 13 4.4.1. Nguyªn lý 4 - 13 4.4.2. Bµi to¸n øng dông 4 - 14 bµi tËp 4 - 18 ch­¬ng 5 : bµi to¸n liÖt kª 5 - 1 5.1. ph¸t biÓu bµi to¸n liÖt kª 5 - 1 5.2. ph­¬ng ph¸p sinh 5 - 2 5.2.1. Thø tù tõ ®iÓn vµ ph­¬ng ph¸p sinh 5 - 2 5.2.2. D·y nhÞ ph©n ®é dµi n 5 - 3 5.2.3. Tæ hîp chËp r tõ n phÇn tö 5 - 4 5.2.4. Ho¸n vÞ 5 - 5 5.3. thuËt to¸n quay lui 5 - 7 5.3.1. Néi dung thuËt to¸n 5 - 7 5.3.2. LiÖt kª d·y nhÞ ph©n ®é dµi n 5 - 8 5.3.3. LiÖt kª ho¸n vÞ 5 - 9 5.3.4. Tæ hîp chËp r tõ n phÇn tö 5 - 10 5.3.5. Bµi to¸n xÕp HËu 5 - 11 5.3.6. Bµi to¸n h×nh ch÷ nhËt La tinh 5 - 13 bµi tËp 5 - 15

Ch­¬ng 6: bµi to¸n tèi ­u 6 - 1 6.1. Giíi thiÖu 6 - 1 6.1.1. Ph¸t biÓu bµi to¸n tèi ­u 6 - 1 6.1.2. Mét sè vÝ dô 6 - 1 6.2. thuËt to¸n johnson gi¶i bµi to¸n

lËp lÞch gia c«ng 2 m¸y 6 - 5 Ch­¬ng 7: ®¹i sè boole 7 - 1 7.1. ®¹i sè boole 7 - 1 7.1.1. §¹i sè Boole 7 - 1

TrÇn Quèc ChiÕn To¸n rêi r¹c

Môc lôc iii

7.1.2. Hµm Boole 7 - 6 7.2. biÓu diÔn hµm boole 7 - 8 7.2.1. C¸c d¹ng chuÈn t¾c 7 - 8 7.2.2. TÝnh ®Çy ®ñ 7 - 12 bµi tËp 7 - 14 Ch­¬ng 8: m¹ch tæ hîp 8 - 1 8.1. m¹ch tæ hîp 8 - 1 8.1.1. §Þnh nghÜa 8 - 1 8.1.2. Tæ hîp c¸c cæng 8 - 2 8.2. cùc tiÓu ho¸ m¹ch 8 - 4 8.2.1. Bµi to¸n cùc tiÓu ho¸ m¹ch 8 - 4 8.2.2. Ph­¬ng ph¸p b¶n ®å Karnaugh 8 - 5 8.2.3. Ph­¬ng ph¸p Quine-McCluskey 8 - 10 8.3. øng dông 8 - 14 8.3.1. M¹ch biÓu quyÕt theo ®a sè 8 - 14 8.3.2. M¹ch ®iÒu khiÓn nhiÒu c«ng t¾c 8 - 16 bµi tËp 8 - 19 tµi liÖu tham kh¶o